Hechos Geométricos en el Triángulo (2013)

Angel Montesdeoca

(Última actualización: )

Hechos Geométricos en el Triángulo de:
(2014) (2015) (2016) (2017) (2018) (2019) (2020) (2021) (2022) (2023) (2024)

Cómo es el enlace a un Hecho Geométrico correspondiente a un día concreto:
http://amontes.webs.ull.es/otrashtm/HGT2013.htm#HGddmmaa
EJEMPLO: Domingo, 3 de noviembre del 2013
http://amontes.webs.ull.es/otrashtm/HGT2013.htm#HG031113

  • sábado, 28 de diciembre del 2013

    Construcción del triángulo: ha ma A=B

    ( Construcción de triángulos #174)

    Construir un triángulo sabiendo que es isósceles y conociendo la altura y la mediana desde uno de los vértices de ángulos iguales.

    ( Mostrar/Ocultar figura )

    Construimos el triángulo AHaMa tal que los lados AHa y AMa sean de las longitudes dadas ha y ma, respectivamente, y que el ángulo en Ha es recto.
    Reflejamos el punto A en Ma, obteniéndose el punto A'.
    Las tangentes desde A' a la parábola de foco A y directriz HaMa, cortan a la recta HaMa en los puntos C1 y C2. Las reflexiones de estos puntos en Ma dan los puntos B1 y B2.
    Los triángulos AB1C1 y AB2C2 son dos soluciones al problema propuesto.

  • jueves, 19 de diciembre del 2013

    Cónica pasando por las reflexiones de un punto respecto a los lados de un cuadrivértice

    ( Quadri-Figures-Group #391 Randy Hutson)

    Sean ABC un triángulo, Q un punto, y el cuadrivértice ℘ = P1P2P3P4=GPaPbPc, formado por el baricentro y los vértices de triángulo antimedial.
    Consideremos las reflexiones de Q respecto de los lados del cuadrivértice ℘:

    Qij es la reflexión de Q en las rectas PiPj para todo i,j en {1,2,3,4}, i<j.

    El lugar geométrico de los puntos Q tales que sus reflexiones Qij, en los lados de ℘, estén en una misma cónica es una séptica que tiene a los vértices de ℘ como puntos singulares aislados, contiene a los vértices del triángulos diagonal y a los vértices(1) del triángulo de Miquel (QA-Tr2 ) de ℘, pasa por X316, Droussent pivot (QA-P4 "Isogonal Center") y tiene como punto del infinito al conjugado isogonal del foco X110 de la parábola de Kiepert: X523 ("Involutory Conjugate" de QA-P2=X99).

    ( Mostrar/Ocultar figura )

    Si Q=X316 la cónica que pasa por sus seis reflexiones respecto a los lados del cuadrivértice ℘ = GPaPbPc contiene a X316 y tiene por ecuación: Con centro es QA-P2=X99, punto de Steiner.

    (1) The vértices of QA-Tr2 can be constructed as the 2nd intersection point of the circles (QA-P4,Pi,Pj) and (QA-P4,Pk,Pl), where (i,j,k,l) = (1,2,3,4) / (1,3,2,4) / (1,4,2,3).

  • viernes, 6 de diciembre del 2013

    Una descripción geométrica del centro X691

    Sean ABC un triángulo y P un punto. Consideremos los dos triángulos equiláteros AAbAc y AA'bA'c tales que Ab y A'b están en la ceviana BP, Ac y A'c están en la ceviana CP (ver HG03113). Denotamos por A' el punto de intersección de AbAc y A'bA'c. Se definen cíclicamente los puntos B' y C'.
    Las rectas AA', BB', CC' son paralelas.


    Si P varía sobre la circunferencia circunscrita, el lugar geométrico del punto A' es una circunferencia Γa que pasa por A y su centro Oa tiene coordenadas baricéntricas:

    Oa = ( -2 (a^2 - b^2) (a^2 - c^2) (b^2 - c^2), b^2 (a^2 - b^2) (a^2 + b^2 - 2 c^2), c^2 (-a^2 + c^2) (a^2 - 2 b^2 + c^2) : ... : ... ).

    Similarmente, se obtienen sendas circunferencias Γb y Γc, con centros Ob y Oc , al proceder cíclicamente. Las rectas AOa, BOb, COc concurren en el centro X691.
    (a^2/((b^2-c^2)(2a^2-b^2-c^2)) : b^2/((c^2-a^2)(2b^2-c^2-a^2)) : c^2/((a^2-b^2)(2c^2-a^2-b^2)) ).

    Si P es un punto sobre la circunferencia circunscrita, sea Q el conjugado isogonal del punto del infinito determinado por las rectas paralelas AA', BB', CC'. Las rectas PQ son paralelas, cuando P varía; el punto del infinito que determina sus direcciones es X542 y el punto del infinito determinado por la dirección perpendicular a las de las rectas PQ es X690, cuyo conjugado isogonal es X691.
    ( Mostrar/Ocultar figura )

    Las tangentes en A, B, C a las circunferencias Γa, Γb, Γc, respectivamente concurren en X842, antipodal de X691.

  • miércoles, 4 de diciembre del 2013

    Puntos conjugados isogonales y centros ortológicos

    ( Anopolis #1135 Antreas P. Hatzipolakis)

    Sean ABC un triángulo, P un punto, P* su conjugado isogonal, A'B'C' y A"B"C" los triángulos pedales de P y P*, respectivamente.
    Denotamos por:
    Ab la intersección de la paralela a AP* por A' con AB.
    Ac la intersección de la paralela a AP* por A' con AC.
    A2 la intersección de la paralela a AP por A" con AB.
    A3 la intersección de la paralela a AP por A" con AC.
    Los puntos Ab, Ac, A2 y A3 están en una circunferencia, por construcción. Sea Oa su centro. Similarmente se definen Ob y Oc.

    El lugar geométrico de los puntos P tales que los triángulos ABC y OaObOb son ortológicos es una curva algebraica de grado seis, invariante por isoconjugación, que pasa por el circuncentro, ortocentro, por los puntos X1113 y X1114 (en los que la recta de Euler corta a la circunferencia circunscrita) y por los conjugados de éstos, X2574, X2575, en la recta del infinito, de ecuación baricéntrica:

    a^4 c^6 x^3 y^3 - b^4 c^6 x^3 y^3 - a^2 c^8 x^3 y^3 + b^2 c^8 x^3 y^3 - a^4 b^4 c^2 x^4 y z + a^2 b^6 c^2 x^4 y z + a^4 b^2 c^4 x^4 y z - a^2 b^2 c^6 x^4 y z - a^6 b^2 c^2 x^3 y^2 z - a^4 b^4 c^2 x^3 y^2 z + 2 a^2 b^6 c^2 x^3 y^2 z + 3 a^4 b^2 c^4 x^3 y^2 z - 2 b^6 c^4 x^3 y^2 z - 3 a^2 b^2 c^6 x^3 y^2 z + b^4 c^6 x^3 y^2 z + b^2 c^8 x^3 y^2 z - 2 a^6 b^2 c^2 x^2 y^3 z + a^4 b^4 c^2 x^2 y^3 z + a^2 b^6 c^2 x^2 y^3 z + 2 a^6 c^4 x^2 y^3 z - 3 a^2 b^4 c^4 x^2 y^3 z - a^4 c^6 x^2 y^3 z + 3 a^2 b^2 c^6 x^2 y^3 z - a^2 c^8 x^2 y^3 z - a^6 b^2 c^2 x y^4 z + a^4 b^4 c^2 x y^4 z - a^2 b^4 c^4 x y^4 z + a^2 b^2 c^6 x y^4 z + a^6 b^2 c^2 x^3 y z^2 - 3 a^4 b^4 c^2 x^3 y z^2 + 3 a^2 b^6 c^2 x^3 y z^2 - b^8 c^2 x^3 y z^2 + a^4 b^2 c^4 x^3 y z^2 - b^6 c^4 x^3 y z^2 - 2 a^2 b^2 c^6 x^3 y z^2 + 2 b^4 c^6 x^3 y z^2 + a^8 c^2 x y^3 z^2 - 3 a^6 b^2 c^2 x y^3 z^2 + 3 a^4 b^4 c^2 x y^3 z^2 - a^2 b^6 c^2 x y^3 z^2 + a^6 c^4 x y^3 z^2 - a^2 b^4 c^4 x y^3 z^2 - 2 a^4 c^6 x y^3 z^2 + 2 a^2 b^2 c^6 x y^3 z^2 - a^4 b^6 x^3 z^3 + a^2 b^8 x^3 z^3 - b^8 c^2 x^3 z^3 + b^6 c^4 x^3 z^3 - 2 a^6 b^4 x^2 y z^3 + a^4 b^6 x^2 y z^3 + a^2 b^8 x^2 y z^3 + 2 a^6 b^2 c^2 x^2 y z^3 - 3 a^2 b^6 c^2 x^2 y z^3 - a^4 b^2 c^4 x^2 y z^3 + 3 a^2 b^4 c^4 x^2 y z^3 - a^2 b^2 c^6 x^2 y z^3 - a^8 b^2 x y^2 z^3 - a^6 b^4 x y^2 z^3 + 2 a^4 b^6 x y^2 z^3 + 3 a^6 b^2 c^2 x y^2 z^3 - 2 a^2 b^6 c^2 x y^2 z^3 - 3 a^4 b^2 c^4 x y^2 z^3 + a^2 b^4 c^4 x y^2 z^3 + a^2 b^2 c^6 x y^2 z^3 - a^8 b^2 y^3 z^3 + a^6 b^4 y^3 z^3 + a^8 c^2 y^3 z^3 - a^6 c^4 y^3 z^3 + a^6 b^2 c^2 x y z^4 - a^2 b^6 c^2 x y z^4 - a^4 b^2 c^4 x y z^4 + a^2 b^4 c^4 x y z^4=0.
    ( Mostrar/Ocultar figura )
    Si P=X3 o P=X4 el centro ortológico de OaObOb respecto a ABC es

    ( a^10 - 2 a^8 (b^2 + c^2) + a^6 (b^4 + c^4) - a^4 (b^2 - c^2)^2 (b^2 + c^2) + 2 a^2 (b^8 - b^6 c^2 - b^2 c^6 + c^8) - (b^2 - c^2)^4 (b^2 + c^2) : ... : ... ),

    que tiene (6-9-13)-número de búsqueda en ETC: 6.74586117499516838898883187
    Si P=X3 o P=X4 el centro ortológico de ABC respecto a OaObOb es

    ( a^2/(2 a^10 - 5 a^8 (b^2 + c^2) + 2 a^6 (2 b^4 + b^2 c^2 + 2 c^4)- 2 a^4 (b^2 - c^2)^2 (b^2 + c^2) + 2 a^2 (b^8 - b^6 c^2 - b^2 c^6 + c^8)- (b^2 - c^2)^4 (b^2 + c^2) ) : ... : ... ),

    que tiene (6-9-13)-número de búsqueda en ETC: 146.543260333090931926435953

  • sábado, 30 de noviembre del 2013

    Caracterización de la cuártica de Stammler

    ( Anopolis #1124 Antreas P. Hatzipolakis)

    Sean ABC un triángulo y Q un punto. El lugar geométrico de los puntos P tales que la tripolar de P y la recta QP* (P* conjugado isogonal de P) son perpendiculares es una cuártica C(Q) con puntos nodales en los vértices de ABC, que pasa por el baricentro y por los puntos de intersección, X1113 y X1114, de la recta de Euler con la circunferencia circunscrita.

    Si Q(p:q:r) (coordenadas baricéntricas) la ecuación de la cuártica C(Q) es:

    Esta cuártica es la transformada isogonal de la hipérbola rectangular H(Q) de ecuación:

    Las direcciones de sus asíntotas son las mismas para todo Q, dadas por los conjugados isogonales de X1113 y X1114.

    Casos particulares:

    Q el incentro. El lugar geométrico es la cuártica C(X1) que pasa por los centros X1, X2, X29, X280, X1113, X1114 y X1222, de ecuación:

    ( Mostrar/Ocultar figura )

    Q el baricentro. El lugar geométrico es la cuártica C(X2) que pasa por los centros X2, X4, X6, X253, X523, X1113, X1114, X1222, X2346, de ecuación:

    ( Mostrar/Ocultar figura )

    Q el circuncentro. El lugar geométrico es la cuártica de Stammler (Q066), que pasa por los centros X1, X2, X4, X254, X1113, X1114, X1138, X2184, X3223, X3346 y X3459, de ecuación:

    ( Mostrar/Ocultar figura )

    Q el ortocentro. El lugar geométrico es la recta de Euler (junto con los lados de ABC).



    Una propiedad geométrica de la quíntica de Euler-Morley:

    "La quíntica de Euler-Morley (Q003, Bernard Gibert) es el lugar geométrico de los puntos Q tales la cuártica C(Q) pasa por Q".

    ( Mostrar/Ocultar figura )

    En este caso, la hipérbola H(Q) conjugada isogonal de la cuártica C(Q) pasa, además de por Q y X6, por Q*, que ahora también está en C(Q).
    De esta hipérbola H(Q) conocemos tres puntos (X6, Q, Q*) y las direcciones de las asíntotas (dadas por los conjugados isogonales de los puntos, X1113 y X1114, en los que la recta de Euler corta a la circunferencia circunscrita), por tanto puede ser construida ( IIPPP )

    Más sobre la cúartica de Stammler:
    Triángulos semejantes y homólogos
    Cuártica de Stammler como lugar geométrico
    Cuártica de Stammler e hipérbola de Jerabek
    Cuárticas tipo Stammler
    Otra caracterización de la cuártica de Stammler



  • viernes, 29 de noviembre del 2013

    Ejes radicales de circunferencias centradas en los vértices y una centrada en el centro radical de las anteriores

    (ADGEOM #893, Dao Thanh Oai )

    a Lolilla, por su "cumple"

    Sean ABC un triángulo, A(Ra), B(Rb), C(Rc) circunferencias centradas en los vértices A, B, C, de radios Ra, Ra, Ra, respectivamente. Si P es el centro radical de estas circunferencias, consideremos la circunferencia P(Rp) centrada en P y de radio Rp y los ejes radicales ea, eb, ec de P(Rp) con las circunferencias A(Ra), B(Rb), C(Rc), respectivamente. Entonces:

    El triángulo A1B1C1, delimitado por ea, eb y ec es perspectivo con ABC

    ( Mostrar/Ocultar figura )
    El centro radical P de las circunferencias A(Ra), B(Rb), C(Rc) tiene coordenadas baricéntricas:

    P = ( a4 - a²(b²+c²-2Ra²+Rb²+Rc²) - (b²-c²)(Rb²-Rc²): ... : ... ).


    y el centro de perspectividad D de A1B1C1 y ABC es:

    D = ( 1/(-a^2 b^4 c^2 + b^6 c^2 - a^2 b^2 c^4 - 2 b^4 c^4 + b^2 c^6 + a^6 Ra^2 - 3 a^4 b^2 Ra^2 + 3 a^2 b^4 Ra^2 - b^6 Ra^2 - 3 a^4 c^2 Ra^2 - 2 a^2 b^2 c^2 Ra^2 + b^4 c^2 Ra^2 + 3 a^2 c^4 Ra^2 + b^2 c^4 Ra^2 - c^6 Ra^2 - a^2 b^2 Ra^4 + b^4 Ra^4 - a^2 c^2 Ra^4 - 2 b^2 c^2 Ra^4 + c^4 Ra^4 + 2 a^2 b^2 c^2 Rb^2 + 2 b^4 c^2 Rb^2 - 2 b^2 c^4 Rb^2 + 2 a^2 b^2 Ra^2 Rb^2 - 2 b^4 Ra^2 Rb^2 + 2 b^2 c^2 Ra^2 Rb^2 - a^2 b^2 Rb^4 + b^4 Rb^4 + b^2 c^2 Rb^4 + 2 a^2 b^2 c^2 Rc^2 - 2 b^4 c^2 Rc^2 + 2 b^2 c^4 Rc^2 + 2 a^2 c^2 Ra^2 Rc^2 + 2 b^2 c^2 Ra^2 Rc^2 - 2 c^4 Ra^2 Rc^2 - 4 b^2 c^2 Rb^2 Rc^2 - a^2 c^2 Rc^4 + b^2 c^2 Rc^4 + c^4 Rc^4 - a^6 Rp^2 + 3 a^4 b^2 Rp^2 - 3 a^2 b^4 Rp^2 + b^6 Rp^2 + 3 a^4 c^2 Rp^2 - 2 a^2 b^2 c^2 Rp^2 - b^4 c^2 Rp^2 - 3 a^2 c^4 Rp^2 - b^2 c^4 Rp^2 + c^6 Rp^2) : ... : ... ).


    Si se dejan fijas las circunferencias A(Ra), B(Rb), C(Rc) y se varía el radio Rp, el punto D queda en la hipérbola rectangular circunscrita a ABC que pasa por P. La ecuación de tal hipérbola es:

    (b^2 - c^2 + Rb^2 - Rc^2) (a^4 - a^2 b^2 - a^2 c^2 + 2 a^2 Ra^2 - a^2 Rb^2 - b^2 Rb^2 + c^2 Rb^2 - a^2 Rc^2 + b^2 Rc^2 - c^2 Rc^2) y z + ... = 0.



    Algunos casos particulares

    Hipérbola de Feuerbach   Ra² = bc(b+c-a)/(a+b+c), Rb² = ca(a -b+c)/(a+b+c), Rc² = ab(a+b-c)/(a+b+c).

    Hipérbola de Jerabek   Ra=Rb=Rc = R (radio de la circunferencia circunscrita).

    Hipérbola de Kiepert   Ra² = 1/9 (-a^2+2b^2+2c^2), Rb² = 1/9 (2a^2-b^2+2c^2), Rc² = 1/9 (2a^2+2b^2-c^2).

    Estos valores de los radios, tomados para las circunferencias centradas en los vértices, no son los únicos posibles para describir las citadas hipérbolas. Para la hipérbola de Feuerbach, por ejemplo, se ha tomado las distancias de los vértices al incentro, que está en dicha hipérbola; pero podemos tomar la distancia de los vértices a cualquier otro punto que esté en la hipérbola, como radios de tales circunferencias.

    Así, en general, si tomamos como radios de las circunferencia centradas en los vértices las distancias a un punto de coordenadas baricéntricas (u:v:w), la ecuación de la hipérbola donde está el punto D es:

    u((a^2-b^2+c^2)v - (a^2+b^2-c^2)w)yz + v((a^2+b^2-c^2)w - (-a^2+b^2+c^2)w)zx + w((-a^2+b^2+c^2)u - (a^2-b^2+c^2)v)xy = 0.



    Otros casos de circunferencias fijas centradas en los vértices

    Circunferencias de radios las longitudes de los lados opuestos. Le corresponde la hipérbola que pasa por los centros X4, P=X20, X253, X1249, X1294, X3346 y X3668, Su ecuación es:
    (b^2-c^2) (3 a^4 - 2 a^2 b^2 - b^4 - 2 a^2 c^2 + 2 b^2 c^2 - c^4)yz+ ... = 0.

    ( Mostrar/Ocultar figura )

    Circunferencias tangentes a los lados opuestos del vértice en el que está centrada. La correspondiente hipérbola rectangular contiene a los centros X4, X389, X577 y tiene por ecuación:
    a^4 (b^2 - c^2)(a^2 - b^2 - c^2)^2 (a^6 b^2 - 3 a^4 b^4 + 3 a^2 b^6 - b^8 + a^6 c^2 - 3 a^2 b^4 c^2 + 2 b^6 c^2 - 3 a^4 c^4 - 3 a^2 b^2 c^4 - 2 b^4 c^4 + 3 a^2 c^6 + 2 b^2 c^6 - c^8)yz+ ... = 0.

    Circunferencias de radios los ex-inradios El centro de perspectividad D está en la hipérbola que contiene a los centros X4, X282, X1034, X1490, X3176, X3341, X3347. Tiene en común con la cúbica de Thomson los seis puntos A, B, C, X4, X282 y X3341. Su ecuación es:
    a (a - b - c) (b - c) (a^6 - 2 a^5 b - a^4 b^2 + 4 a^3 b^3 - a^2 b^4 - 2 a b^5 + b^6 - 2 a^5 c - 2 a^4 b c + 2 a b^4 c + 2 b^5 c - a^4 c^2 + 2 a^2 b^2 c^2 - b^4 c^2 + 4 a^3 c^3 - 4 b^3 c^3 - a^2 c^4 + 2 a b c^4 - b^2 c^4 - 2 a c^5 + 2 b c^5 + c^6)yz+ ... = 0.



  • miércoles, 27 de noviembre del 2013

    Una caracterización de la cúbica de Thomson

    ( K002 Bernard Gibert)

    Sean ABC un triángulo, P un punto y A'B'C' el triángulo circunceviano de P. Las rectas BC, CA, AB intersecan a las rectas B'C', C'A', A'B' en 9 puntos, de los cuales 6 no están en la tripolar p del producto ceviano(1) de P y K (simediano); estos seis puntos quedan en una cónica C(P) cuya ecuación baricéntrica, si (u:v:w) son las coordenadas de P, es:

    Σ
    abc uvw xyz
    a^2v^2w^2(c^2v+b^2w)x^2 - uvw(2b^2c^2u^2 + a^2u(c^2v+b^2w) + a^4vw)yz = 0.
    ( Mostrar/Ocultar figura )
    El centro de la cónica C(P) es el punto Q, de coordenadas:

    Q = ( u(a^2v(a^2w-c^2(2v+w)) - b^2(2c^2u(v+w)+a^2w(v+2w))) : ... : ... ).

    El lugar geométrico de los puntos P tales que P, Q, K son distintos y están alineados es la cúbica de Thomson.

    ( Mostrar/Ocultar figura )

    Nota: Si el punto P está sobre la circunferencia circunscrita, su triángulo circunceviano no está definido y si P está sobre la tripolar del simediano (eje de Lemoine) la cónica C(P) degenera en el producto de dos rectas que se cortan en el simediano.
    ( Mostrar/Ocultar figura )


    Los puntos P (no situados en el eje de Lemoine) que son centros del triángulo y figuran actualmente en ETC, a los que corresponde centros Q de la cónica C(P), que también figuran en ETC, son los siguientes pares {P,Q}:

    {X1, X1001}, {X3, X3}, {X4, X6}, {X6, X5024}, {X513, X999}, {X523, X381}, {X2574, X3}, {X2575, X3}.

    El centro de la cónica C(X2) es:

    ( a^4-3a^2(b^2+c^2)-4b^2c^2 : b^4-3b^2(c^2+a^2)-4c^2a^2 : c^4-3c^2(a^2+b^2)-4a^2b^2 ),

    que tiene (6-9-13)-número de búsqueda en ETC: 3.332127762248135206469312965

    (1) Producto ceviano de P y U (o Cevapunto de P y U) es el tripolo de la polar de P (resp, U) en la cónica circunscrita de perspector U (resp. P).

  • martes, 26 de noviembre del 2013

    Incentro, exincentros y circunferencias concurrentes

    ( Anopolis #1120 Antreas P. Hatzipolakis)

    Sean ABC un triángulo, I el incentro y Ia, Ib, Ic los exincentros. Denotemos por N1, N2, N3, los centros de las circunferencias de los nueve puntos de los triángulos IBC, ICA, IAB y por Na, Nb, Nc, los centros de las circunferencias de los nueve puntos de los triángulos IaBC, IbCA, IcAB.

    Las circunferencias circunscritas a los triángulos N1BC, N2CA, N3AB, concurren en X502.

    Las circunferencias circunscritas a los triángulos NaBC, NbCA, NcAB, concurren en X5620, de coordenadas baricéntricas:

    ( (b+c)(a^6 - a^4(b^2+c^2) - a^2(b^4-3b^2c^2+c^4)- 2abc(b-c)^2(b+c) + (b-c)^4(b+c)^2) : ... : ... ).

    ( Mostrar/Ocultar figura )

    •  Información complementaria:
    Las rectas IaNa, IbNb y IcNc concurren en X191.
    Las rectas IaN1, IbN2 y IcN3 concurren en X5506.


    •  Randy Hutson da una construcción alternativa del punto X5620 en Anopolis #1132:

    Sea DEF el triángulo incentral (triángulo ceviano del incentro), Se considera el triángulo BCA' semejante a DEF, construido hacia el interior de ABC. El punto A' es el segundo puntos de intersección de la recta IaNa con la circunferencia circunscrita al triángulo NaBC.
    Se definen B' y C' cíclicamente. Entonces la rectas AA', BB', CC' concurren en X5620.

    Las coordenadas baricéntricas de A' son:
    A' = (2a^2(a+b)(a+c) : -(a+c)(a^3+a^2(b-c)+(b-c)^2(b+c)+a(b^2-b c-c^2)) : -(a+b)(a^3+a^2(c-b)+(b-c)^2(b+c)-a(b^2+b c-c^2)) ).

    ( Mostrar/Ocultar figura )


    •  Caso en que los triángulos semejantes al triángulo incentral son construidos externamente:
    Sea DEF el triángulo incentral (triángulo ceviano del incentro), Se considera el triángulo BCA" semejante a DEF, construido hacia el exterior de ABC.
    Se definen B" y C" cíclicamente. Entonces la rectas AA", BB", CC" concurren en X502. (Randy Hutson, 9/23/2011)

    ( Mostrar/Ocultar figura )
    Los triángulos A"B"C" y IaIbIc son perspectivos, con centro de perspectividad:

    Z = ( a(a^6+2a^5(b+c) + a^4(b^2+4b c+c^2) - a^2(b^4+2b^3c+b^2c^2+2b c^3+c^4) - 2a(b^5+b^4c-b^3c^2-b^2c^3+b c^4+c^5) - (b-c)^2(b+c)^4) : ... : ... )

    que tiene (6-9-13)-número de búsqueda en ETC: -10.786517909750622287908

    Randy Hutson ( Anopolis #1134) nota que este punto es el isogonal conjugado de X399 en el triángulo excentral, IaIbIc.

  • lunes, 25 de noviembre del 2013

    "Central line" y cuártica asociada

    (§ 1.3 Remark 5 Bernard Gibert.- Bicevian Conics)

    Sean ABC un triángulo, δ una recta, D su tripolo, M un punto en δ, C(M) la cónica biceviana de D y M (que pasa por los vértices de los triángulos cevianos DaDbDc y MaMbMc de D y M) y, finalmente, Mδ el centro de C(M).
    El lugar geométrico de Mδ, cuando M varía, es una cuártica que pasa por los puntos de intersección de δ con los lados de ABC.

    Si px+qy+rz=0 es la ecuación baricéntrica de la recta δ, su cuarto punto de intersección con la cuártica tiene de coordenadas:

    Q(δ) = ( (q - r) (2 q r + p (q + r)) : (r - p) (2 r p + q (r + p)) : (p - q) (2 p q + r (p + q)) )

    Este punto es el centro de la cónica C(M), donde M es el punto del infinito de la recta δ. La ecuación de la cónica C(M) es (Bernard Gibert.- Bicevian Conics § 1.3 (1) ):

    p(p-q)(r-p)x^2 + q(q-r)(p-q)y^2 + r(r-p)(q-r)z^2 + p(q-r)^2yz + q(r-p)^2zx + r(p-q)^2xy = 0.

    ( Mostrar/Ocultar figura )

    EJEMPLOS: Alguna "central line" y su correspondiente cuarto punto de intersección con la cuártica asociada. La "central line" la denotamos por Li si sus coeficientes con las coordenadas baricéntricas del centro del triángulo Xi.

    "central line" δ ecuación / nombre centro asociado Q(δ) nº de búsqueda
    L1 ax+by+cz=0 ( (b-c)(2bc+a(b+c)) :...:... ) 137.478425435838696608231248
    L3 a^2SAx+...=0 ( (b^2-c^2)SA(a^6(b^2+c^2)-2a^4(b^4+b^2c^2+c^4)+ a^2(b^2-c^2)^2(b^2+ c^2)+2b^2c^2(b^2-c^2)^2):...:... ) 34.10580608877537813269670609
    L4 SBSCx+ ... =0 X2525
    L7 (a-b+c)(a+b-c)x+ ... =0 X3700
    L75 "Antiorthic Axis" X4979
    L76 "Lemoine axis" ( a^2(b^2-c^2)(2a^2+b^2+c^2):..:... ) 15.27801656267128914852474955
    L99 (c^2-a^2)(a^2-b^2)x+ ... =0 X1648
    L190 (c-a)(a-b)x+ ... =0 X1647
    L514 "Nagel Line" ( (2a-b-c) (2a^2-2a(b+c)+b^2+c^2):..:... ) -41.5918336266397249053218231
    L525 "Euler Line" ( (2a^4-a^2(b^2+c^2)-(b^2-c^2)^2) (2a^8-2a^6(b^2+c^2))- a^4(b^4-4 b^2c^2+c^4)+(b^2-c^2)^2(b^4+c^4):..:... ) -2.926469376504842758944837077
    L648 x/((b^2-c^2)SA)+ ... =0 X1650
    L668 x/(a(b-c))+ ... =0 X1646
    L670 x/(a^2(b^2-c^2))+ ... =0 X1645
    L671 x/(2a^2-b^2-c^2)+ ... =0 X1649
    L850 "Brocard axis" ( a^2(a^2(b^2+c^2)-b^4-c^4) (2a^6- 2a^4(b^2+c^2)-a^2(b^4-4b^2c^2+c^4)+(b^2-c^2)^2(b^2+c^2)):..:... ) -3.75510528847191427972334279
    L3239 "Soddy line" ( (2a^3-a^2(b+c)-(b-c)^2(b+c)) (-2(a-b)(a-c)(a+b-c)^2(a-b+c)^2 + (b-c)(-a+b+ c)^2((a-b)(a+b-c)^2-(a-c)(a-b+c)^2)):..:... ) -3.2612053200015515644411874
    L3265 "VanAubel line" ( (2 a^6 - a^4 (b^2 + c^2) - (b^2 - c^2)^2 (b^2 + c^2)) (-2 (a - b) (a + b) (a - c) (a + c) (a^4 - (b^2 - c^2)^2)^2 + (b - c) (b + c) (-a^2 + b^2 + c^2)^2 (3 a^4 (b^2 - c^2) - (b^2 - c^2)^3 - 2 a^2 (b^4 - c^4))):..:... ) -2.740416687177399065884132982
    L3268 "Fermat axis" ( (2 a^6 - 2 a^4 (b^2 + c^2) - (b^2 - c^2)^2 (b^2 + c^2) + a^2 (b^4 + c^4)) (-2 (a - b) (a + b) (a - c) (a + c) (a^2 - a b + b^2 - c^2) (a^2 + a b + b^2 - c^2) (a^2 - b^2 - a c + c^2) (a^2 - b^2 + a c + c^2) - (b - c) (b + c) (b^2 - c^2) (-a^2 + b^2 - b c + c^2) (-a^2 + b^2 + b c + c^2) (-2 a^4 + (b^2 - c^2)^2 + a^2 (b^2 + c^2))):..:... ) -2.39457958479639858720527989
    L4373 x/(b+c-3a)+ ... =0 X4394


  • viernes, 22 de noviembre del 2013

    Punto en la circunferencia circunscrita

    ( r1958, Quim Castellsaguer)
    (Antreas Hatzipolakis)

    Sean un triángulo ABC, I el incentro; Na, Nb , Nc los centros de las circunferencias de los nueve puntos de IBC, ICA, IAB. Las circunferencias {ANbNc}, {BNcNa}, {CNaNb} concurren en un punto de la circunferencia circunscrita.


    El centro de la circunferencia de los nueve puntos del triángulo IBC tiene por coordenadas baricéntricas:

    Na = (-a (b + c) : a^2 + a b - (b + c)^2 : a^2 + a c - (b + c)^2).

    Permutando cíclicamente estas coordenadas se obtienen las de los puntos Nb y Nc.
    La ecuación de la circunferencia circunscrita al triángulo ANbNc es:

    c^2 x y + b^2 x z + a^2 y z + (x + y + z) (-(((a^4 c - a^3 b c - a^2 b^2 c + a b^3 c + 2 a^3 c^2 - b^3 c^2 + b^2 c^3 - 2 a c^4 + b c^4 - c^5) y)/( 2 (a^3 + a^2 b - a b^2 - b^3 + a^2 c - a b c + b^2 c - a c^2 + b c^2 - c^3))) - ((-a^4 b - 2 a^3 b^2 + 2 a b^4 + b^5 + a^3 b c - b^4 c + a^2 b c^2 - b^3 c^2 - a b c^3 + b^2 c^3) z)/( 2 (-a^3 - a^2 b + a b^2 + b^3 - a^2 c + a b c - b^2 c + a c^2 - b c^2 + c^3)))=0.

    Por permutación cíclica obtenemos las ecuaciones de las circunferencias circunscritas a los triángulos BNcNa y CNaNb.

    El punto de concurrencia de estas tres circunferencias es:

    (a/((b - c) (a^3 - a^2 (b + c) - a (b^2 + b c + c^2) + (b + c)^3)) : ... : ... ),

    que tiene (6-9-13)-número de búsqueda en ETC: 0.2921043592404663185080644011
    ( Mostrar/Ocultar figura )


  • jueves, 21 de noviembre del 2013

    Parábolas con focos los vértices del triángulo circunceviano del simediano y directrices las medianas

    Sean ABC un triángulo. Se construyen triángulos semejantes variables ABBa, rectángulo en B, y ACCa, rectángulo en C, tales que θ = ∠BABa = - ∠CACa (ángulos orientados). La envolvente de las rectas BaCa es una parábola ℘a.

    En efecto, la correspondencia Ba (θ) ↦ Ca (-θ), entre puntos de la recta perpendicular a AB por B y los de la recta perpendicular a AC por C, es una proyectividad; por lo que, las rectas BaCa envuelven un cónica, tangente a estas rectas y también al lado BC en un punto Ta, ya que BC. Finalmente, como los puntos del infinito de cada recta se corresponden, la recta del infinito es también tangente: se trata de una parábola.

    La ecuación baricéntrica de ℘a es:

    4 b²c²x² + SB²y² + SC²z² + 2 SB SC yz + 4 b²SB zx + 4 c² SCxy = 0,

    su directriz es la mediana por A y su foco es el punto D donde la simediana por A vuelve a cortar a la circunferencia circunscrita a ABC.

    Los puntos de tangencia de la parábola ℘a con las rectas perpendiculares a AB y a AC por B y C, respectivamente, están en la altura por A; el punto Ta de tangencia con el lado BC es:

    Ta = (0 : a^2 + b^2 - c^2 : -a^2 + b^2 - c^2).


    ( Mostrar/Ocultar figura )
      HG211113.png
    Descargar fichero GeoGebra

    Procediendo cíclicamente, se obtienen las parábolas ℘b y ℘c. Se verifica:

    Las parábolas ℘a, ℘b y ℘c son tangentes a los lados de ABC en puntos alineados, sobre la tripolar del ortocentro X4.

    Gracias a Viktor Kitaysky, por una corrección



  • miércoles, 20 de noviembre del 2013

    Bisectrices de un triángulo y los centros X(171) y X(238)

    Sean ABC un triángulo y Wa el pie de la bisectriz exterior en A. Denotamos por Ωa la única cónica circunscrita que tiene las mismas tangentes desde Wa que la circunferencia circunscrita, su ecuación baricéntrica es:

    b c x (y + z) + a^2 y z = 0.

    Es tangente en A al lado (y+z=0) del triángulo antimedial y pasa por el punto Ta de intersección de la bisectriz interior en A con la mediatriz de BC (cuarto punto de intersección de Ωa con la circunferencia circunscrita).

    Para la construcción de la cónica Ωa ver: Cónica: dado tres puntos y dos tangentes (PPPtt) o bien (PPPtP), (PPPtP(1)).

    ( Mostrar/Ocultar figura )

    Análogamente, procediendo cíclicamente sobre los vértices de ABC, se consideran la cónicas Ωb y Ωc. Tomamos el cuarto punto Qa de intersección de las cónicas Ωb y Ωc y similarmente definimos los puntos Qb y Qc, entonces:

    Los triángulos ABC y QaQbQc son perspectivos con centro de perspectividad el punto X238.

    (a (a^2 - b c) : b (b^2 - c a) : c (c^2 - a b))

    ( Mostrar/Ocultar figura )

    La tangente ta en Ta a la cónica Ωa es:
    b(b^2 c + 2 b c^2 + c^3) x + a^2 c^2 y + a^2 b^2 z =0.

    Al tomar, de forma análoga, las tangentes tb y tc, se forma un triángulo perspectivo con ABC y el centro de perspectividad es el punto X985:

    (a/(b^2+b c+c^2) : b/(c^2+c a+a^2) : c/(a^2+a b+b^2))

    ( Mostrar/Ocultar figura )


    •   La cónica Ωa puede ser descrita como un lugar geométrico de la siguiente forma:

    Sean M un punto variable sobre la circunferencia circunscrita y Wa el pie de la bisectriz exterior en A, entonces el punto de intersección de la recta MWa con la recta simétrica de AM, respecto a la bisectriz en A, describe la cónica Ωa.
    ( Mostrar/Ocultar figura )


    •   Un estudio similar se puede hacer si se considera el pie Va de la bisectriz interior en A:

    Sean M un punto variable sobre la circunferencia circunscrita y Va el pie de la bisectriz interior en A, entonces el punto de intersección de la recta MVa con la recta simétrica de AM, respecto a la bisectriz en A, describe la cónica Ψa.

    La cónica Ψa está circunscrita al triángulo ABC, es tangente en A al lado del triángulo antimedial y pasa por el punto T'a de intersección de la bisectriz exterior en A con la circunferencia circunscrita. Su ecuación baricéntrica es:

    b c x (y + z) - a^2 y z = = 0.

    Similarmente, se definen la cónicas Ψb y Ψc.
    Tomamos el cuarto punto Pa de intersección de las cónicas Ψb y Ψc y similarmente definimos los puntos Pb y Pc, entonces:

    Los triángulos ABC y PaPbPc son perspectivos con centro de perspectividad el punto X171.

    (a (a^2 + b c) : b (b^2 + c a) : c (c^2 + a b))

    ( Mostrar/Ocultar figura )


    La tangente t'a en T'a a la cónica Ψa es:
    b(-b^2 c + 2 b c^2 - c^3) x + a^2 c^2 y + a^2 b^2 z = 0.

    Al tomar, de forma análoga, las tangentes t'b y t'c, se forma un triángulo perspectivo con ABC y el centro de perspectividad es el punto X983:

    (a/(b^2-b c+c^2) : b/(c^2-c a+a^2) : c/(a^2-a b+b^2))

    ( Mostrar/Ocultar figura )


  • martes, 12 de noviembre del 2013

    Curva de Durán Loriga

    en recuerdo de Ricardo Mariño Caruncho, mi profesor.
    1912-2003, La Coruña
    Escritor, periodista, Licenciado en ciencias exactas, estudioso y admirador de la obra de Juan Jacobo Durán Loríga.
    Era hijo de Pedro Mariño Ortega el gran Arquitecto Municipal entre cuyas obras destaca el Ayuntamiento de La Coruña.
    Otras facetas de su vida además de las matemáticas son sus conocimientos lingüísticos:inglés,francés en este idioma escribe un libro "Geometrie Euclidianne Abstraite" publicación muy valorada en el campo matemático europeo.
    Obras suyas son además: Elementos de la geometría vectorial, Notas sobre Juan Jacobo Durán-Loríga y Estudio genealógico sobre Juan Jacobo Durán Loríga.

    Sea ABC un triángulo, para todo punto P situado en una recta d que pasa por el baricentro, el lugar geométrico de los puntos del plano del triángulo igualmente iluminados por tres focos de igual intensidad, colocados en sus vértices, que por otro de intensidad suma, situado en P, tiene una asíntota perpendicular a la recta d.


    El lugar geométrico descrito es el de los puntos Q del plano del triángulo, tales que el cuadrado de la distancia de Q a P sea media armónica de los cuadrados de las distancias de Q a sus vértices.

    3/QP² = 1/QA² + 1/QB² + 1/QC².

    El lugar geométrico es un curva algebraica de grado cinco, cuya ecuación consta de gran número de sumandos (junto con la recta del infinito).
    La curva corta a la recta del infinito en los puntos cíclicos (dobles) y en los puntos cuyas coordenadas baricéntricas, si (u:v:w) son las P, tienen la expresión:

    ( 3a^2(v-w) + (b^2-c^2)(2u-v-w) : 3b^2(w-u) + (c^2-a^2)(2v-w-u) : 3c^2(u-v) + (a^2-b^2)(2w-u-v) ).

    Este punto está en la dirección perpendicular a la de la recta GP.

    En el caso particular de P=G (baricentro), la quíntica se descompone en la recta del infinito y la cuártica de ecuación:

    a^2 b^2 c^2 x^4 - 2 b^4 c^2 x^4 - 2 b^2 c^4 x^4 + a^4 c^2 x^3 y - 4 a^2 b^2 c^2 x^3 y + b^4 c^2 x^3 y - 3 a^2 c^4 x^3 y + 9 b^2 c^4 x^3 y + 2 c^6 x^3 y - 2 a^4 c^2 x^2 y^2 + 2 a^2 b^2 c^2 x^2 y^2 - 2 b^4 c^2 x^2 y^2 + 11 a^2 c^4 x^2 y^2 + 11 b^2 c^4 x^2 y^2 - 11 c^6 x^2 y^2 + a^4 c^2 x y^3 - 4 a^2 b^2 c^2 x y^3 + b^4 c^2 x y^3 + 9 a^2 c^4 x y^3 - 3 b^2 c^4 x y^3 + 2 c^6 x y^3 - 2 a^4 c^2 y^4 + a^2 b^2 c^2 y^4 - 2 a^2 c^4 y^4 + a^4 b^2 x^3 z - 3 a^2 b^4 x^3 z + 2 b^6 x^3 z - 4 a^2 b^2 c^2 x^3 z + 9 b^4 c^2 x^3 z + b^2 c^4 x^3 z + a^6 x^2 y z - 6 a^4 b^2 x^2 y z + 8 a^2 b^4 x^2 y z - 3 b^6 x^2 y z - 6 a^4 c^2 x^2 y z + 8 a^2 b^2 c^2 x^2 y z + b^4 c^2 x^2 y z + 8 a^2 c^4 x^2 y z + b^2 c^4 x^2 y z - 3 c^6 x^2 y z - 3 a^6 x y^2 z + 8 a^4 b^2 x y^2 z - 6 a^2 b^4 x y^2 z + b^6 x y^2 z + a^4 c^2 x y^2 z + 8 a^2 b^2 c^2 x y^2 z - 6 b^4 c^2 x y^2 z + a^2 c^4 x y^2 z + 8 b^2 c^4 x y^2 z - 3 c^6 x y^2 z + 2 a^6 y^3 z - 3 a^4 b^2 y^3 z + a^2 b^4 y^3 z + 9 a^4 c^2 y^3 z - 4 a^2 b^2 c^2 y^3 z + a^2 c^4 y^3 z - 2 a^4 b^2 x^2 z^2 + 11 a^2 b^4 x^2 z^2 - 11 b^6 x^2 z^2 + 2 a^2 b^2 c^2 x^2 z^2 + 11 b^4 c^2 x^2 z^2 - 2 b^2 c^4 x^2 z^2 - 3 a^6 x y z^2 + a^4 b^2 x y z^2 + a^2 b^4 x y z^2 - 3 b^6 x y z^2 + 8 a^4 c^2 x y z^2 + 8 a^2 b^2 c^2 x y z^2 + 8 b^4 c^2 x y z^2 - 6 a^2 c^4 x y z^2 - 6 b^2 c^4 x y z^2 + c^6 x y z^2 - 11 a^6 y^2 z^2 + 11 a^4 b^2 y^2 z^2 - 2 a^2 b^4 y^2 z^2 + 11 a^4 c^2 y^2 z^2 + 2 a^2 b^2 c^2 y^2 z^2 - 2 a^2 c^4 y^2 z^2 + a^4 b^2 x z^3 + 9 a^2 b^4 x z^3 + 2 b^6 x z^3 - 4 a^2 b^2 c^2 x z^3 - 3 b^4 c^2 x z^3 + b^2 c^4 x z^3 + 2 a^6 y z^3 + 9 a^4 b^2 y z^3 + a^2 b^4 y z^3 - 3 a^4 c^2 y z^3 - 4 a^2 b^2 c^2 y z^3 + a^2 c^4 y z^3 - 2 a^4 b^2 z^4 - 2 a^2 b^4 z^4 + a^2 b^2 c^2 z^4=0.


    Si el triángulo ABC es equilátero (que suponemos de lado unidad) y P es su centro, la ecuación de la cuártica queda:

    SumaCiclica[x^4 - 2 x^3 (y + z) - 3 x^2 y z - 3 y^2 z^2]=0

    ( Mostrar/Ocultar figura )
    A esta cuártica se le ha dado por llamar curva de Durán-Loriga, en honor al matemático gallego que la estudió:
    DURÁN LORIGA, J. J. (1909): Sobre un problema de física, Revista de la Real Academia de Ciencias Exactas, Físicas y Naturales de Madrid, VIII: 242-271. (PDF)

    En este artículo la ecuación de esta cuártica viene expresada de la forma (α, β y γ las coordenadas baricéntricas de un punto cualquiera):

    S4 - 6ωS2 + 9Sαβγ + 3 ω2=0,
    S = α+β+γ      ω = αβ+βγ+γα.

    Durán-Loriga
    Juan Jacobo Durán Loriga
    (Fuente: http://www.culturagalega.org/albumdaciencia/detalle.php?id=246)


    En la gráfica que se expone a continuación se muestran de las quínticas DL(X1) y DL(X8) , que corresponde a los caso en que P es el incentro y el punto de Nagel, respectivamente. Sus asíntotas, que tienen la dirección del punto X3667, son perpendiculares a la recta X1X8, que pasa por el baricentro, X2.
    ( Mostrar/Ocultar figura )


  • lunes, 11 de noviembre del 2013

    Circunferencias con diámetros sobre los lados de un triángulo y la cúbica de Lucas

    Sean ABC un triángulo, P un punto no situado en sus lados y DEF el triángulo ceviano de P. Las circunferencias de diámetros AE y AF se vuelven a cortar en A'. Los puntos B' y C' se definen cíclicamente. Los triángulos ABC y A'B'C' son perspectivos si y solo si P está en cúbica de Lucas (K007). El centro de perspectividad queda en la cúbica de Darboux.

    ( Mostrar/Ocultar figura )

    Si P=(u:v:w), en coordenadas baricéntricas respecto a ABC, las coordenadas de A' son:

    ( u(SAu(v-w)² + SC(u+v)w² + SBv²(u+w)) : vw(SC(u+v)w + SAu(-v+w)) : vw(SAu(v-w) + SBv(u+w)) ).


    Si Ha, Hb y Hc son los ortocentros de los triángulos AEF, BFD y CDE, respectivamente, el centro de perspectividad de ABC y HaHbHc es también el punto Q, cuando P varía en la cúbica de Lucas. (Ver el mensaje ADGEOM #437 de Nikos Dergiades, y K645, en catálogo de cúbicas de Bernard Gibert).

  • domingo, 03 de noviembre del 2013

    Triángulos equiláteros con vértices en tres cevianas

    ( ADGEOM #693)

    Sean ABC un triángulo y P un punto.
    Consideremos los tres triángulos equiláteros AAbAc, BBcBa y CCaCb, tales que Ba y Ca están en la ceviana AP, Cb y Ab están en la ceviana BP, y Ac y Bc están en la ceviana CP.
    El punto Ab es la intersección de la recta BP con la recta pac que resulta de girar PC alrededor de A un ángulo de 60° en el sentido antihorario. El punto Ac es la intersección de la recta CP con la recta pab que resulta de girar BP alrededor de A un ángulo de 60° en el sentido horario. Procediendo cíclicamente, se construyen los puntos Bc, Ba, Ca y Cb.

    Los centros Oa, Ob y Oc de estos tres triángulos equiláteros están alineados con P

    ( Mostrar/Ocultar figura )
    Si (u:v:w) son las coordenadas baricéntricas de P, la recta L(P) que contiene a los centros de los triángulos equiláteros tiene ecuación:

    (a^2vw(v-w)-b^2w(v^2+uw+vw)+c^2v(w^2+uv+vw))x + (b^2wu(w-u)-c^2u(w^2+vu+wu)+a^2w(u^2+vw+wu))y + (c^2uv(u-v)-a^2v(u^2+wv+uv)+b^2u(v^2+wu+uv))z = 0

    Ocurre además que el triángulo A'B'C' delimitado por las rectas AbAc, BcBa y CaCb es equilátero.

    Cuando el punto P recorre la circunferencia circunscrita, la recta L(P) pasa por el punto fijo X110, foco de la parábola de Kiepert.
    ( Mostrar/Ocultar figura )

    Existe un único punto Ω, sobre la circunferencia circunscrita, para el cual las rectas AbAc, BcBa y CaCb son concurrentes, y sus coordenadas baricéntricas son:

    Ω = (1 / ((b²-c²)(3√‍3 b²c²SA+S(S²+9SA²))) : ... : ... ).

    Este centro tiene primera coordenada trilineal exacta en el triángulo de ETC: 0.00000387325998676449736 y ha sido incluido en ETC con el nombre X(5618) = "1st Montesdeoca Equilateral Triangles Point".

    [Peter Moses, December 4, 2013]. X5618 es el punto medio de X13 (punto de Fermat) y X5623 (cociente ceviano del punto del infinito de la recta de Euler y X13).
    X5623 es el punto de intersección de las rectas AbAc, BcBa, CaCb, cuando P=X5618.
    ( Mostrar/Ocultar figura )

    El lugar geométrico de los puntos P para los cuales las rectas AbAc, BcBa y CaCb son concurrentes es una curva algebraica de grado ocho cuya ecuación es bastante complicada.
    ( Mostrar/Ocultar figura )

    Invirtiendo los sentidos de los giros para construir el triángulo equilátero con un vértice en A y los otros dos vértices en las cevianas que no parten de A (así como para los que tienen uno de sus vértices en B o en C), obtenemos la descripción del punto X5619 = "2nd montesdeoca equilateral triangles point"

    X5619 = (1 / ((b²-c²)(3√‍3 b²c²SA-S(S²+9SA²))) : ... : ... ).

    Sea el punto A'b de intersección de BP con la recta qac que resulta de girar CP alrededor de A un ángulo de 60° en el sentido de las agujas del reloj. Y el punto A'c de intersección de CP con la recta qab que resulta de girar BP alrededor de A un ángulo de 60° en el sentido antihorario. El triángulo AA'bA'c resulta ser equilátero.
    Se definen los puntos B'c, B'a, C'a y C'b cíclicamente.

    X5619 es el único punto sobre la circunferencia circunscrita para el que las rectas A'bA'c, B'cB'a, C'aC'b son concurrentes.

    [Peter Moses, December 4, 2013]:   X5619 es el punto medio de X14 (segundo punto de Fermat) y X5624 (cociente ceviano del punto del infinito de la recta de Euler y X14).
    X5624 es el punto de intersección de las rectas A'bA'c, B'cB'a, C'aC'b, cuando P=X5619.
    ( Mostrar/Ocultar figura )

    La figura siguiente expone los triángulos AAbAc y AA'bA'c simultáneamente, para un punto P arbitrario en el plano.
    ( Mostrar/Ocultar figura )


  • sábado, 2 de noviembre del 2013

    In Memoriam (José María Pedret)

    ( TriangulosCabri #693, Ricardo Barroso )

    Sean ABC un triángulo y Q un punto. Encontrar los puntos P sobre la ceviana AQ tales que si Pb y Pc son los pies de las cevianas BP y CP, respectivamente, se cumpla que APb=APc.


    Los cuadrados de las distancias de A a Pb y a Pc son, respectivamente,

    (b^2 w^2)/(u + w)^2,      (c^2 v^2)/(u + v)^2.

    Por lo que, APb = APc si P está en una de las dos cónicas circunscritas a ABC con centro en el punto medio de BC y tangentes en A a las bisectrices interior y exterior, de ecuaciones respectivas:

    cy(x+z) + bz(x+y) = 0,    cy(x+z) - bz(x+y) = 0.

    Así, las solución del problema planteado son los dos puntos A1 y A2: segundos puntos de intersección de la ceviana AQ con cada una de las cónicas consideradas:

    A1 = (-(b+c)qr : q(cq+br) : r(cq+br)),    A2 = ((b-c)qr : q(cq-br) : r(cq-br)).

    En el caso de que Q=H (ortocentro) :

    A1 = (-a^4+(b^2-c^2)^2 : (a+b-c)(a-b+c)(a^2+b^2-c^2) : (a+b-c)(a-b+c)(a^2-b^2+c^2))
    A2 = (a^4-(b^2-c^2)^2 : (b+c-a)(a+b+c)(a^2+b^2-c^2) : (b+c-a)(a+b+c)(a^2-b^2+c^2))

    ( Mostrar/Ocultar figura )

    Construcción geométrica de los puntos A1 y A2:
    Aparte de la construcción descrita anteriormente, ya que tales cónicas pueden ser construidas (PPPtP o PPPtP(2)), podemos proceder como sigue:

    Si Qa es el pie de la ceviana AQ, sea L el conjugado armónico de Qa respecto a B y C, es decir. L es el punto donde la tripolar de Q (respecto a ABC) corta a la recta BC.
    La perpendicular por L a la bisectriz interior en A, corta a AB y AC en los pies de las cevianas BA1 y CA1.
    La perpendicular por L a la bisectriz exterior en A, corta a AB y AC en los pies de las cevianas BA2 y CA2.

    (Ver otra solución en el caso de Q el ortocentro de Ercole Suppa)



  • domingo, 27 de octubre del 2013

    Una caracterización geométrica de la cúbica de Lucas

    Sean ABC un triángulo, P un punto y DEF el triángulo ceviano de P.
    La transformación afín que lleva los puntos A, B y C en D, E y F, respectivamente, tiene sus rectas fijas (propias) perpendiculares si y solo si P está en la cúbica de Lucas (K007).

    Si las coordenadas baricéntricas de P, respecto a ABC, son (u:v:w) la matriz asociada a esta transformación afín es:

    0u(u+v)(v+w)u(u+w)(v+w)
    v(u+v)(u+w)0v(u+w)(v+w)
    w(u+v)(u+w)w(u+v)(v+w)0
    El punto fijo propio, correspondiente a la raíz λ1=-(v+w)(u^2+uv+uw+vw) del polinomio característico, es Q=(u(v+w) : v(w+u) : w(u+v)), complemento del conjugado isotómico de P, es decir el centro de la cónica inscrita de perspector P (§10.7.3 The perspector of a conic. Paul Yiu).

    Las otras dos raíces son:

    λ2,3 = 1/2 ((u+v)(u+w)(v+ w) ± (v+w)½ ((u+v)(u+ w)(u(v^2-w^2)+v(w^2-u^2)+w(u^2-v^2)))½)

    La condición necesaria y suficiente para que los puntos fijos (en la recta del infinito) correspondientes a las raíces λ2 y λ3, determinen dos direcciones perpendiculares es:

    (b^2+c^2-a^2)u(v^2+w^2) + (c^2+a^2-b^2)v(w^2+u^2) + (a^2+b^2-c^2)w(u^2+v^2)=0.

    Es decir, el punto P debe de estar en la cúbica de Lucas. En este caso, el punto fijo Q está sobre la cúbica de Thomson (K002).
    Las rectas fijas de la transformación afín coinciden con los ejes de la cónica inscrita de perspector P, ellas son paralelas a las asíntotas de la hipérbola (rectangular) circunscrita que pasa por P y Q.

    Comparar esta propiedad de la cúbica de Lucas con la nº 7 en la web de Bernard Gibert:
    "Let P be the perspector of an inscribed conic (C) with center Q. The circum-conic (C') passing through P and Q is a rectangular hyperbola if and only if P lies on the Lucas cubic".

    ( Mostrar/Ocultar figura )


    Nota:
    Cuando el punto P está en la recta del infinito, la transformación afín que lleva ABC en el triángulo ceviano DEF de P, es una homología, con centro en P y eje el de perspectividad de ABC y DEF.

  • miércoles, 23 de octubre del 2013

    La cúbica nodal de Tucker como lugar geométrico

    Sean ABC un triángulo, P un punto y DEF el triángulo ceviano de P.
    Consideremos los puntos U, V y W tales que:

    AU : UD = BV : VE = CW : WF = m : n

    Si (u:v:w) son las coordenadas baricéntricas de P, existen dos pares (m1,m1) y (m2,m2) reales o imaginarios:

    ( u^2(v+w) + v^2(w+u) + w^2(u+v) + 2uvw ∓ √[(u+v)(v+w)(w+u)] √[u(v-w)^2+v(w-u)^2+w(u-v)^2], 4uvw )

    para los cuales los puntos U, V y W están alineados. Las rectas que los contienen se cortan en el "crosspoint" de P y el baricentro.

    Existe un solo par (m,n), proporcional a (2,1), si P está en la cúbica nodal de Tucker. En este caso, el punto U coincide con la intersección de la recta AD y la paralela por el baricentro a BC, similarmente ocurre con los puntos V y W. Esta propiedad es la número 1 en K015.

    Al variar P sobre la cúbica K015 las rectas UVW son tangentes a la elipse inscrita de Steiner. En particular, los puntos de tangencia son los centros X(3163), X(115) y X(2482) si P coincide con los centros X(4240), X(5466) y X(5468), respectivamente.
    ( Mostrar/Ocultar figura )

    Más sobre la cúbica nodal de Tucker:
    Propiedades de la cúbica nodal de Tucker
    "The tripolar centroid and the Tucker nodal cubic"
    Otra descripción de las isocúbicas cónico-pivotales cK(#Q,X2)
    Transformación afín y triángulos cevianos



  • martes, 22 de octubre del 2013

    Construcción del homólogo de un punto mediante una homografía

    ( AdvancedPlaneGeometry #730, Bernard Gibert )

    Se considera la aplicación definida en términos de coordenadas baricéntricas, respecto a un triángulo ABC, por las ecuaciones:

    x' = (2SB SC - S²)x + (2SB² + S²)y + (2SC² + S²)z
    y' = (2SA² + S²)x + (2SA SC - S²)y + (2SC² + S²)z
    z' = (2SA² + S²)x + (2SB² + S²)y + (2SA SB - S²)z
    Se trata de una transformación lineal (homografía), ya que el determinante de la matriz asociada es proporcional a (a^2+b^2+c^2)^2S^4>0.

    Pares de puntos homólogos {P,P'}, centros del triángulo, son los siguientes:
    {X(2),X(5304)}, {X(4),X(2)}, {X(5),X(6)}, {X(10),X(614)}, {X(30),X(230)}, {X(64),X(1249)}, {X(140),X(5306)}, {X(511),X(3291)}, {X(512),X(647)}, {X(516),X(3011)}, {X(517),X(3290)}, {X(541),X(3018)}, {X(546),X(3815)}, {X(690),X(1637)}, {X(1499),X(523)}, {X(1503),X(468)}, {X(1513),X(385)}, {X(1514),X(858)}, {X(1538),X(44)}, {X(2780),X(2492)}, {X(2821),X(3310)}, {X(2883),X(25)}, {X(3309),X(650)}, {3566),X(2501)}, {X(3835),X(657)}, {X(3853),X(3054)}, {X(3861),X(3055)}.

    Los cuatro centros X(4), X(5), X(10) y X(511) son tales que no hay tres de ellos alineados, en un triángulo escaleno, ya que:
    Det[X(4), X(5), X(10)] = -(1/8) (a-b)(a-c)(b-c)(a-b-c)(a+b-c)(a-b+c)(a+b+c)^3,
    Det[X(4), X(5), X(511)] = -(1/8) (a^2-b^2)(a^2-c^2)(b^2-c^2)(a-b-c)^2(a+b-c)^2(a-b+c)^2(a+b+c)^2,
    Det[X(4), X(10), X(511)] = 1/4 (a-b)(a-c)(b-c)(a-b-c)(a+b-c)(a-b+c)(a+b+c)^3(ab+ac+bc),
    Det[X(5), X(10), X(511)] = abc(a-b)(a-c)(b-c)(a-b-c)(a+b-c)(a-b+c)(a+b+c)^2.

    En consecuencia, los cuatro pares de puntos homólogos {X(4),X(2)}, {X(5),X(6)}, {X(10),X(614)} y {X(511),X(3291)} determinan completamente la homografía.
    La construcción del homólogo de un punto se expone en un Applet GeoGebra:
    ( Mostrar/Ocultar figura )

    Bernard Gibert (en comunicación personal) obtiene que los puntos dobles de esta homografía son los de intersección (distintos de X(2)) de la hipérbola de Kiepert y la hipérbola equilátera que pasa por X(2), X(5) y X(6) y de asíntotas paralelas a las de la hipérbola de Jerabek.
    (Construcción de una hipérbola dado tres puntos y las direcciones de sus asíntotas (IIPPP)
    ( Mostrar/Ocultar figura )

    (Ver otro ejemplo de de construcción de puntos homólogos en una homografía, con Cabri)

  • sábado, 19 de octubre del 2013

    La isocúbica pivotal pK(X3926,X3926)

    (CL007: pK(W,W) cubics, parallel tripolars cubics, Bernard Gibert )

    a Kake, por su "cumple"

    Sean ABC un triángulo, A'B'C' su triángulo antipodal, P un punto y DEF el triángulo pedal de P.
    Designamos por D' el conjugado armónico de D respecto a B y C. Definimos los conjugados armónicos E' y F' similarmente.
    El punto A1 es la intersección del lado BC con la perpendicular a AD' por A'. De forma similar se definen B1 y C1.

    Los triángulos ABC y A1B1C1 son perspectivos si y solo si P queda en al cúbica de Darboux.

    El lugar geométrico de los centros de perspectividad de los triángulos ABC y A1B1C1, cuando P varía sobre la cúbica de Darboux, es la isocúbica pivotal pK(X3926,X3926), de ecuación baricéntrica:
    pK(X3926,X3926)
    Esta cúbica contiene a los centros X2, X69, X345, X348, X3926 y al cociente ceviano X3926/X2.
    ( Mostrar/Ocultar figura )

    Otra caracterización de la cúbica de Darboux:
    Ya que si P está en la cúbica de Darboux el triángulo pedal DEF de P es el triángulo ceviano de un punto P'; entonces, se tiene que los puntos D', E' y F' están alineados sobre la tripolar de P'. El recíproco también es cierto, es decir:

    "La cúbica de Darboux es el lugar geométrico de los puntos P, en el plano del triángulo ABC, tal que los conjugados armónicos (respecto a los vértices de ABC) de los vértices de su triángulo pedal están alineados."



    NOTA:

    Si se reemplaza el triángulo pedal DEF de P por el triángulo ceviano y hacemos la misma construcción anterior para este caso, resulta que los triángulos ABC y A1B1C1 son siempre perspectivos y el centro de perspectividad es el producto baricéntrico de P y X69.
    ( Mostrar/Ocultar figura )


  • lunes, 14 de octubre del 2013

    Baricentros de los triángulos antimediales

    ( AdvancedPlaneGeometry #709 Francisco Javier García Capitán )

    Resultado r1896 (Quim Castellsaguer):
    Los baricentros Ga, Gb, Gc de los triángulos antimediales de ABC y el baricentro G de ABC son concíclicos en una circunferencia de centro el mismo de la circunferencia de los nueve puntos.
    Otros centros sobre esta circunferencia son los de su intersección con el eje de Fermat: X381 (punto medio de X2X4) y X5465 (proyección ortogonal de X2 sobre el eje de Fermat).

    ( Mostrar/Ocultar figura )

    Los triángulos GaGbGc y ABC son inversamente semejantes. La matriz de coeficientes asociada a las ecuaciones de la semejanza, en coordenadas baricéntricas respecto a ABC, es:

    \left(
\begin{array}{ccc}
 2 a^2 b^2 c^2 & a^2 c^2 \left(a^2+2
   b^2-c^2\right) & a^2 b^2 \left(a^2-b^2+2
   c^2\right) \\
 b^2 c^2 \left(2 a^2+b^2-c^2\right) & 2 a^2
   b^2 c^2 & a^2 b^2 \left(-a^2+b^2+2
   c^2\right) \\
 -b^2 c^2 \left(-2 a^2+b^2-c^2\right) & a^2
   c^2 \left(-a^2+2 b^2+c^2\right) & 2 a^2
   b^2 c^2
\end{array}
\right)

    Esta transformación tiene tres puntos fijos correspondientes a las tres raíces del polinomio característico:

    (6a^2b^2c^2 - λ) (a^8b^2c^2 - a^6b^4c^2 - a^4b^6c^2 + a^2b^8c^2 - a^6b^2c^4 + 3a^4b^4c^4 - a^2b^6c^4 - a^4b^2c^6 - a^2b^4c^6 + a^2b^2c^8 - λ^2)=0.

    El punto correspondiente a la raíz λ=6a^2b^2c^2 es el centro de la semejanza inversa:

    S= (a^2(a^4 - 3a^2(b^2 + c^2) + 2b^4 - 11b^2c^2 +2c^4): ... : ... )


    Los puntos correspondientes a las otras dos raíces del polinomio característico son los puntos del infinito de la hipérbola equilátera de Jerabek. Por lo que las rectas fijas de la semejanza son las paralelas por S a las asíntotas de esta hipérbola.

    (Ver otro ejemplo de semejanza invesa)

  • viernes, 11 de octubre del 2013

    Una propiedad de la cúbica K172

    ( Anopolis #1072 Antreas P. Hatzipolakis)

    Sean ABC un triángulo, P un punto y DEF el triángulo pedal de P, Denotamos por E' el punto sobre BC tal que las circunferencias de los nueve puntos de ABC y A'B'C' son tangentes. Los puntos E' y F' se definen de forma similar, procediendo cíclicamente sobre los lados de ABC.
    Los triángulos ABC y D'E'F son perspectivos si y solo si P está sobre la cúbica de Darboux ( K008). El lugar de los centros de perspectividad están sobre la cúbica K172.

    ( Mostrar/Ocultar figura )


  • jueves, 3 de octubre del 2013

    Los puntos X(74), X(262) y X(3426) como centros de perspectividad de triángulos

    (ETC Clark Kimberling )

    a Silvia, por su "cumple"

    Sean ABC un triángulo, Ab el punto de intersección de la altura por B con la perpendicular a AB por A, y Ac el punto de intersección de la altura por C con la perpendicular a AC por A. Similarmente y cíclicamente se define los puntos Bc, Ba, Ca y Cb , entonces las rectas AbAc, BcBa y CaCb determinan un triángulo A'B'C' perspectivo con ABC, con centro de perspectividad en X262, de coordenadas baricéntricas:

    ( 1/(a^4 - a^2(b^2+c^2) - 2b^2c^2) : 1/(b^4 - b^2(c^2+a^2) - 2c^2a^2) : 1/(c^4 - c^2(a^2+b^2) - 2a^2b^2) ).

    ( Mostrar/Ocultar figura )

    El triángulo A"B"C" delimitado por las rectas BcCb, CaAc y AbBa es perspectivo con ABC, con centro de perspectividad en X3426, de coordenadas baricéntricas:

    (a^2/(5a^4 - 4a^2(b^2+c^2) - (b^2-c^2)^2) : b^2/(5b^4 - 4b^2(c^2+a^2) - (c^2-a^2)^2) : c^2/(5c^4 - 4c^2(a^2+b^2) - (a^2-b^2)^2) ).

    (Otra propiedad de X3426 en "El punto X(3426) como centro ortológico" )
    ( Mostrar/Ocultar figura )

    Si ahora consideramos el punto Mab de intersección de la mediatriz de AC con la perpendicular a AB por A y el punto Mac de intersección de la mediatriz de AB con la perpendicular a AC por A, las rectas AbAc y MabMac se cortan en un punto A'''.
    Similarmente, se definen los puntos B''' y C'''; entonces, los triángulos A"B"C" y A'''B'''C''' son perspectivos, con centro de perspectividad en X74, que es el cuarto punto de intersección de la circunferencia circunscrita a ABC y la hipérbola de Jerabek.
    ( Mostrar/Ocultar figura )


  • miércoles, 2 de octubre del 2013

    Un centro del triángulo asociado a polares de cónicas

    Con las notaciones dadas en "A question from Mathesis and Bicentric Pairs" :

    En el plano del triángulo ABC el lugar geométrico de los puntos P tales que las perpendiculares trazadas desde los vértices A, B, C respectivamente a las rectas PC, PA, PB son concurrentes en un punto Q es la cónica Φ1 circunscrita a ABC, que pasa por el punto de Tarry (X98) y por el conjugado isogonal P(40) del primer punto de Beltrami, P(2) (inverso del primer punto de Brocard, Ω1, respecto a la circunferencia circunscrita).
    El lugar de los puntos Q, cuando P varía en la cónica Φ1 es la cónica Φ2 circunscrita a ABC, que pasa por el punto de Tarry (X98) y por el conjugado isogonal U(40) del segundo punto de Beltrami, U(2) (inverso del segundo punto de Brocard Ω2, respecto a la circunferencia circunscrita).

    Cuando P=A, el punto Q (que denotamos por Ac) es la intersección de la perpendicular por A a AC con la altura por C.
    Cuando Q=A, el punto P (que denotamos por Ab) es la intersección de la perpendicular por A a AB con la altura por B.

    El lugar geométrico del punto D=PC ∩ QB, cuando P varía sobre la cónica Φ1, es una cónica, Γa, tangente en B y C a las perpendiculares a AB y AC, respectivamente; además pasa por Db=AC∩BAc y por Dc=AB∩CAb. Su ecuación baricéntrica respecto a ABC es:

    SA² x² + SBSC yz + b² SB zx + c² SC xy =0.

    Procediendo cíclicamente, resultan las cónicas Γb y Γc, lugares geométricos de los puntos E=PA ∩ QC y F=PB ∩ QA. Sus ecuaciones son:

    SB² y² + a² SA yz + SCSA zx + c² SC xy=0,    SC² z² + a² SA yz + b² SB zx + SASB xy =0


    Las polares de A, B y C respecto a las cónicas Γa, Γb y Γc, respectivamente, determinan un triángulo A'B'C perspectivo con ABC, con centro de perspectividad:

    ( 1/(2a²SA³ - b²c²SBSC) : 1/(2b²SB³ - c²a²SCSA) : 1/(2c²SC³ - a²b²SASB) ).

    Este centro tiene primera coordenada trilineal exacta en el triángulo de ETC: -5.73512334607375866920459161
    ( Mostrar/Ocultar figura )


  • martes, 1 de octubre del 2013

    El punto X(3426) como centro ortológico

    ( Resultat #1622 Quim Castellsaguer )

    Sean A'B'C' el triángulo antipodal de ABC; A"B"C" el triángulo ceviano del retrocentro R (X69 conjugado isotómico del ortocentro); D, E, F los puntos medios de A'A", B'B", C'C". Los triángulos ABC y DEF son ortológicos, y un centro de ortología es el punto de De Longchamps, X20.
    El otro centro de ortología es el centro X3426 = X(3)-VERTEX CONJUGATE OF X(6).

    Los triángulos circuncevianos del circuncentro y simediano son, respectivamente, A'B'C' y LMN; entonces las rectas A'L, B'M, C'N delimitan un triángulo A1B1C1 perspectivo con ABC, con centro de perspectividad en X3426.
    (Otra propiedad de X3426 en "Los puntos X(74), X(262) y X(3426) como centros de perspectividad de triángulos" )

    ( Mostrar/Ocultar figura )
    Nota justo después del X(3414) en ETC: ( http://faculty.evansville.edu/ck6/encyclopedia/ETCPart3.html#X3414 ) Let T be the vertex triangle of the circumcevian triangles, AUBUCU and AXBXCX, of U and X; viz., the sidelines of T are AUAX, BUBX, CUCX. Then T is perspective to ABC, and the perspector is the U-vertex conjugate of X.

  • lunes, 30 de septiembre del 2013

    "A question from Mathesis and Bicentric Pairs"

    ( AdvancedPlaneGeometry #674 Ercole Suppa )
    (J.Neuberg, Mathesis 1885, vol. 5, pag. 208, Questions 258, 259)

    En el plano del triángulo ABC el lugar geométrico de los puntos P tales que las perpendiculares trazadas desde los vértices A, B, C respectivamente a las rectas PC, PA, PB son concurrentes en un punto Q es la cónica Φ1 circunscrita a ABC, que pasa por el punto de Tarry (X98) y por el conjugado isogonal P(40) del primer punto de Beltrami, P(2) (inverso del primer punto de Brocard, Ω1, respecto a la circunferencia circunscrita).
    El lugar de los puntos Q, cuando P varía en la cónica Φ1 es la cónica Φ2 circunscrita a ABC, que pasa por el punto de Tarry (X98) y por el conjugado isogonal U(40) del segundo punto de Beltrami, U(2) (inverso del segundo punto de Brocard Ω2, respecto a la circunferencia circunscrita).

    El lugar geométrico de los puntos P tales que las perpendiculares trazadas desde los vértices A, B, C respectivamente a las rectas PB, PC, PA son concurrentes en un punto Q es la cónica Φ2 circunscrita a ABC, que pasa por el punto de Tarry (X98) y por el conjugado isogonal del segundo punto de Beltrami (inverso del segundo punto de Brocard respecto a la circunferencia circunscrita). El lugar de los puntos Q, cuando P varía en la cónica Φ2 es la cónica Φ1 circunscrita a ABC, que pasa por el punto de Tarry (X98) y por el conjugado isogonal del primer punto de Beltrami (inverso del primer punto de Brocard respecto a la circunferencia circunscrita).

    Las ecuaciones baricéntricas de las cónicas Φ1 y Φ2 son respectivamente:
    (a^2+b^2-c^2)y z+ (-a^2+b^2+c^2)z x + (a^2-b^2+c^2)x y =0,    (a^2-b^2+c^2)y z+ (a^2+b^2-c^2)z x + (-a^2+b^2+c^2)x y =0.

    ( Mostrar/Ocultar figura )


    •  Cuando P=X98, Q=U(40) y cuando P=P(40), Q=X98.

    •  Las cónicas Φ1 y Φ2 son congruentes, con ejes respectivos perpendiculares y el centro de giro que transforma una en otra es el punto de Vecten (X485).

    •  Los perspectores de las cónicas Φ1 y Φ2, (a^2+b^2-c^2 : -a^2+b^2+c^2 : a^2-b^2+c^2) y (a^2-b^2+c^2 : a^2+b^2-c^2 : -a^2+b^2+c^2) son el par bicéntrico P(45) y U(45).

    •  Los centros de las cónicas Φ1 y Φ2, son:

    (a^4 + 2a^2(b^2-2c^2) + b^4-4b^2c^2+3c^4 : ... : ...) y
    (a^4 + 2a^2(c^2-2b^2) + c^4-4b^2c^2+3b^4 : ... : ...)

    que forman un par bicéntrico, que no figura en la lista confeccionada por Clark Kimberling. La recta que pasa por los centro de las cónicas solamente contiene un centro de ETC: su punto del infinito, X3566.

    •  La recta P(40)U(40) vuelve a cortar a las cónicas Φ1 y Φ2, respectivamente, en los puntos de coordenadas baricéntricas:

    ((a^2-b^2)(b^2-c^2)(a^2+b^2-c^2) : (b^2-c^2)(c^2-a^2)(-a^2+b^2+c^2) : (c^2-a^2)(a^2-b^2)(a^2-b^2+c^2)),
    ((a^2-c^2)(c^2-b^2)(a^2-b^2+c^2) : (b^2-a^2)(a^2-c^2)(a^2+b^2-c^2) : (c^2-b^2)(b^2-a^2)(-a^2+b^2+c^2)),

    que forman un par bicéntrico, que no figura en la lista confeccionada por Clark Kimberling.

    •  Los polos de la recta P(40)U(40) respecto a las cónicas Φ1 y Φ2, respectivamente, tienen coordenadas baricéntricas:

    (a^8 - a^6(b^2+c^2) + a^4b^2c^2 + a^2(b^2-c^2)^3 - (b^2-c^2)^2(b^4+b^2c^2-c^4) : ... : ...),
    (a^8 - a^6(b^2+c^2) + a^4b^2c^2 - a^2(b^2-c^2)^3 - (b^2-c^2)^2(c^4+b^2c^2-b^4) : ... : ...),

    que forman un par bicéntrico, que no figura en la lista confeccionada por Clark Kimberling.

  • sábado, 14 de septiembre del 2013

    Pares bicéntricos deducidos de los triángulos pedal y simétrico de un punto respecto a un triángulo

    ( Anopolis #999 Antreas P. Hatzipolakis)

    Sean ABC un triángulo, P un punto, A'B'C' el triángulo pedal de P y A", B", C" las reflexiones de P en los lados BC, CA, AB, respectivamente.

    El centro radical Rp de las circunferencias de diámetros A'B", B'C", C'A" y el centro radical Sp de las circunferencias de diámetros A'C", B'A", C'B" forman un par bicéntrico

    Si (u:v:w) son las coordenadas baricéntricas de P:

    Rp = (a^2 (3 b^2 c^2 u^2 (c^2 v + b^2 w) + u (c^4 (a^2 + b^2 - c^2) v^2 + 2 b^4 (a^2 - b^2 + c^2) w^2 + b^2 c^2 (3 a^2 - 2 b^2 + 2 c^2) v w) + a^2 (a^2 - b^2 - c^2)v w (2 c^2 v + b^2 w)) : ... : ... ),

    Sp = (a^2 (3 b^2 c^2 u^2 (c^2 v + b^2 w) + u (2 c^4 (a^2 + b^2 - c^2) v^2 + b^2 c^2 (3 a^2 + 2 b^2 - 2 c^2) v w + b^4 (a^2 - b^2 + c^2) w^2) + a^2 (a^2 - b^2 - c^2) v w (c^2 v + 2 b^2 w) ) : ... : ... ).


    ( Mostrar/Ocultar figura )
    Algunos casos particulares (coordenadas baricéntricas):
    P=X1
    f(a,b,c)=a (a^2 (b + 2 c) + a (b^2 - c^2)-2 b^3 + 3 b c^2 - c^3)
    La recta que pasa por este par bicéntrico, sólo contiene al centro X65, su punto medio.

    P=X6
    f(a,b,c)=5 a^2 - 2 b^2
    La recta que pasa por este par bicéntrico, sólo contiene a los centros X523, su punto del infinito X1992, su punto medio.

  • viernes, 13 de septiembre del 2013

    Circunferencias asociadas a los puntos isodinámicos

    ( Anopolis #1000 Antreas P. Hatzipolakis)

    Sean ABC un triángulo, P un punto, A'B'C' el triángulo pedal de P y A", B", C" las reflexiones de P en los lados BC, CA, AB, respectivamente.

    Los seis puntos medios de los segmentos A'B", B'C", C'A", A'C", B'A", C'B" están sobre una cónica, ya que los lados opuestos del hexágono que ellos forman son paralelos.

    ( Mostrar/Ocultar figura )
    La cónica es una circunferencia si y sólo si P es uno de los dos puntos isodinámicos (X15 y X16).

    Si P=X6 es el simediano, la cónica tiene centro en este punto.
    Cuando P se mueve sobre el eje de Brocard, los centros de las cónicas están sobre la recta X5X6. Sobre la que está los centros de las dos cónicas que son circunferencias.

  • jueves, 12 de septiembre del 2013

    Rectas de Euler paralelas

    ( AdvancedPlaneGeometry #613 Dominik Burek)

    El lugar geométrico de los puntos P tales que las rectas de Euler de los triángulos ABC y IPP* son paralelas es una séxtica que pasa por A, B, C, (puntos dobles), por I, por los exincentros, por los conjugados isogonales de sus puntos.
    Ecuación baricéntrica: (Mostrar)

      -a^4 b c^4 x^4 y^2 + a^3 b^2 c^4 x^4 y^2 + a^2 b^3 c^4 x^4 y^2 - a b^4 c^4 x^4 y^2 + 2 a^3 b c^5 x^4 y^2 + b^4 c^5 x^4 y^2 - b^3 c^6 x^4 y^2 - 2 a b c^7 x^4 y^2 - b^2 c^7 x^4 y^2 + b c^8 x^4 y^2 + a^5 c^4 x^3 y^3 - a^4 b c^4 x^3 y^3 - a b^4 c^4 x^3 y^3 + b^5 c^4 x^3 y^3 + a^4 c^5 x^3 y^3 + 2 a^3 b c^5 x^3 y^3 + 2 a b^3 c^5 x^3 y^3 + b^4 c^5 x^3 y^3 - 3 a^3 c^6 x^3 y^3 + a^2 b c^6 x^3 y^3 + a b^2 c^6 x^3 y^3 - 3 b^3 c^6 x^3 y^3 - a^2 c^7 x^3 y^3 - 4 a b c^7 x^3 y^3 - b^2 c^7 x^3 y^3 + 2 a c^8 x^3 y^3 + 2 b c^8 x^3 y^3 - a^4 b c^4 x^2 y^4 + a^3 b^2 c^4 x^2 y^4 + a^2 b^3 c^4 x^2 y^4 - a b^4 c^4 x^2 y^4 + a^4 c^5 x^2 y^4 + 2 a b^3 c^5 x^2 y^4 - a^3 c^6 x^2 y^4 - a^2 c^7 x^2 y^4 - 2 a b c^7 x^2 y^4 + a c^8 x^2 y^4 - 2 a^4 b^3 c^2 x^4 y z + a^3 b^4 c^2 x^4 y z + 3 a^2 b^5 c^2 x^4 y z - a b^6 c^2 x^4 y z - b^7 c^2 x^4 y z - 2 a^4 b^2 c^3 x^4 y z + 4 a^3 b^3 c^3 x^4 y z - a^2 b^4 c^3 x^4 y z - 2 a b^5 c^3 x^4 y z + b^6 c^3 x^4 y z + a^3 b^2 c^4 x^4 y z - a^2 b^3 c^4 x^4 y z + 3 a^2 b^2 c^5 x^4 y z - 2 a b^3 c^5 x^4 y z - a b^2 c^6 x^4 y z + b^3 c^6 x^4 y z - b^2 c^7 x^4 y z + 3 a^5 b^2 c^2 x^3 y^2 z - 3 a^4 b^3 c^2 x^3 y^2 z - 3 a^3 b^4 c^2 x^3 y^2 z + 3 a^2 b^5 c^2 x^3 y^2 z + 2 a^5 b c^3 x^3 y^2 z - 5 a^4 b^2 c^3 x^3 y^2 z + 4 a^3 b^3 c^3 x^3 y^2 z - a^2 b^4 c^3 x^3 y^2 z - 4 a^4 b c^4 x^3 y^2 z - 6 a^3 b^2 c^4 x^3 y^2 z + a b^4 c^4 x^3 y^2 z - 3 b^5 c^4 x^3 y^2 z + 6 a^2 b^2 c^5 x^3 y^2 z - 4 a b^3 c^5 x^3 y^2 z + 3 b^4 c^5 x^3 y^2 z + 4 a^2 b c^6 x^3 y^2 z + 5 a b^2 c^6 x^3 y^2 z + 3 b^3 c^6 x^3 y^2 z - 2 a b c^7 x^3 y^2 z - 3 b^2 c^7 x^3 y^2 z + 3 a^5 b^2 c^2 x^2 y^3 z - 3 a^4 b^3 c^2 x^2 y^3 z - 3 a^3 b^4 c^2 x^2 y^3 z + 3 a^2 b^5 c^2 x^2 y^3 z - a^4 b^2 c^3 x^2 y^3 z + 4 a^3 b^3 c^3 x^2 y^3 z - 5 a^2 b^4 c^3 x^2 y^3 z + 2 a b^5 c^3 x^2 y^3 z - 3 a^5 c^4 x^2 y^3 z + a^4 b c^4 x^2 y^3 z - 6 a^2 b^3 c^4 x^2 y^3 z - 4 a b^4 c^4 x^2 y^3 z + 3 a^4 c^5 x^2 y^3 z - 4 a^3 b c^5 x^2 y^3 z + 6 a^2 b^2 c^5 x^2 y^3 z + 3 a^3 c^6 x^2 y^3 z + 5 a^2 b c^6 x^2 y^3 z + 4 a b^2 c^6 x^2 y^3 z - 3 a^2 c^7 x^2 y^3 z - 2 a b c^7 x^2 y^3 z - a^7 c^2 x y^4 z - a^6 b c^2 x y^4 z + 3 a^5 b^2 c^2 x y^4 z + a^4 b^3 c^2 x y^4 z - 2 a^3 b^4 c^2 x y^4 z + a^6 c^3 x y^4 z - 2 a^5 b c^3 x y^4 z - a^4 b^2 c^3 x y^4 z + 4 a^3 b^3 c^3 x y^4 z - 2 a^2 b^4 c^3 x y^4 z - a^3 b^2 c^4 x y^4 z + a^2 b^3 c^4 x y^4 z - 2 a^3 b c^5 x y^4 z + 3 a^2 b^2 c^5 x y^4 z + a^3 c^6 x y^4 z - a^2 b c^6 x y^4 z - a^2 c^7 x y^4 z - a^4 b^4 c x^4 z^2 + 2 a^3 b^5 c x^4 z^2 - 2 a b^7 c x^4 z^2 + b^8 c x^4 z^2 + a^3 b^4 c^2 x^4 z^2 - b^7 c^2 x^4 z^2 + a^2 b^4 c^3 x^4 z^2 - b^6 c^3 x^4 z^2 - a b^4 c^4 x^4 z^2 + b^5 c^4 x^4 z^2 + 2 a^5 b^3 c x^3 y z^2 - 4 a^4 b^4 c x^3 y z^2 + 4 a^2 b^6 c x^3 y z^2 - 2 a b^7 c x^3 y z^2 + 3 a^5 b^2 c^2 x^3 y z^2 - 5 a^4 b^3 c^2 x^3 y z^2 - 6 a^3 b^4 c^2 x^3 y z^2 + 6 a^2 b^5 c^2 x^3 y z^2 + 5 a b^6 c^2 x^3 y z^2 - 3 b^7 c^2 x^3 y z^2 - 3 a^4 b^2 c^3 x^3 y z^2 + 4 a^3 b^3 c^3 x^3 y z^2 - 4 a b^5 c^3 x^3 y z^2 + 3 b^6 c^3 x^3 y z^2 - 3 a^3 b^2 c^4 x^3 y z^2 - a^2 b^3 c^4 x^3 y z^2 + a b^4 c^4 x^3 y z^2 + 3 b^5 c^4 x^3 y z^2 + 3 a^2 b^2 c^5 x^3 y z^2 - 3 b^4 c^5 x^3 y z^2 - 2 a^7 b c x y^3 z^2 + 4 a^6 b^2 c x y^3 z^2 - 4 a^4 b^4 c x y^3 z^2 + 2 a^3 b^5 c x y^3 z^2 - 3 a^7 c^2 x y^3 z^2 + 5 a^6 b c^2 x y^3 z^2 + 6 a^5 b^2 c^2 x y^3 z^2 - 6 a^4 b^3 c^2 x y^3 z^2 - 5 a^3 b^4 c^2 x y^3 z^2 + 3 a^2 b^5 c^2 x y^3 z^2 + 3 a^6 c^3 x y^3 z^2 - 4 a^5 b c^3 x y^3 z^2 + 4 a^3 b^3 c^3 x y^3 z^2 - 3 a^2 b^4 c^3 x y^3 z^2 + 3 a^5 c^4 x y^3 z^2 + a^4 b c^4 x y^3 z^2 - a^3 b^2 c^4 x y^3 z^2 - 3 a^2 b^3 c^4 x y^3 z^2 - 3 a^4 c^5 x y^3 z^2 + 3 a^2 b^2 c^5 x y^3 z^2 + a^8 c y^4 z^2 - 2 a^7 b c y^4 z^2 + 2 a^5 b^3 c y^4 z^2 - a^4 b^4 c y^4 z^2 - a^7 c^2 y^4 z^2 + a^4 b^3 c^2 y^4 z^2 - a^6 c^3 y^4 z^2 + a^4 b^2 c^3 y^4 z^2 + a^5 c^4 y^4 z^2 - a^4 b c^4 y^4 z^2 + a^5 b^4 x^3 z^3 + a^4 b^5 x^3 z^3 - 3 a^3 b^6 x^3 z^3 - a^2 b^7 x^3 z^3 + 2 a b^8 x^3 z^3 - a^4 b^4 c x^3 z^3 + 2 a^3 b^5 c x^3 z^3 + a^2 b^6 c x^3 z^3 - 4 a b^7 c x^3 z^3 + 2 b^8 c x^3 z^3 + a b^6 c^2 x^3 z^3 - b^7 c^2 x^3 z^3 + 2 a b^5 c^3 x^3 z^3 - 3 b^6 c^3 x^3 z^3 - a b^4 c^4 x^3 z^3 + b^5 c^4 x^3 z^3 + b^4 c^5 x^3 z^3 - 3 a^5 b^4 x^2 y z^3 + 3 a^4 b^5 x^2 y z^3 + 3 a^3 b^6 x^2 y z^3 - 3 a^2 b^7 x^2 y z^3 + a^4 b^4 c x^2 y z^3 - 4 a^3 b^5 c x^2 y z^3 + 5 a^2 b^6 c x^2 y z^3 - 2 a b^7 c x^2 y z^3 + 3 a^5 b^2 c^2 x^2 y z^3 - a^4 b^3 c^2 x^2 y z^3 + 6 a^2 b^5 c^2 x^2 y z^3 + 4 a b^6 c^2 x^2 y z^3 - 3 a^4 b^2 c^3 x^2 y z^3 + 4 a^3 b^3 c^3 x^2 y z^3 - 6 a^2 b^4 c^3 x^2 y z^3 - 3 a^3 b^2 c^4 x^2 y z^3 - 5 a^2 b^3 c^4 x^2 y z^3 - 4 a b^4 c^4 x^2 y z^3 + 3 a^2 b^2 c^5 x^2 y z^3 + 2 a b^3 c^5 x^2 y z^3 - 3 a^7 b^2 x y^2 z^3 + 3 a^6 b^3 x y^2 z^3 + 3 a^5 b^4 x y^2 z^3 - 3 a^4 b^5 x y^2 z^3 - 2 a^7 b c x y^2 z^3 + 5 a^6 b^2 c x y^2 z^3 - 4 a^5 b^3 c x y^2 z^3 + a^4 b^4 c x y^2 z^3 + 4 a^6 b c^2 x y^2 z^3 + 6 a^5 b^2 c^2 x y^2 z^3 - a^3 b^4 c^2 x y^2 z^3 + 3 a^2 b^5 c^2 x y^2 z^3 - 6 a^4 b^2 c^3 x y^2 z^3 + 4 a^3 b^3 c^3 x y^2 z^3 - 3 a^2 b^4 c^3 x y^2 z^3 - 4 a^4 b c^4 x y^2 z^3 - 5 a^3 b^2 c^4 x y^2 z^3 - 3 a^2 b^3 c^4 x y^2 z^3 + 2 a^3 b c^5 x y^2 z^3 + 3 a^2 b^2 c^5 x y^2 z^3 + 2 a^8 b y^3 z^3 - a^7 b^2 y^3 z^3 - 3 a^6 b^3 y^3 z^3 + a^5 b^4 y^3 z^3 + a^4 b^5 y^3 z^3 + 2 a^8 c y^3 z^3 - 4 a^7 b c y^3 z^3 + a^6 b^2 c y^3 z^3 + 2 a^5 b^3 c y^3 z^3 - a^4 b^4 c y^3 z^3 - a^7 c^2 y^3 z^3 + a^6 b c^2 y^3 z^3 - 3 a^6 c^3 y^3 z^3 + 2 a^5 b c^3 y^3 z^3 + a^5 c^4 y^3 z^3 - a^4 b c^4 y^3 z^3 + a^4 c^5 y^3 z^3 + a^4 b^5 x^2 z^4 - a^3 b^6 x^2 z^4 - a^2 b^7 x^2 z^4 + a b^8 x^2 z^4 - a^4 b^4 c x^2 z^4 - 2 a b^7 c x^2 z^4 + a^3 b^4 c^2 x^2 z^4 + a^2 b^4 c^3 x^2 z^4 + 2 a b^5 c^3 x^2 z^4 - a b^4 c^4 x^2 z^4 - a^7 b^2 x y z^4 + a^6 b^3 x y z^4 + a^3 b^6 x y z^4 - a^2 b^7 x y z^4 - a^6 b^2 c x y z^4 - 2 a^5 b^3 c x y z^4 - 2 a^3 b^5 c x y z^4 - a^2 b^6 c x y z^4 + 3 a^5 b^2 c^2 x y z^4 - a^4 b^3 c^2 x y z^4 - a^3 b^4 c^2 x y z^4 + 3 a^2 b^5 c^2 x y z^4 + a^4 b^2 c^3 x y z^4 + 4 a^3 b^3 c^3 x y z^4 + a^2 b^4 c^3 x y z^4 - 2 a^3 b^2 c^4 x y z^4 - 2 a^2 b^3 c^4 x y z^4 + a^8 b y^2 z^4 - a^7 b^2 y^2 z^4 - a^6 b^3 y^2 z^4 + a^5 b^4 y^2 z^4 - 2 a^7 b c y^2 z^4 - a^4 b^4 c y^2 z^4 + a^4 b^3 c^2 y^2 z^4 + 2 a^5 b c^3 y^2 z^4 + a^4 b^2 c^3 y^2 z^4 - a^4 b c^4 y^2 z^4=0
    ( Mostrar/Ocultar figura )


  • martes, 10 de septiembre del 2013

    Centros ortológicos sobre una cónica

    ( Anopolis #991 Antreas P. Hatzipolakis)

    Sean ABC un triángulo, P un punto, A'B'C' el triángulo tangencial, P un punto y L una recta por P.
    Consideremos las proyecciones ortogonales A*, B*, C* de los vértices A, B, C sobre la recta L, respectivamente.
    Los puntos medios Ma, Mb, Mc de AA*, BB*, CC*, respectivamente.
    Los puntos medios M1, M2, M3 de A'Ma, B'Mb, C'Mc, respectivamente.

    Entonces, los triángulos ABC y M1M2M3 son ortológicos.
    Si (u:v:w) son las coordenadas baricéntricas de P, las perpendiculares por los vértices M1, M2 y M3 a los lados BC, AC y AB, respectivamente, concurren sobre una cónica. cuyo centro tiene coordenadas baricéntricas:

    Q = ( a^4(2u+v+w) - 2a^2(b^2+c^2)(3u+2(v+w)) + (b^2-c^2)^2(4u+3(v+w)) : ... : ... ).

    ( Mostrar/Ocultar figura )
    Algunos casos particulares para centros del triángulo:

    P=X1, el centro de la cónica es el punto de coordenadas:

    (2a^4 - a^3(b+c) - a^2(5b^2-2bc+5c^2) + a(b-c)^2(b+c) + 3(b^2-c^2)^2 :...:...)

    que tiene (6-9-13)-número de búsqueda en ETC: 1.473737731682850782032396958

    P=X2, Q=X547, punto medio de X2 y X5.

    P=X3, Q=X3628, baricentro del conjunto de los vértices del triángulo medial y X5.
    En este caso, la cónica lugar de los centros ortológicos es la circunferencia de ecuación:

    a^2yz+b^2zx+c^2xy + (3/64)(x+y+z)((3a^2-5b^2-5c^2)x + (3b^2-5c^2-5a^2)y + (3c^2-5a^2-5b^2)z) = 0.



    P=X4, Q=X5.

    P=X5, en centro de la cónica es el punto de coordenadas:

    (2a^4 - 9a^2(b^2+c^2) + 7(b^2-c^2)^2:...:..)

    que tiene (6-9-13)-número de búsqueda en ETC: 1.3315581357050847119617684585

    P=X6, en centro de la cónica es el punto de coordenadas:

    (2a^6 - 5a^4(b^2+c^2) - 16a^2b^2c^2 + 3(b^2-c^2)^2(b^2+c^2) : ..:...)

    que tiene (6-9-13)-número de búsqueda en ETC: 1.3865627298839303658207405225

    P=X20, Q=X140, punto medio de X3 y X5.


    P=X40, Q=X3634, baricentro del conjunto de los vértices del triángulo medial y X10.

  • lunes, 9 de septiembre del 2013

    Par bicéntrico sobre la recta X(140)X(1499)

    Cresta de gallo

    ( Anopolis #980 Antreas P. Hatzipolakis)

    Sean ABC un triángulo, O el circuncentro, DEF el triángulo medial y A', B', C' los puntos medios de AO, BO, CO, respectivamente.

    Los centros radicales:
    P, de las circunferencias de diámetros A'E, B'F, C'D, y U, de las circunferencias de diámetros A'F, B'D, C'E, forman un par bicéntrico (que podría recibir el nombre de una flor: CRESTA DE GALLO).

    Sus coordenadas baricéntricas son:

    P = (3a^4-a^2(2b^2+7c^2)+b^4-3b^2c^2+2c^4 : ... : ... )

    U = (3a^4-a^2(7b^2+2c^2)+2b^4-3b^2c^2+c^4 : ... : ... ).

    ( Mostrar/Ocultar figura )
    El punto medio de PU es X140 (punto medio de ON). El punto del infinito de PU es X1499 (punto Biham).

  • domingo, 8 de septiembre del 2013

    Cúbica de Brocard (segunda), hipérbola de Jerabek y tripolar de X(2966)

    ( AdvancedPlaneGeometry #573 Dominik Burek)

    Sean ABC un triángulo y P un punto. La circunferencia circunscrita al triángulo BCP interseca a las rectas AB y AC en los puntos Ca y Ba. Similarmente, tenemos los puntos Ab, Cb, Bc y Ac. El lugar geométrico de los puntos P tales que Ca, Ba, Ab, Cb, Bc y Ac estén en una cónica es la (segunda) cúbica de Brocard ( K018 del catálogo de Bernard Gibert):

    Locus property:
    6. Locus of point M such that the three circles MBC, MCA, MAB meet the sidelines of triangle ABC again at six points lying on a same conic.

    ( Mostrar/Ocultar figura )

    Cuando P está en K018 las rectas BcCb, CaAc y AbBa delimitan un triángulo A'B'C' perspectivo con ABC, con centro de perspectividad Q en la hipérbola de Jerabek.
    El centro de perspectividad correspondiente al conjugado isogonal P* de P es el punto Q', diametralmente opuesto a Q en la hipérbola de Jerabek.

    En particular (para los puntos X2, X6, X13, X14, X15, X16, X111, X368, X534 sobre K018):
    Si P=X2 (P*=X6), Q=X67 y Q'=X6.
    Si P=X13 (P*=X15), Q=X4 y Q'=X74.
    Si P=X14 (P*=X16), Q=X4 y Q'=X74.
    Si P=X111 (P*=X524), Q=X6 y Q'=X76.
    Del punto X368 no se disponen coordenadas.

    La cúbica K018 y la hipérbola de Jerabek tienen seis puntos comunes: A, B, C, X6 y otros dos reales o imaginarios. La recta determinada por estos dos últimos es la tripolar de X2996.

    X2996 =(1/((b^2 - c^2) (b^4 + c^4 - a^2 (b^2 + c^2))) : ... : .... ).
    Barycentric product X(98)*X(99).
    Let P(2) and U(2) be the 1st and 2nd Beltrami points (as indexed at Bicentric Pairs), and let P(40) and U(40) be the isogonal conjugates of P(2) and U(2), respectively. Then X(2966) is the point of intersection of the lines P(2)U(40) and P(40)U(2). (Peter Moses, July 1, 2009).
  • sábado, 07 de septiembre del 2013

    Caracterización de la cúbica de Thomson

    (Anopolis #959 Antreas P. Hatzipolakis)

    Sean ABC un triángulo, O su circuncentro, P un punto y Oa, Ob, Oc los circuncentros de los triángulos PBC, PCA, PAB, respectivamente.

    La cúbica de Thomson es el lugar geométrico de los puntos P tales que el baricentro de OaObOc está sobre la recta OP.

    ( Mostrar/Ocultar figura )

  • viernes, 06 de septiembre del 2013

    Pares bicéntricos con punto medio en la recta de Euler

    (Anopolis #954 Antreas P. Hatzipolakis)

    Sea ABC un triángulo y A'B'C' el triángulo homotético a ABC por la homotecia de centro en el baricentro y razón t.
    Se denota por:
    Ab = B'C' ∩ AC, Ac = B'C' ∩ AB, Bc = C'A' ∩ BA, Ba = C'A' ∩ BC, Ca = A'B' ∩ CB, Cb = A'B' ∩ CA.

    Rt el centro radical de las circunferencias A(AAb), B(BBc), C(CCa):
    ( c^2 (b^2 - c^2) (2 + t)^2 + a^4 (-5 + 4 t + t^2) + a^2 (-3 b^2 (1 + 4 t + t^2) + c^2 (17 + 8 t + 2 t^2)) : -a^4 (2 + t)^2 + b^2 (b^2 (-5 + 4 t + t^2) - 3 c^2 (1 + 4 t + t^2)) + a^2 (c^2 (2 + t)^2 + b^2 (17 + 8 t + 2 t^2)) : -b^4 (2 + t)^2 + c^4 (-5 + 4 t + t^2) + b^2 c^2 (17 + 8 t + 2 t^2) + a^2 (b^2 (2 + t)^2 - 3 c^2 (1 + 4 t + t^2)) )
    St el centro radical de las circunferencias A(AAc), B(BBa), C(CCb):
    ( -b^2 (b^2 - c^2) (2 + t)^2 + a^4 (-5 + 4 t + t^2) + a^2 (-3 c^2 (1 + 4 t + t^2) + b^2 (17 + 8 t + 2 t^2)) : -c^4 (2 + t)^2 + b^4 (-5 + 4 t + t^2) + b^2 c^2 (17 + 8 t + 2 t^2) + a^2 (c^2 (2 + t)^2 - 3 b^2 (1 + 4 t + t^2)) : -a^4 (2 + t)^2 + c^2 (c^2 (-5 + 4 t + t^2) - 3 b^2 (1 + 4 t + t^2)) + a^2 (b^2 (2 + t)^2 + c^2 (17 + 8 t + 2 t^2)) )

    Rt y St forman un par bicéntrico de punto medio sobre la recta de Euler, si t=m/n:
    Mt = ( (b^2-c^2)^2(m+2 n)^2 + a^2(b^2+c^2)(m^2+4mn-14n^2) - 2a^4(m^2+ 4mn-5n^2) : ... : .... ).

    ( Mostrar/Ocultar figura )


    Situaciones particulares del punto medio del par bicéntrico:
    t=1,-5 X(5) ABC=A'B'C'
    t=-1/2 X(3530) A'B'C'= triángulo medial
    t=-2 X(3) A'B'C'= triángulo anticomplementario
    t=-1 (16a^4 - 17a^2(b^2+c^2) + (b^2-c^2)^2: ... : ...) A'B'C'= triángulo reflexión de ABC en el baricentro
    t↦0 X(5054) A'B'C' degenera en el baricentro
    t= -2 ± Sqrt[6] X(2)
    t= -2 ± 3Sqrt[2] X(4)

    Para todo t, el punto del infinito de la recta RtSt, determinada por cada par bicéntrico es el punto de Biham, X1499.

  • jueves, 5 de septiembre del 2013

    Propiedades de los puntos X(960), X(2883), X(3743), como centros de cónicas

    ( AdvancedPlaneGeometry #458 Dominik Burek)

    Sean ABC un triángulo, P un punto y DEF el triángulo ceviano de P. Las rectas que pasan por E y F, paralelas a AD intersecan a BC en A1 y A2. Similarmente tenemos los puntos B1, B2, C1, C2. Entonces los seis puntos A1, A2, B1, B2, C1, C2 están en una cónica.

    Su ecuación baricéntrica, si P(u:v:w), es

    Σ v^2w^2(u+v+w)x^2 - uvw(2u^2+(v+w)(2u+v+w))yz=0.

    Esta cónica es homotética a la cónica circunscrita de centro en el complemento P' de P (PG=2GP'). ( Francisco Javier García Capitán )
    ( Mostrar/Ocultar figura )

    Cuando P=X4 esta cónica es la circunferencia de Taylor.

    Otros casos particulares:
    P X(1) X(2) X(4) X(8) X(20)
    Centro cónica X(3743) X(2) X(389) X(960) X(2883)

  • miércoles, 04 de septiembre del 2013

    El centro del triángulo X84 como centro ortológico

    (Anopolis #947 Antreas P. Hatzipolakis)

    Sea ABC un triángulo. Se denota por: Ab y Ac las proyecciones ortogonales de A sobre las bisectrices por B y C, respectivamente. Ha es el ortocentro del triángulo AAbAc. Similarmente, se consideran los ortocentros Hb y Hc.

    Entonces los triángulos ABC y HaHbHb son ortológicos y sus centros ortológicos son el ortocentro de ABC y el centro del triángulo X84.

    ( Mostrar/Ocultar figura )

  • martes, 03 de septiembre del 2013

    El centro del triángulo X3521 como centro ortológico

    (Anopolis #943 Antreas P. Hatzipolakis)

    Sean ABC un triángulo y O su circuncentro. Se denota por: Na, Nb, Nc los centros de las circunferencias de los nueve puntos de los triángulos OBC, OCA, OAB, respectivamente, y por Ma, Mb, Mc los puntos medios de ANa, BNb, CNc, respectivamente.
    Entonces, los triángulos ABC y MaMbMc son ortológicos. Los centro ortológicos son X5 y X3521.

    ( Mostrar/Ocultar figura )

  • miércoles, 28 de agosto del 2013

    Un centro del cuadrivértice

    (Quadri-Figures-Group #206 Chris van Tienhoven)


    Sean ABC un triángulo, P4 un punto y P1P2P3 su triángulo anticeviano; entonces, P1P2P3P4 es un cuadrivértice con puntos diagonales A, B y C (ABC es el triángulo diagonal del cuadrivértice).
    Un sistema de QA-DT-coordenadas relativas al cuadrivértice P1P2P3P4, se define tomando como triángulo de referencia el triángulo diagonal y uno de los vértices del cuadrivértices, por ejemplo P4, con coordenadas (p:q:r); los otros tres vértices forman el triángulo anticeviano de P4: P1(-p:q:r), P2(p:-q:r), P3(p:q:-r).

    Denotamos por Ci el centro de la cónica que pasa por los cinco puntos A, B, C, Pi y QA-P10 (i=1,2,3,4), donde QA-P10 es el baricentro del triángulo diagonal ABC. Se verifica que las rectas PiCi son concurrentes en el punto de DT-coordenadas:

    ( p^2(q^2+r^2) : q^2(r^2+p^2) : r^2(p^2+q^2 ).

    ( Mostrar/Ocultar figura )

    GENERALIZACIÓN:
    Sean X un punto y Ci el centro de la cónica que pasa por los cinco puntos A, B, C, Pi y X (i=1,2,3,4).
    Las rectas PiCi son concurrentes si solo si X queda en la cónica de los nueve puntos (QA-Co1: Nine-point Conic) circunscrita al triángulo diagonal y centro en el baricentro del cuadrivértice (QA-P1), junto con una cuártica que pasa P1, P2, P3, P4 A, B, C, QA-P10 de ecuación ((Eckart Schmidt Quadri-Figures-Group #209 ):

    p^2(q^2-r^2)y^2z^2 + q^2(r^2-p^2)z^2x^2 + r^2(p^2-q^2)x^2y^2 = 0.

    Esta cuártica es conjugada isogonal de la cónica de ecuación:

    p^2x^2(q^2-r^2)+q^2y^2(r^2-p^2)+r^2z^2(p^2-q^2)=0.

    Esta cónica contiene a las vértices del triángulo anticeviano de sus puntos. Como pasa por los centros QA-P10 y QA-P19 también pasa por los vértices de sus triángulos anticevianos, GaGbGc (triángulo antimedial del triángulo diagonal) y LMN, respectivamente. Así, cónica y cuártica pueden construirse.

    La cónica y la cuártica tienen tangentes comunes en los vértices del triángulo antimedial, las cuales determinan un triángulo perspectivo con el triángulo diagonal; su centro de proyectividad tiene DT-coordenadas:

    ( 1/(p^2(q^2-r^2)) : 1/(q^2(r^2-p^2)) : 1/(r^2(p^2-q^2)) ).



    • Si X recorre la cuártica el punto Y de intersección de las rectas PiCi describe la cúbica QA-Cu6 (QA-P1-InvolutionCenter Cubic), de ecuación:

    q^2 r^2 (q^2 - r^2) x^3 + q^2 r^2 (2 p^2 (y - z) - (q^2 - r^2) (y + z)) x^2 + p^2 r^2 (r^2 - p^2) y^3 + p^2 r^2 (2 q^2 (z - x) - (r^2 - p^2) (x + z)) y^2 + p^2 q^2 (p^2 - q^2) z^3 + p^2 q^2 (2 r^2 (x - y) - (p^2 - q^2) (x + y)) z^2=0.

    ( Mostrar/Ocultar figura )

    • Si X recorre la cónica de los nueve puntos (QA-Co1: Nine-point Conic) el punto Y de intersección de las rectas PiCi describe la misma cúbica que cuando X recorre la cuártica.
    ( Mostrar/Ocultar figura )



  • sábado, 24 de agosto del 2013

    Otro par bicéntrico

    Morgallana

    (Anopolis #871 Antreas P. Hatzipolakis)

    Sean ABC un triángulo y A'B'C' el triángulo ceviano del incentro.
    Sea Ab la reflexión de A' en BB', y se definen Bc y Ca cíclicamente. Sea Ac la reflexión de A' en BC', y se definen Ba y Cb cíclicamente.
    Se denota por:
    (Oab), (Oac) las circunferencias circunscritas a los triángulos AAbA', AAcA', resp.
    (Obc), (Oba) las circunferencias circunscritas a los triángulos BAcB', BBaB', resp.
    (Oca), (Ocb) las circunferencias circunscritas a los triángulos CCaC', CCbC', resp.

    Las coordenadas baricéntricas del centro radical R de las circunferencias (Obc), (Oca), (Oab) son:
    R = (a (a^2(b-2c) - a(b^2+b c-c^2) - (b-c)c^2 ) : ... : ... ).

    Las coordenadas baricéntricas del centro radical S de las circunferencias (Oba), (Ocb), (Oac) son:
    S = (a (a^2(2b-c) - a(b^2-b c-c^2) - b^2(b-c)) : ... : ... ).

    Los puntos R y S forman un par bicéntrico P(110) (que podría recibir el nombre de una flor: MORGALLANA o botón de oro), por tanto el punto del infinito de la recta RS es un centro: X1938, que coincide con el punto del infinito del par bicéntrico PU(15).

    La suma de las coordenadas baricéntricas de este par baricéntrico es el centro X942, inverso en la circunferencia inscrita del inverso en la circunferencia circunscrita del incentro.

    La diferencia de las coordenadas baricéntricas de este par baricéntrico es el centro de coordenadas baricéntricas:

    ( a(b-c)(3a^2-2a b-b^2-2a c-c^2): ... : ...),

    con número de búsqueda en ETC: -8.16982446237566024925964383. El punto medio de RS es un centro, de coordenadas baricéntricas:

    M = (a (3a^4(b-c)^2 + a^3(-5b^3+4 b^2c+4b c^2-5c^3) + a^2(b^4+5b^3c-14b^2c^2+5b c^3+c^4) + a(b-c)^2(b^3+c^3) - b c(b-c)^2(b^2+c^2)) : ... : ...),

    con número de búsqueda en ETC: 1.2448168635877587605351158452.
    ( Mostrar/Ocultar figura )

  • viernes, 23 de agosto del 2013

    Baricentro de un triángulo asociado al triángulo medial

    (Anopolis #872 Antreas P. Hatzipolakis)

    Sean ABC un triángulo y A'B'C' su triángulo medial. Se denota por:
    Obc y Ocb los circuncentros de los triángulos BCB' y BCC', respectivamente, y por Ma el punto medio de ObcOcb. Similarmente se definen Mb y Mc.
    El baricentro G' de MaMbMc queda en la recta de Euler de ABC y sus coordenadas baricéntricas son:

    G' = (2 SB SC + 11 a^2 SA : 2 SC SA + 11 b^2 SB : 2 SA SB + 11 c^2 SC),

    es el complemento de X3845 y divide al segmento GO en la razón GG':G'O = 3:1.
    ( Mostrar/Ocultar figura )

  • miércoles, 21 de agosto del 2013

    Un par bicéntrico

    Canarina

    (Anopolis #860 Antreas P. Hatzipolakis)

    Sean ABC un triángulo y A'B'C' el triángulo ceviano del incentro. Se denota por:
    (Nab), (Nac) las circunferencias de los nueve puntos de los triángulos AIB', AIC', resp.
    (Nbc), (Nba) las circunferencias de los nueve puntos de los triángulos BIC', BIA', resp.
    (Nca), (Ncb) las circunferencias de los nueve puntos de los triángulos CIA', CIB', resp.

    Las coordenadas baricéntricas del centro radical R de las circunferencias (Nbc), (Nca), (Nab) son:
    R = ( a(a + b - c) (a - b + c) (a^3 b - a b^3 + 2 a^3 c - 2 a b^2 c - b^3 c + a^2 c^2 - 3 a b c^2 - 3 b^2 c^2 - 2 a c^3 - 3 b c^3 - c^4) : ... : ... ).

    Las coordenadas baricéntricas del centro radical S de las circunferencias (Nba), (Ncb), (Nac) son:
    S = ( a (a + b - c) (a - b + c) (2 a^3 b + a^2 b^2 - 2 a b^3 - b^4 + a^3 c - 3 a b^2 c - 3 b^3 c - 2 a b c^2 - 3 b^2 c^2 - a c^3 - b c^3) : ... : ... ).

    Los puntos R y S forman un par bicéntrico P(111) (que podría recibir el nombre de una flor: CANARINA o bicácaro), por tanto el punto del infinito de la recta RS es un centro: X513.
    También el punto medio de RS es un centro, de coordenadas baricéntricas:

    M = (a(3a^5(b+c)+ a^4(b^2+c^2) - 2a^3(3b^3+b^2c+b c^2+3c^3) - 2a^2(b^4+ b^3c+4b^2c^2+b c^3+c^4) + a(b-c)^2(3b^3+5b^2c+5b c^2+3c^3)+ (b-c)^2(b+c)^4) : ... : ...),

    con número de búsqueda en ETC: 2.235165076122804495064988816.
    ( Mostrar/Ocultar figura )
    Los ejes radicales de los tres pares de circunferencias (Nab), (Nac); (Nbc), (Nba) y (Nca), (Ncb) concurren en el centro X3649
  • martes, 20 de agosto del 2013

    La cúbica pivotal pK(X1073,X253)

    (Anopolis #859 Antreas P. Hatzipolakis)

    Sean ABC un triángulo, P un punto y A'B'C' el triángulo ceviano de P. Denotamos por Ab la reflexión, respecto a C, de la proyección ortogonal de A' sobre AC y por Ac la reflexión, respecto a B, de la proyección ortogonal de A' sobre AB . Sea Oa el circuncentro del triángulo AAbAc; similarmente, se consideran los circuncentros Ob y Oc.

    • Los triángulos ABC y OaObOc son perspectivos si y solo si P está en la cúbica pivotal pK(X1073, X253). Esta cúbica pasa por los centros X(2), X(3), X(64), X(69), X(253), X(1073), ...

    ( Mostrar/Ocultar figura )

    • Los triángulos A'B'C' y OaObOc son perspectivos si y solo si P está en la cubica pivotal pK(Ω, X), donde polo y pivote son, respectivamente::

    Ω = ( a^2SA/(a^4 + 2a^2(b^2+c^2) - 3(b^2-c^2)^2) : ... : ...),

    con número de búsqueda en ETC: 5.40902452312896952998608493.

    X = ( 1/(a^4 + 2a^2(b^2+c^2) - 3(b^2-c^2)^2) : ... : ...),

    con número de búsqueda en ETC: 6.399682552061115067504709200.
    ( Mostrar/Ocultar figura )
    Esta cúbica contiene a su pivote (como toda isocúbica pivotal), a su polo y a los centros X(2), X(3), X(69), ...

  • sábado, 17 de agosto del 2013

    Propiedad de las isocúbicas pK(X2,X264) y pK(X216,X4)

    (Anopolis #842 Antreas P. Hatzipolakis)

    a Clara, por su "cumple"

    Sean ABC un triángulo, P un punto y A'B'C' el triángulo ceviano de P. Denotamos por Ab la reflexión de C respecto a la perpendicular por A' a AC y por Ac la reflexión de B respecto a la perpendicular por A' a AB. Sea Oa el circuncentro del triángulo AAbAc; similarmente, se consideran los circuncentros Ob y Oc.
    El lugar geométrico de P tal que los triángulos A'B'C' y OaObOc son perspectivos es la isocúbica pK(X2,X264) (K045 del catálogo de Bernard Gibert).
    El lugar geométrico de los centros de perspectividad es la isocúbica pK(X216,X4) (K044 del catálogo de Bernard Gibert).
    ( Mostrar/Ocultar figura )

    • Los triángulos ABC y OaObOc son perspectivos si y solo si P está sobre la cúbica que pasa a través de los puntos X(2), X(3), X(54), X(69), X(95), X(96), X(97),... los pies de las cevianas de X(95); de ecuación baricéntrica:

    Σ[ x(c^2(a^2+b^2-c^2)(a^4-a^2 b^2-2a^2c^2-b^2c^2+ c^4) y^2 - b^2(a^2-b^2+ c^2)(a^4-2a^2b^2+b^4-a^2c^2-b^2c^2) z^2)] =0.


    Esta cúbica ha sido incluida en el Catálogo de Bernard Gibert con el número K646 = Montesdeoca cubic, pK(X97, X95)
    ( Mostrar/Ocultar figura )
    En este caso el centro de perspectividad de los triángulos ABC y OaObOc queda sobre la cúbica K044 = pK(X216,X4).

  • domingo, 11 de agosto del 2013

    Un centro ortológico relativo al triángulo medial

    Sean ABC un triángulo, A'B'C' el triángulo medial y A"B"C" el triángulo ceviano del incentro. Se denota por Ab y Ac las proyecciones ortogonales de A sobre BB" y CC". Similarmente, se definen los puntos Bc, Ba, Ca y Ca.
    Si Ma, Mb y Mc son los puntos medios de AbAc, BcBa y CaCb, entonces:
    • Los triángulos ABC y MaMbMc son ortológicos, con centros ortológicos X84 y X946.
    • Los triángulos A'B'C' y MaMbMc son ortológicos, con centros ortológicos X946 y el punto X de coordenadas baricéntricas:

    X = ( (a^3+a^2(b+c) - a(b+c)^2 - (b-c)^2(b+c)) (a^3(b+c) - a^2(b-c)^2 - a(b-c)^2(b+c) + (b^2-c^2)^2) : ... : ... ).

    que tiene (6-9-13)-número de búsqueda en ETC: 30.3292407999481595887008929
    ( Mostrar/Ocultar figura )


  • sábado, 10 de agosto del 2013

    Cónicas con centro y perspector coincidentes

    En una cónica (no autoadjunta) expresada en coordenadas baricéntricas respecto a un triángulo ABC por la ecuación:

    fx^2 + gy^2 + hz^2 + 2pyz + 2qzx + 2rxy = 0,

    su centro y perspector (centro de perspectividad de ABC y el triángulo formado por las polares de sus vértices respecto a la cónica) coinciden si y sólo si p=q=r≠0.
    (Paul Yiu).

    UN EJEMPLO:
    (Anopolis #774 Antreas P. Hatzipolakis)

    Sean ABC un triángulo, A'B'C' su triángulo antimedial y A"B"C" el triángulo pedal del incentro.
    Se denota por Ab = A"B" ∩ B'C' y por Ac = A"C" ∩ B'C'.
    Similarmente y de forma cíclica de definen Bc, Ba y Ca, Cb.

    Los seis puntos Ab, Ac, Bc, Ba, Ca y Cb están en una misma cónica de ecuación:

    (b+c-a)^2(a+b-3c)(a-3b+c)x^2 + (c+a-b)^2(b+c-3a)(b-3c+a)y^2 + (a+b-c)^2(c+a-3b)(c-3a+b)z^2 + 8S^2(yz+zx+xy) =0.

    Su centro y perspector coinciden en el centro de coordenadas:

    ( 1/((b+c-a)(a^3-a^2(b+c) - a(b^2-8b c+c^2) + (b+c)(b^2-4b c+c^2)) : ... : ...),

    que tiene (6-9-13)-número de búsqueda en ETC: -1.411602064280868133943591227.
    ( Mostrar/Ocultar figura )
    Los triángulos ABC y el triángulo DEF formado por las polares de sus vértices respecto a la cónica son homotéticos. En el caso general que la cónica tenga por ecuación fx^2 + gy^2 + hz^2 + 2k(yz + zx + xy) = 0, el centro de homotecia (perspector de la cónica) es:

    (1/(f-k) : 1/(g-k) : 1/(h-k)),

    y la razón de homotecia es:

    (k^2(f+g+h-2k)-fgh)/((f-k)(g-k)(h-k)).

    El centro de perspectividad del triángulo antimedial A'B'C' y el triángulo DEF formado por las polares de sus vértices respecto a la cónica son homotéticos, con centro de homotecia:

    (f g h - 2 g h k - f k^2 + g k^2 + h k^2, f g h - 2 f h k + f k^2 - g k^2 + h k^2, f g h - 2 f g k + f k^2 + g k^2 - h k^2).



  • viernes, 9 de agosto del 2013

    Construir un triángulo dado a, A, (b-c)/(b+c)

    (Anopolis #777 Antreas P. Hatzipolakis)

    Trazamos un segmento BC de longitud a dada.
    Sea un punto V sobre el arco capaz de ángulo A sobre el segmento BC.
    La circunferencia de centro V y radio VB corta a la recta CV en los punto S y D, tales que CS=VC+VB y SD=VC-CB.
    El punto Cv que divide al segmento DS en la relación DCv : CvS = m:n, describe una circunferencia que pasa por B y C.
    Deberemos localizar la ubicación del punto V tal que Cv coincida con C.
    El punto V solución es la intersección de la tangente en C a tal circunferencia con el arco capaz considerado.

    ( Mostrar/Ocultar figura )

    Más construcciones de triángulos en: http://amontes.webs.ull.es/pdf/trresolu.pdf

  • jueves, 8 de agosto del 2013

    Propiedad del centro X3663

    (Anopolis #786 Antreas P. Hatzipolakis)

    Sean ABC un triángulo, I el incentro y A'B'C' el triángulo medial y A"B"C" el triángulo ceviano del incentro.
    Se toman los puntos

    Ab=BB" ∩ B'C',   Ac=CC" ∩ B'C',   Bc=CC" ∩ C'A',   Ba=AA" ∩ C'A',   Ca=AA" ∩ A'B',   Cb=BB" ∩ A'B'.

    La tangente en Ab a la cónica que pasa por los puntos Ab, Bc, Ba, Ca, Cb, de ecuación baricéntrica:

    (-a^3-a(b+c)^2+c(b+c)^2+ a^2(2b+c))x+ (a-c)(a-b+c)^2y + (-a^3+3a^2c-(b-c)^2c+ a(b^2-4b c-c^2))z=0,

    y la tangente en Ac a la cónica que pasa por los puntos Ac, Bc, Ba, Ca, Cb, de ecuación baricéntrica:

    (a^3+a(b+c)^2-b(b+c)^2-a^2(b+2c))x + (a^3-3a^2b+b(b-c)^2+ a(b^2+4b c-c^2))y -(a-b)(a+b-c)^2z=0,

    se cortan en el centro X3663:

    ( (b-c)^2+a(b+c) : (c-a)^2+b(c+a) : (a-b)^2+c(a+b) ).

    ( Mostrar/Ocultar figura )

    Un proceso cíclico en la construcción anterior nos conduce al mismo centro del triángulo.

  • miércoles, 7 de agosto del 2013

    La cúbica de McCay como lugar geométrico

    ( AdvancedPlaneGeometry #458 Dominik Burek - Bernard Gibert)

    Sean ABC un triángulo, P un punto, O1, O2 y O3 los circuncentros de los triángulos BCP, CAP y ABP. Entonces el triángulo O1O2O3 y el triángulo pedal del conjugado isogonal P* de P, con respecto a ABC, son homotéticos con centro de homotecia, si P(u:v:w), de coordenadas baricéntricas:

    Q = ( a^2v w(a^2(u^2+2v w + u(v+w)) - u(b^2(u+v-w) + c^2(u-v+w)): ... : ... )


    Los puntos P, P*, QP, Q, O ) están alineados si y solo si P está sobre la cúbica de McCay (K003 = pK(X6,X3) del catálogo de Bernard Gibert):
    Σ a^2(b^2+c^2-a^2)x(c^2y^2-b^2z)=0.
    ( Mostrar/Ocultar figura )


  • martes, 6 de agosto del 2013

    Lugares geométricos y circunferencias asociadas al incentro

    (Anopolis #759 Antreas P. Hatzipolakis)

    Sean ABC un triángulo, I el incentro y Oa, Ob, Ob los circuncentros de los triángulos IBC, ICA, IAB, respectivamente.
    Sean P un punto y Aa, Ab, Ac las proyecciones ortogonales de Oa sobre AP, BP, CP, respectivamente, y O1 el circuncentro del triángulo AaAbAc. Similarmente, se definen O2 y O3.

    El circuncentro OP del triángulo O1O2O3 es el punto medio de P y el circuncentro O de ABC.

    ( Mostrar/Ocultar figura )

    Los ejes radicales de la circunferencia circunscrita a O1O2O3 y las circunscritas a AaAbAc, BaBbBc, CaCbCc delimitan un triángulo A'B'C' homotético a ABC.
    Si P=(u:v:w) el centro de homotecia Q tiene coordenadas baricéntricas:

    Q = ( a(2b c u^2 - a^2v w- b(b-2c)u w + c(2b-c)u v):
    b(2c a v^2 - b^2w u- c(c-2a)v u + a(2c-a)v w):
    c(2a b w^2 - c^2u v- a(a-2b)w v + b(2a-b)w u) ).

    • El punto Q coincide con P, cuando éste está en la circunferencia circunscrita o coincide con el incentro.

    • Cuando el punto P varía sobre una recta, el punto Q describe un cónica que pasa por el incentro y por los puntos de intersección de la recta con la circunferencia circunscrita, si existen.

    Si P recorre una recta tangente a la circunferencia circunscrita en un punto D, la cónica descrita por el punto Q es tangente a dicha recta en Q y a la paralela por el incentro en éste (por lo que el centro es el punto medio de ID.
    ( Mostrar/Ocultar figura )


  • lunes, 5 de agosto del 2013

    Las cúbicas K007 y K279 como lugares geométricos

    (Anopolis #758 Antreas P. Hatzipolakis)

    Sean ABC un triángulo, A'B'C' el triángulo ceviano de un punto P y Oa, Ob, Oc los centros de las circunferencias circunscritas a los triángulos AB'C', BC'A', CA'B', respectivamente, los cuales se cortan en un punto M. Se denota por O* el circuncentro del triángulo OaObOc.

    • El lugar geométrico de los puntos P tales que O* está en la recta de Euler es la cúbica (K279 = pK(X2,X3260) del catálogo de Bernard Gibert):
    Σ b^2c^2 (2a^4-a^2(b^2+c^2)-(b^2-c^2)^2)x(y^2-z^2)=0.

    ( Mostrar/Ocultar figura )
    • El lugar geométrico de los puntos P tales que M, O*, O están alineados es la cúbica de Lucas (K007 = pK(X2,X69) del catálogo de Bernard Gibert):
    Σ (b^2+c^2-a^2)x(y^2-z^2)=0.
    ( Mostrar/Ocultar figura )
    Cuando P recorre la cúbica de Lucas, el punto M recorre la cúbica de Darboux.

  • domingo, 4 de agosto del 2013

    Cónica conteniendo ortocentros

    ( AdvancedPlaneGeometry #436 Barry Wolk)

    Sean ABC un triángulo, D, E, F puntos sobre los lados BC, CA, AB, respectivamente. Se denota por Ha, Hb, Hc los ortocentros de los triángulos AEF, BFD, CDE, respectivamente, entonces los puntos D, E, F, Ha, Hb, Hc quedan sobre una cónica.

    ( Mostrar/Ocultar figura )
    Considérese el hexágono DHcEHaFHbD. Las rectas DHc y FHa son ambas perpendiculares a AC, y cíclicamente; así, los lados opuestos de este hexágono son paralelos (se cortan dos a dos en la recta del infinito). Por el recíproco del teorema de Pascal, estos seis puntos están sobre una cónica.

  • sábado, 3 de agosto del 2013

    Centros de circunferencias de nueve puntos sobre rectas por el circuncentro

    (Anopolis #728 Antreas P. Hatzipolakis)

    Sean ABC un triángulo, A'B'C' el triángulo ceviano de un punto P y A", B", C" los puntos simétricos de P respecto a A', B', C', respectivamente.

    El centro N" de la circunferencia de los nueve puntos de A"B"C" está en la recta OP si P queda en una séptica

      c^4 x^4 y^3 - c^4 x^3 y^4 + a^2 c^2 x^4 y^2 z + 2 b^2 c^2 x^4 y^2 z - c^4 x^4 y^2 z + 2 a^2 c^2 x^3 y^3 z - 2 b^2 c^2 x^3 y^3 z - 2 a^2 c^2 x^2 y^4 z - b^2 c^2 x^2 y^4 z + c^4 x^2 y^4 z - a^2 b^2 x^4 y z^2 + b^4 x^4 y z^2 - 2 b^2 c^2 x^4 y z^2 + a^2 b^2 x^3 y^2 z^2 - b^4 x^3 y^2 z^2 - a^2 c^2 x^3 y^2 z^2 + c^4 x^3 y^2 z^2 + a^4 x^2 y^3 z^2 - a^2 b^2 x^2 y^3 z^2 + b^2 c^2 x^2 y^3 z^2 - c^4 x^2 y^3 z^2 - a^4 x y^4 z^2 + a^2 b^2 x y^4 z^2 + 2 a^2 c^2 x y^4 z^2 - b^4 x^4 z^3 - 2 a^2 b^2 x^3 y z^3 + 2 b^2 c^2 x^3 y z^3 - a^4 x^2 y^2 z^3 + b^4 x^2 y^2 z^3 + a^2 c^2 x^2 y^2 z^3 - b^2 c^2 x^2 y^2 z^3 + 2 a^2 b^2 x y^3 z^3 - 2 a^2 c^2 x y^3 z^3 + a^4 y^4 z^3 + b^4 x^3 z^4 + 2 a^2 b^2 x^2 y z^4 - b^4 x^2 y z^4 + b^2 c^2 x^2 y z^4 + a^4 x y^2 z^4 - 2 a^2 b^2 x y^2 z^4 - a^2 c^2 x y^2 z^4 - a^4 y^3 z^4=0
    circunscrita a los triángulos ABC, medial y excentral, y que pasa por el incentro, baricentro y circuncentro.
    ( Mostrar/Ocultar figura )
    Las tangentes en los vértices del triángulo medial son las mediatrices de ABC.

    • Más en general, si en vez de tomar las reflexiones de P en los vértices de su triángulo ceviano, se consideran los puntos At, Bt, Ct sobre las rectas AA', BB', CC', respectivamente, tales que AtA' / AtP = BAtB'/BtP = CtC'/CtP = t.
    El centro Nt de la circunferencia de los nueve puntos de AtBtCt está en la recta OP, cuando t varía, si P queda en la misma séptica anterior.


  • viernes, 2 de agosto del 2013

    Propiedad del centro del triángulo X3579

    (Anopolis #741 Antreas P. Hatzipolakis)

    Sean ABC un triángulo, entonces el centro de la circunferencia de los nueve puntos del triángulo medial DEF del triángulo excentral IaIbIc es el centro X3579.

    ( Mostrar/Ocultar figura )


  • jueves, 1 de agosto del 2013

    Cuárticas asociadas a triángulos circuncevianos

    Sean ABC un triángulo, P un punto, DEF el triángulo circunceviano de P y D1, E1, F1 los puntos de intersección de BC, CA, AB con EF, FD, DE, respectivamente. Los puntos D1, E1, F1 están en una recta que denotamos por dP y, si P=(u:v:w), el tripolo de dP (respeto a ABC) es el punto:

    U = ( 1/(c^2v+b^2w) : 1/(a^2w+c^2u) : 1/(b^2u+a^2v) ),

    que es el "cevapoint" de P y el simediano.
    ( Mostrar/Ocultar figura )

    Las rectas AD1, BE1, CF1 vuelven a cortar a la circunferencia circunscrita en A', B', C', respectivamente. Los triángulos DEF y A'B'C' son perspectivos con centro de perspectividad:

    V = ( (c^2uv+b^2uw-a^2vw)/(c^2v+b^2w) : (a^2wu+c^2vu-b^2wu)/(a^2w+c^2u) : (b^2wu+a^2wv-c^2uv)/(b^2u+a^2v) ).

    que es el producto baricéntrico de U y el anticomplemento del conjugado isogonal P* de P (ADGEOM #422).

    Los puntos P, U, V están alineados.

    El triángulo A"B"C" determinado por las rectas AA', BB', CC' es perspectivo con ABC y con A'B'C', con centros de perspectividad U y V, respectivamente.

    • El punto U está en la recta del infinito (la recta dP es tangente a la elipse inscrita de Steiner) si y solo si P está sobre la elipse circunscrita de Steiner del triángulo tangencial TaTbTc.
    ( Mostrar/Ocultar figura )


    • El punto V está en la recta del infinito si y solo si P está sobre una cuártica
      b^2*c^4*x^3*y + a^2*c^4*x^2*y^2 + b^2*c^4*x^2*y^2 - c^6*x^2*y^2 + a^2*c^4*x*y^3 + b^4*c^2*x^3*z + a^2*b^2*c^2*x^2*y*z + a^2*b^2*c^2*x*y^2* z + a^4*c^2*y^3*z + a^2*b^4*x^2*z^2 - b^6*x^2*z^2 + b^4*c^2*x^2*z^2 + a^2*b^2*c^2*x*y*z^2 - a^6*y^2*z^2 + a^4*b^2*y^2*z^2 + a^4*c^2*y^2*z^2 + a^2*b^4*x*z^3 + a^4*b^2*y*z^3=0
    tangente en A, B,C a los lados del triángulo tangencial, y que pasa por los vértices de éste.
    ( Mostrar/Ocultar figura )

    • Las rectas paralelas a BC, CA, AB que pasan respectivamente por los vértices Ta, Tb, Tc del triángulo tangencial, determinan un triángulo T'aT'bT'c, que es perspectivo con el triángulo DEF circunceviano de P si y sólo si P está sobre una cuártica
      a^2*b^4*c^4*x^3*y - b^6*c^4*x^3*y + a^2*b^2*c^6*x^3*y + b^2*c^8*x^3*y + 2*a^4*b^2*c^4*x^2*y^2 - 2*a^2*b^4*c^4*x^2*y^2 + a^6*c^4*x*y^3 - a^4*b^2*c^4*x*y^3 - a^2*b^2*c^6*x*y^3 - a^2*c^8*x*y^3 - a^2*b^6*c^2*x^3*z - b^8*c^2*x^3*z - a^2*b^4*c^4*x^3*z + b^4*c^6*x^3*z - a^4*b^4*c^2*x^2*y*z - a^2*b^6*c^2*x^2*y*z + a^4*b^2*c^4*x^2*y* z + a^2*b^2*c^6*x^2*y*z + a^6*b^2*c^2*x*y^2*z + a^4*b^4*c^2*x*y^2* z - a^2*b^4*c^4*x*y^2*z - a^2*b^2*c^6*x*y^2*z + a^8*c^2*y^3*z + a^6*b^2*c^2*y^3*z + a^4*b^2*c^4*y^3*z - a^4*c^6*y^3*z - 2*a^4*b^4*c^2*x^2*z^2 + 2*a^2*b^4*c^4*x^2*z^2 - a^6*b^2*c^2*x*y*z^2 + a^2*b^6*c^2*x*y* z^2 - a^4*b^2*c^4*x*y*z^2 + a^2*b^4*c^4*x*y*z^2 + 2*a^4*b^4*c^2*y^2* z^2 - 2*a^4*b^2*c^4*y^2*z^2 - a^6*b^4*x*z^3 + a^2*b^8*x*z^3 + a^4*b^4*c^2*x*z^3 + a^2*b^6*c^2*x*z^3 - a^8*b^2*y*z^3 + a^4*b^6*y*z^3 - a^6*b^2*c^2*y*z^3 - a^4*b^4*c^2*y*z^3=0
    circunscrita a ABC y a >T'aT'bT'c, que pasa por los centros X1, X3, X25, X56, X251, X5004, X5005.

    Para los puntos P = X1, X3, X25, X56, X251 los centros de perspectividad Q son, respectivamente:

    -- X3,
    -- (a^2 (a^8 - 8a^4b^2c^2 - 2a^6(b^2+c^2) + 2a^2(b^2+c^2)^3- (b^2-c^2)^4):...:...), está en la recta de Euler.
    -- X25,
    -- ( a^2 (a+b-c)(a-b+c)(a^6 - a^4(b-c)^2 + 4a^3b c(b+c) - a^2(b^4-4b^3c+22 b^2c^2-4b c^3+c^4) + 4a b c(b+c)^3 + (b-c)^2(b+c)^4 ) : ... : ...),
    -- X1180.

    ( Mostrar/Ocultar figura )


  • miércoles, 31 de julio del 2013

    Circunferencias concurrentes y conjugados isogonales de ortocentros

    (Anopolis #532 Antreas P. Hatzipolakis)

    Sean ABC un triángulo, P un punto y Ha, Hb, Hc los ortocentros de los triángulos PBC, PCA, PAB, respectivamente.
    Denotamos por Da, Db, Dc los conjugados isogonales de Ha, Hb, Hc, con respecto al triángulo ABC.

    • Los puntos Da, Db, Dc están alineados con el circuncentro O en la recta dP de ecuación baricéntrica, si P=(u:v:w),
    dP:   b^2c^2u(a^2(w-v)+(b^2-c^2)(v+w))x + a^2c^2v(b^2(u-w)-a^2(u+w)+c^2(u+w))y + a^2b^2w(c^2(v-u) + a^2(u+v)- b^2(u+v))z =0.

    ( Mostrar/Ocultar figura )

    • Las circunferencias circunscritas a los triángulos DaBC, DbCA, DcAB son concurrentes en el punto D, sobre la recta dP, de coordenadas
    D = (a^2 v w (-a^2 (u + v) (u + w) + u (b^2 (u + v) + c^2 (u + w))) : b^2 u w (a^2 v (u + v) - (-c^2 v + b^2 (u + v)) (v + w)) : c^2 u v (-c^2 (u + w) (v + w) + w (a^2 (u + w) + b^2 (v + w))).

    El punto D es el QA-P4 in EQF (Encyclopedia of Quadri-Figures, Chris van Tienhoven)

    • Las circunferencias circunscritas a los triángulos ADbDc, BDcDa, CDaDb son concurrentes en el punto D', sobre la sobre la circunferencia circunscrita, de coordenadas baricéntricas:
    D' = ( a^2 (a^2 u - b^2 u - c^2 u + a^2 v - b^2 v + c^2 v) (a^2 u - b^2 u - c^2 u + a^2 w + b^2 w - c^2 w) (a^2 c^2 u^2 v^2 - b^2 c^2 u^2 v^2 - c^4 u^2 v^2 + a^2 c^2 u v^3 - b^2 c^2 u v^3 + c^4 u v^3 - a^4 u^2 v w + 3 a^2 b^2 u^2 v w - 2 b^4 u^2 v w + 2 a^2 c^2 u^2 v w - b^2 c^2 u^2 v w - c^4 u^2 v w + 2 a^2 b^2 u v^2 w - 2 b^4 u v^2 w + 2 b^2 c^2 u v^2 w + a^4 v^3 w - a^2 b^2 v^3 w + a^2 c^2 v^3 w - 2 b^4 u^2 w^2 - a^4 u v w^2 - a^2 b^2 u v w^2 - 2 b^4 u v w^2 + 2 a^2 c^2 u v w^2 + 3 b^2 c^2 u v w^2 - c^4 u v w^2 - a^4 v^2 w^2 - a^2 b^2 v^2 w^2 + a^2 c^2 v^2 w^2) (2 c^4 u^2 v^2 + a^4 u^2 v w - 2 a^2 b^2 u^2 v w + b^4 u^2 v w - 3 a^2 c^2 u^2 v w + b^2 c^2 u^2 v w + 2 c^4 u^2 v w + a^4 u v^2 w - 2 a^2 b^2 u v^2 w + b^4 u v^2 w + a^2 c^2 u v^2 w - 3 b^2 c^2 u v^2 w + 2 c^4 u v^2 w - a^2 b^2 u^2 w^2 + b^4 u^2 w^2 + b^2 c^2 u^2 w^2 - 2 a^2 c^2 u v w^2 - 2 b^2 c^2 u v w^2 + 2 c^4 u v w^2 + a^4 v^2 w^2 - a^2 b^2 v^2 w^2 + a^2 c^2 v^2 w^2 - a^2 b^2 u w^3 - b^4 u w^3 + b^2 c^2 u w^3 - a^4 v w^3 - a^2 b^2 v w^3 + a^2 c^2 v w^3) : ... : ... ).

    • Si P está en la circunferencia circunscrita, el ortocentro de HaHbHc es P y los triángulos ABC y HaHbHc son simétricos respecto al punto medio (sobre la circunferencia de los nueve puntos) de P y el ortocentro H.

    Cuando P varía sobre la circunferencia circunscrita, el punto D coincide con el circuncentro y el punto Q=PH∩dP describe la cúbica de Lemoine (K009 del catálogo de Bernard Gibert).
    ( Mostrar/Ocultar figura )
    La circunferencia circunscrita a OBC (donde queda Da) interseca a la cúbica de Lemoine además en dos puntos A1, A2. Procediendo cíclicamente, de definen los dos pares de puntos B1, B2 y C1, C2.
    Las rectas A1A2, B1BA2 y C1, C2 concurren en X1147, sobre la cúbica.

  • martes, 30 de julio del 2013

    Circunferencias concurrentes asociadas al triángulo ceviano de un punto

    (Anopolis #729 Antreas P. Hatzipolakis)

    Sean ABC un triángulo, P un punto y A'B'C' el triángulo ceviano de P. Se denota por Pa, Pb, Pc los conjugados isogonales de P con respecto a los triángulos AB'C', BC'A', CA'B', respectivamente.
    Entonces:
    1. Las circunferencias circunscritas a los triángulos PaB'C', PbC'A', PcA'B' son concurrentes en un punto R1.
    2. Las circunferencias circunscritas a los triángulos A'PbPc, B'PcPa, C'PaPb son concurrentes en un punto R2.

    ( Mostrar/Ocultar figura )
    Si P(u:v:w) se tiene que:
    Pa = (-a^2(u+v)(u+w) + u(b^2(u+v-w) + c^2(u-v+w)) : b^2u*w : c^2u*v),
    Pb = (a^2v*w : -b^2(v+w)(v+u) + v(c^2(v+w-u) + a^2(v-w+u)) : c^2u*v),
    Pc = (a^2v*w : b^2u*w : -c^2(w+u)(w+v) + w(a^2(w+u-v) + b^2(w-u+v))).


    Las coordenadas baricéntricas de R1=( f1(u,v,w)
      a^2*(-(a^2*c^2*u^4*v^2) + b^2*c^2*u^4*v^2 + c^4*u^4*v^2 - a^2*c^2*u^3*v^3 + b^2*c^2*u^3*v^3 + a^4*u^4*v*w - 2*a^2*b^2*u^4*v*w + b^4*u^4*v*w - 2*a^2*c^2*u^4*v*w + 2*b^2*c^2*u^4*v*w + c^4*u^4*v*w + 2*a^4*u^3*v^2*w - 4*a^2*b^2*u^3*v^2*w + 2*b^4*u^3*v^2*w - 4*a^2*c^2*u^3*v^2*w + b^2*c^2*u^3*v^2*w + 2*c^4*u^3*v^2*w + a^4*u^2*v^3*w - 2*a^2*b^2*u^2*v^3*w + b^4*u^2*v^3*w - 2*a^2*c^2*u^2*v^3*w - b^2*c^2*u^2*v^3*w - a^2*b^2*u^4*w^2 + b^4*u^4*w^2 + b^2*c^2*u^4*w^2 + 2*a^4*u^3*v*w^2 - 4*a^2*b^2*u^3*v*w^2 + 2*b^4*u^3*v*w^2 - 4*a^2*c^2*u^3*v*w^2 + b^2*c^2*u^3*v*w^2 + 2*c^4*u^3*v*w^2 + 4*a^4*u^2*v^2*w^2 - 5*a^2*b^2*u^2*v^2* w^2 + b^4*u^2*v^2*w^2 - 5*a^2*c^2*u^2*v^2*w^2 - 2*b^2*c^2*u^2* v^2*w^2 + c^4*u^2*v^2*w^2 + 2*a^4*u*v^3*w^2 - 2*a^2*b^2*u*v^3*w^2 - a^2*c^2*u*v^3*w^2 - a^2*b^2*u^3*w^3 + b^2*c^2*u^3*w^3 + a^4*u^2*v*w^3 - 2*a^2*b^2*u^2*v*w^3 - 2*a^2*c^2*u^2*v* w^3 - b^2*c^2*u^2*v*w^3 + c^4*u^2*v*w^3 + 2*a^4*u*v^2*w^3 - a^2*b^2*u*v^2*w^3 - 2*a^2*c^2*u*v^2* w^3 + a^4*v^3*w^3)* (-(a^2*c^2*u^4*v^2) + b^2*c^2*u^4*v^2 + c^4*u^4*v^2 - 2*a^2*c^2*u^3*v^3 + 2*b^2*c^2*u^3*v^3 - a^2*c^2*u^2*v^4 + b^2*c^2*u^2*v^4 - c^4*u^2*v^4 + a^4*u^4*v*w - 2*a^2*b^2*u^4*v*w + b^4*u^4*v*w - 2*a^2*c^2*u^4*v*w + 2*b^2*c^2*u^4*v*w + c^4*u^4*v*w + 2*a^4*u^3*v^2*w - 4*a^2*b^2*u^3*v^2*w + 2*b^4*u^3*v^2*w - 4*a^2*c^2*u^3*v^2*w + 4*b^2*c^2*u^3*v^2*w + 2*c^4*u^3*v^2*w + a^4*u^2*v^3*w - 2*a^2*b^2*u^2*v^3*w + b^4*u^2*v^3*w - 4*a^2*c^2*u^2*v^3*w + 4*b^2*c^2*u^2*v^3*w - 2*c^4*u^2*v^3*w - 2*a^2*c^2*u*v^4*w + 2*b^2*c^2*u*v^4*w - 2*c^4*u*v^4*w - a^2*b^2*u^4*w^2 + b^4*u^4*w^2 + b^2*c^2*u^4*w^2 + 2*a^4*u^3*v*w^2 - 4*a^2*b^2*u^3*v*w^2 + 2*b^4*u^3*v*w^2 - 4*a^2*c^2*u^3*v*w^2 + 4*b^2*c^2*u^3*v*w^2 + 2*c^4*u^3*v*w^2 + 4*a^4*u^2*v^2*w^2 - 4*a^2*b^2*u^2*v^2* w^2 - 4*a^2*c^2*u^2*v^2*w^2 + 6*b^2*c^2*u^2*v^2*w^2 + 2*a^4*u*v^3*w^2 - 2*b^4*u*v^3*w^2 - 2*a^2*c^2*u*v^3*w^2 + 6*b^2*c^2*u*v^3* w^2 - 4*c^4*u*v^3*w^2 + a^2*b^2*v^4*w^2 - b^4*v^4*w^2 - a^2*c^2*v^4*w^2 + 2*b^2*c^2*v^4*w^2 - c^4*v^4*w^2 - 2*a^2*b^2*u^3*w^3 + 2*b^2*c^2*u^3*w^3 + a^4*u^2*v*w^3 - 4*a^2*b^2*u^2*v*w^3 - 2*b^4*u^2*v*w^3 - 2*a^2*c^2*u^2*v*w^3 + 4*b^2*c^2*u^2*v* w^3 + c^4*u^2*v*w^3 + 2*a^4*u*v^2*w^3 - 2*a^2*b^2*u*v^2*w^3 - 4*b^4*u*v^2*w^3 + 6*b^2*c^2*u*v^2*w^3 - 2*c^4*u*v^2*w^3 + a^4*v^3*w^3 - 2*b^4*v^3*w^3 + 4*b^2*c^2*v^3*w^3 - 2*c^4*v^3*w^3 - a^2*b^2*u^2*w^4 - b^4*u^2*w^4 + b^2*c^2*u^2*w^4 - 2*a^2*b^2*u*v*w^4 - 2*b^4*u*v*w^4 + 2*b^2*c^2*u*v*w^4 - a^2*b^2*v^2*w^4 - b^4*v^2*w^4 + a^2*c^2*v^2*w^4 + 2*b^2*c^2*v^2*w^4 - c^4*v^2*w^4)
    :g1(u,v,w):h1(u,v,w)) son polinomios de grado 12.
    Las coordenadas baricéntricas de R2=( f2(u,v,w)
      a^10*v^3*(u + v)^3*(v - w)^2*w^3*(u + w)^3* (u + v + w) - u^3*(-(c^2*v) + b^2*(u + v))*(v + w)^5* (-(b^2*w) + c^2*(u + w))* (b^2*(u + v)*w - c^2*v*(u + w))^2* (b^2*(u + v - w) + c^2*(u - v + w)) + a^8*v^2*(u + v)^2*(v - w)*w^2*(u + w)^2* (-(b^2*(u + v)*w*(u^3*w + 2*v*w*(v + w)^2 + u^2*(3*v^2 + 2*v*w + 2*w^2) + u*(3*v^3 + 4*v^2*w + 2*v*w^2 + w^3))) + c^2*v*(u + w)* (u^3*v + 2*v*w*(v + w)^2 + u^2*(2*v^2 + 2*v*w + 3*w^2) + u*(v^3 + 2*v^2*w + 4*v*w^2 + 3*w^3))) - a^2*u^2*(v + w)^3* (-(b^8*(u + v)^3*w^3*(u^3*w - v*w*(v + w)^2 - u^2*(3*v^2 + v*w + w^2) + u*(-3*v^3 - 2*v^2*w + v*w^2 - 2*w^3))) + c^8*v^3*(u + w)^3*(-(u^3*v) + v*w*(v + w)^2 + u^2*(v^2 + v*w + 3*w^2) + u*(2*v^3 - v^2*w + 2*v*w^2 + 3*w^3)) - 2*b^6*c^2*(u + v)^2*w^2* (2*v^2*w^2*(v + w)^2 + u^4*(v^2 + w^2) + u^3*(5*v^3 + 3*v^2*w + v*w^2 + 2*w^3) + u*v*w*(7*v^3 + 6*v^2*w + 2*v*w^2 + 5*w^3) + u^2*(4*v^4 + 8*v^3*w + 3*v^2*w^2 + 6*v*w^3 + w^4)) - 2*b^2*c^6*v^2* (u + w)^2*(2*v^2*w^2*(v + w)^2 + u^4*(v^2 + w^2) + u^3*(2*v^3 + v^2*w + 3*v*w^2 + 5*w^3) + u*v*w*(5*v^3 + 2*v^2*w + 6*v*w^2 + 7*w^3) + u^2*(v^4 + 6*v^3*w + 3*v^2*w^2 + 8*v*w^3 + 4*w^4)) + b^4*c^4*v*(u + v)*w*(u + w)* (6*v^2*w^2*(v + w)^2 + 2*u^4*(2*v^2 + v*w + 2*w^2) + 6*u*v*w*(3*v^3 + 2*v^2*w + 2*v*w^2 + 3*w^3) + u^3*(11*v^3 + 7*v^2*w + 7*v*w^2 + 11*w^3) + u^2*(7*v^4 + 20*v^3*w + 12*v^2*w^2 + 20*v*w^3 + 7*w^4))) + a^6*v*(u + v)*w*(u + w)* (b^4*(u + v)^2*w^2*(v^2*w^2*(v + w)^3 + u^4*v*w*(3*v + w) + 2*u*v*w*(v + w)^2* (2*v^2 - v*w + w^2) + u^3*(2*v^4 + 4*v^3*w + 5*v^2*w^2 - w^4) + u^2*(2*v^5 + 3*v^4*w + 5*v^3*w^2 + 7*v^2*w^3 + 2*v*w^4 - w^5)) + c^4*v^2*(u + w)^2* (v^2*w^2*(v + w)^3 + u^4*v*w* (v + 3*w) + 2*u*v*w*(v + w)^2* (v^2 - v*w + 2*w^2) + u^3*(-v^4 + 5*v^2*w^2 + 4*v*w^3 + 2*w^4) + u^2*(-v^5 + 2*v^4*w + 7*v^3*w^2 + 5*v^2*w^3 + 3*v*w^4 + 2*w^5)) - 2*b^2*c^2*v*w* (v^3*w^3*(v + w)^3 + 4*u*v^2*w^2* (v + w)^2*(v^2 + w^2) + 2*u^5*(v^2 + v*w + w^2)^2 + u^6*(v^3 + v^2*w + v*w^2 + w^3) + u^3*v*w*(6*v^4 + 15*v^3*w + 20*v^2*w^2 + 15*v*w^3 + 6*w^4) + u^4*(v^5 + 6*v^4*w + 14*v^3*w^2 + 14*v^2*w^3 + 6*v*w^4 + w^5) + u^2*v*w*(3*v^5 + 10*v^4*w + 16*v^3*w^2 + 16*v^2*w^3 + 10*v*w^4 + 3*w^5))) - a^4*u*(v + w)*(-(b^2*c^4*v^2*w*(u + w)^2* (-3*v^3*(v - w)*w^2*(v + w)^2 + 2*u^5*v*(v^2 + v*w + w^2) + u^4*(9*v^4 + 10*v^3*w + 5*v^2*w^2 - 6*v*w^3 - 6*w^4) + u*v^2*w* (4*v^4 + 6*v^3*w - 5*v^2*w^2 - 6*v*w^3 + w^4) + u^2*v*(5*v^5 + 11*v^4*w + 16*v^3*w^2 - v^2*w^3 - 19*v*w^4 - 8*w^5) + u^3*(12*v^5 + 15*v^4*w + 10*v^3*w^2 - 5*v^2*w^3 - 16*v*w^4 - 6*w^5))) + c^6*v^3*(u + w)^3* (-(v^2*(v - w)*w^2*(v + w)^2) + u^4*v*(v^2 + 2*v*w + 3*w^2) + u*v^2*w*(2*v^3 + 3*v^2*w - 2*v*w^2 - 3*w^3) + 2*u^3*(v^4 + v^3*w + 2*v^2*w^2 - w^4) + u^2*(v^5 + 2*v^4*w + 6*v^3*w^2 + 2*v^2*w^3 - 5*v*w^4 - 2*w^5)) + b^6*(u + v)^3*w^3*(v^2*(v - w)*w^2* (v + w)^2 + u^4*w*(3*v^2 + 2*v*w + w^2) + u*v*w^2*(-3*v^3 - 2*v^2*w + 3*v*w^2 + 2*w^3) + 2*u^3*(-v^4 + 2*v^2*w^2 + v*w^3 + w^4) + u^2*(-2*v^5 - 5*v^4*w + 2*v^3*w^2 + 6*v^2*w^3 + 2*v*w^4 + w^5)) - b^4*c^2*v*(u + v)^2*w^2* (3*v^2*(v - w)*w^3*(v + w)^2 + 2*u^5*w*(v^2 + v*w + w^2) + u*v*w^2*(v^4 - 6*v^3*w - 5*v^2*w^2 + 6*v*w^3 + 4*w^4) + u^4*(-6*v^4 - 6*v^3*w + 5*v^2*w^2 + 10*v*w^3 + 9*w^4) + u^2*w*(-8*v^5 - 19*v^4*w - v^3*w^2 + 16*v^2*w^3 + 11*v*w^4 + 5*w^5) + u^3*(-6*v^5 - 16*v^4*w - 5*v^3*w^2 + 10*v^2*w^3 + 15*v*w^4 + 12*w^5)))
    :g2(u,v,w):h2(u,v,w)) son polinomios de grado 15.

    Si P=(a:b:c) es el incentro,

    R1 = (a^2(a^2-b^2-b*c-c^2)(a^3+a^2b-a*b^2-b^3+a^2c+a*b*c+b^2c-a*c^2+b*c^2-c^3)
    (a^6-2a^4b^2+a^2b^4+a^2b^3c-b^5c-2a^4c^2-a^2b^2c^2+a^2b*c^3+2b^3c^3+a^2c^4-b*c^5) :...:...),

    con número de búsqueda en ETC: 2.60524307195861030629200757

    R2 = ( (a-b-c) (b-c)^2 (a^3+a^2b-a*b^2-b^3+a^2c+a*b*c+b^2c-a*c^2+b*c^2-c^3)
    (a^6-a^4b^2-a^2b^4+b^6-2a^2b^3c+2b^5c-a^4c^2-a^2b^2c^2-b^4c^2-2a^2b*c^3 -4b^3c^3-a^2c^4-b^2c^4+2b*c^5+c^6):...:...),

    con número de búsqueda en ETC: -0.96947306546315331344307981

    Si P=(1:1:1) es el baricentro.

    R1 = (a^2(4a^4-7a^2b^2-2b^4-7a^2c^2+14b^2c^2-2c^4) (4a^4-6a^2b^2+2b^4-6a^2c^2+b^2c^2+2c^4):...:...),

    con número de búsqueda en ETC: 5.184059393723475028771865549

    R2 = ((b-c)^2(b+c)^2(-7a^2+2b^2+2c^2)(-2a^4+2b^4-5b^2c^2+2c^4):...:...),

    con número de búsqueda en ETC: -0.196430427865079067265962063

  • jueves, 25 de julio del 2013

    Propiedad del centro X3649

    ( Art of Problem Solving) Sean ABC un triángulo y DEF el triángulo pedal del incentro, X1. Las paralelas por D, E, F a las rectas de Euler de los triángulos BCX1, CAX1, ABX1, concurren en el centro X3649 ( KS(INTOUCH TRIANGLE), Peter Moses and Seiichi Kirikami):

    ( (b+c)(2a+b+c)(a-b+c)(a+b-c) : (c+a)(2b+c+a)(b-c+a)(b+c-a) : (a+b)(2c+a+b)(c-a+b)(c+a-b) ).


    Las paralelas citadas anteriormente son las rectas DO1, EO2, FO3, siendo O1, O2, O3 los circuncentros de los triángulos AEF, BFD, CDE, respectivamente.

    ( Mostrar/Ocultar figura )


  • miércoles, 24 de julio del 2013

    Rectángulos de diagonal mínima inscritos en un triángulo

    ( AdvancedPlaneGeometry #387 Angel Montesdeoca)

    Sea ABC un triángulo, de los rectángulos inscritos en ABC, con uno de sus lados sobre los lados del triángulo, tomemos los tres que tienen diagonal de longitud mínima. Los lados de estos rectángulos paralelos a los de ABC, delimitan un triángulo A'B'C' homotético a ABC, con centro de homotecia en el punto de coordenadas baricéntricas:

    U = ( a^4 / (3a^4+ 2a^2(b^2 + c^2) - (b^2 - c^2)^2 ) : b^4 / (3b^4+ 2b^2(c^2 + a^2) - (c^2 - a^2)^2 ) : c^4 / (3c^4+ 2c^2(a^2 + b^2) - (a^2 -b^2)^2 ) ) =
    = ( a^4/(a^4+S^2) : b^4/(b^4+S^2) : c^4/(c^4+S^2) ),

    que tiene números de búsqueda en : (1.41771794983177, 1.92240041055474, 1.65544052852413 ) y es la intersección de las rectas X39X394, X275X5286 y X577X3051.

    ( Mostrar/Ocultar figura )

    En consecuencia, los seis puntos de intersección de los lados de los triángulos ABC y A'B'C' están en una cónica.

    CONSTRUCCIÓN DE LOS RECTÁNGULOS INSCRITOS

    Sea K un punto variable sobre el lado BC y KLcMbNc un rectángulo, con Lc, Mb, Nc sobre los lados AB, AC, BC, respectivamente.
    El lugar geométrico de los puntos X y X' sobre la perpendicular por K a BC tales que KX=KX'=KMb, es una hipérbola de eje secundario BC y tangente en A a AB.
    Esta hipérbola pasa por A, cuando K es el pie de la altura por A (el rectángulo KLcMbNc degenera en el segmento KA); además, como KLc es menor que la diagonal KMb=KX, la recta AB es tangente a la hipérbola. El punto A', simétrico de A respecto a BC también es de la hipérbola.
    Otros puntos sobre la hipérbola son Xb, X'b sobre la perpendicular por B a BC, tales que KXb=KX'b=BC.

    Ya podemos construir la hipérbola, dado cuatro puntos y la tangente en uno de ellos (PPPtP1). Su ecuación baricéntrica puede obtenerse teniendo en cuenta que A(1:0:0), A'(2a^2:SB:SC), Xb(-a^2:SC-S:SB), X'b(-a^2 :SC+S:SB) y la tangente en A es z=0.
    La ecuación del haz de cónicas es:

    z (SB x + a^2 z) + t (SB y + (-SC + S) z) (-SB y + (SC + S) z)=0,

    e imponiendo que pase por A', t= -2 a^2/(4 S^2).

    La posición del punto K para el cual los puntos X y X' coinciden con los vértices V y V' de la hipérbola, corresponde a un vértice del rectángulo solución de diagonal mínima.

    A la misma solución se llega si se toma el rectángulo variable KLbMcNb el rectángulo con Lb, Mc, Nb sobre los lados AC, AB, BC, respectivamente.
    Los puntos Y y Y' sobre la perpendicular por K a BC tales que KY=KY'=KMc, describen una hipérbola de eje secundario BC y tangente en A a AC. La posición del punto K para el cual los puntos Y y Y' coinciden con los vértices W y W' de la hipérbola, corresponde a un vértice del rectángulo solución.
    ( Mostrar/Ocultar figura )


    Otra construcción, más sencilla, de los rectángulos de diagonal mínima es dada por Paul Yiu :
    ( Mostrar/Ocultar figura )
    (1) Let BB'C'C be the rectangle with the line B'C' containing the vertex A.
    (2) Construct the diagonal B'C and drop the perpendicular from B to intersect at X.
    (3) Construct the parallel through X to BC to intersect AC and AB at Ba and Ca respectively.

    BaCa is the side (parallel to BC) of the inscribed rectangle with minimum diagonal.


  • sábado, 20 de julio del 2013

    Cúbicas de Darboux y central de Spieker

    (Anopolis #638 Antreas P. Hatzipolakis)

    Sean ABC un triángulo, P un punto y A'B'C' el triángulo antipedal de P. Si P está sobre la cúbica de Darboux, las paralelas a IA', IB' y I'C (I el incentro) por A,B y C, concurren en un punto R sobre la cúbica central de Spieker (K033 del catálogo de Bernard Gibert).

    ( Mostrar/Ocultar figura )


    Cuando P coincide con los antipodales de A, B y C, en la circunferencia circunscrita, el triángulo antipedal A'B'C' de P degenera en los vértices de ABC y las rectas IA', IB', I'C son paralelas a las bisectrices (direcciones de las asíntotas de la cúbica K003).

  • viernes, 19 de julio del 2013

    Cúbica de Darboux y circunferencias coaxiales

    (Anopolis #626 Antreas P. Hatzipolakis)

    Sean ABC un triángulo, A1B1C1 el triángulo órtico y P un punto. Las rectas AP,BP,CP vuelven a cortas a las circunferencias circunscritas a los triángulos PBC,PCA,PAB, resp. en A',B',C', resp.

    Las circunferencias circunscritas a los triángulos AA1A', BB1B', CC1C' son coaxiales si P está sobre la cúbica de Darboux (K004 del catálogo de Bernard Gibert).
    El eje radical de las circunferencias coaxiales pasa por el ortocentro.

    ( Mostrar/Ocultar figura )


    Si P=X1, X3, X20, X40, X64, el eje radical de las tres circunferencia coaxiales pasa por X1 . (X6 (Art of Problem Solving)), X64, X57, X2 (recta de Euler), respectivamente.

  • jueves, 18 de julio del 2013

    Triángulos a mitad de camino

    (Anopolis #616 Antreas P. Hatzipolakis)

    Sean ABC un triángulo, P un punto y P* su conjugado isogonal. Denotamos por A',B',C' los puntos medios de AP, BP, CP, respectivamente, y por A",B",C" los puntos medios de AP*, BP*, CP*, respectivamente.
    Sean D,E,F los puntos medios de A'A", B'B", C'C", respectivamente, entonces los triángulos ABC y DEF son ortológicos (las perpendiculares desde los vértices de uno de ellos a los correspondientes lados del otro son concurrentes) y los centros ortológicos son el circuncentro y el punto de coordenadas, P(u:v:w):

    (a^2vw(4u+3v+3w)+ b^2uw(3u+2v+2w) + c^2uv(3u+2v+2w)) /
    3b^2c^4u^4v^2 + a^2c^4u^3v^3 + 5b^2c^4u^3v^3 + 3c^6u^3v^3 + a^2c^4u^2v^4 + 2b^2c^4u^2v^4 + 2c^6u^2v^4 + a^2b^2c^2u^4vw + 3b^4c^2u^4vw + 3b^2c^4u^4vw + 7a^2b^2c^2u^3v^2*w + 5b^4c^2u^3v^2w + 19b^2c^4u^3v^2w + a^4c^2u^2v^3w + 9a^2b^2c^2u^2v^3w + 2b^4c^2u^2v^3w + 9a^2c^4u^2v^3w + 12b^2c^4u^2v^3w + 2c^6u^2v^3w + a^4c^2uv^4w + 3a^2b^2c^2uv^4w + 5a^2c^4uv^4w + 3b^4c^2u^4w^2 + 7a^2b^2c^2u^3vw^2 + 19b^4c^2u^3vw^2 + 5b^2c^4u^3vw^2 + 34a^2b^2c^2u^2v^2w^2 + 12b^4c^2u^2v^2w^2 + 12b^2c^4u^2v^2w^2 + 7a^4c^2uv^3w^2 + 19a^2b^2c^2uv^3w^2 + 5a^2c^4uv^3w^2 + 3a^4c^2v^4w^2 + a^2b^4u^3w^3 + 3b^6u^3w^3 + 5b^4c^2u^3w^3 + a^4b^2u^2vw^3 + 9a^2b^4u^2vw^3 + 2b^6u^2vw^3 + 9a^2b^2c^2u^2vw^3 + 12b^4c^2u^2vw^3 + 2b^2c^4u^2vw^3 + 7a^4b^2uv^2w^3 + 5a^2b^4uv^2w^3 + 19a^2b^2c^2uv^2w^3 + a^6v^3w^3 + 3a^4b^2v^3w^3 + 3a^4c^2v^3w^3 + a^2b^4u^2w^4 + 2b^6u^2w^4 + 2b^4c^2u^2w^4 + a^4b^2uvw^4 + 5a^2b^4uvw^4 + 3a^2b^2c^2uvw^4 + 3a^4b^2v^2w^4 : ... : ...).

    ( Mostrar/Ocultar figura )

    Cuando P el baricentro o el circuncentro, estos puntos son, respectivamente:

    ( (10a^2 + 7(b^2+c^2))/ (a^6+15a^4(b^2+c^2) + 3a^2(7b^4+37b^2c^2+7c^4)+7(b^6+9b^4c^2+9b^2c^4+c^6)) : ... : ...)

    ( (2a^4 - 5a^2(b^2+c^2)+ 3(b^2-c^2)^2 )/ (a^6+a^4(b^2+c^2) - 5a^2(b^4+b^2c^2+c^4) + 3(b^2-c^2)^2 (b^2+c^2)) : ... : ...)



  • miércoles, 17 de julio del 2013

    Cúbica de Neuberg y circunferencias coaxiales

    (Anopolis #616 Antreas P. Hatzipolakis)

    Sean ABC un triángulo, A'B'C' el triángulo órtico, P un punto y A", B", C" los puntos medios de AP, BP, CP, respectivamente.
    Si P está sobre la cúbica de Neuberg (K001 del catálogo de Bernard Gibert) las circunferencias circunscritas a los triángulos AA'A", BB'B", CC'C", son coaxiales, con eje la recta de Euler.

    ( Mostrar/Ocultar figura )

    Cuando P=O los puntos de concurrencia de las circunferencias son el centro de la circunferencia de los nueve puntos, X5, y el inverso del ortocentro en la circunferencia circunscrita a ABC, X186.

  • martes, 16 de julio del 2013

    Puntos de Kosnita y circunferencias concurrentes

    (Anopolis #603 Antreas P. Hatzipolakis)

    Sean ABC un triángulo, I el incentro y Na, Nb, Nc los centros de las circunferencias de los nueve puntos de los triángulos IBC, ICA, IAB, resp.
    Se denota por Ka, Kb, Kc los conjugados isogonales de Na, Nb, Nc con respecto a los triángulos IBC, ICA, IAB, resp. (es decir, Ka, Kb, Kc son los puntos de Kosnita de los triángulos IBC, ICA, IAB).
    Las coordenadas baricéntricas del punto de Kosnita de IBC son:
    Ka = (a^2(a-b)(a-c) : b(a-b)(a^2- b^2+c^2-a(b+2c)) : c(a-c)(a^2+b^2-c^2-a(2b+c)).

    Las circunferencias circunscritas a ABC, AKbKc, BKcKa, CKaKb son concurrentes en el punto X de coordenadas baricéntricas:

    X = ( a^2/((b-c)(2a^4-a^3(b+c)-a^2(3b^2+4b*c+3c^2)+a(b+c)(b^2+3b*c+c^2)+(b^2-c^2)^2)) : ... : ... ),

    que tiene (6-9-13)-número de búsqueda en ETC: -0.4151794812633478152167512428

    ( Mostrar/Ocultar figura )


    Las circunferencias circunscritas a los triángulos KaBC, KbCA, KcAB concurren en X484.

  • domingo, 14 de julio del 2013

    Triángulos medial y órtico. Conjugados isogonales

    (Anopolis #588 Antreas P. Hatzipolakis)

    Sean ABC un triángulo, HaHbHc el triángulo órtico y MaMbMc triángulo medial.

    A'=(2a^4+a^2(b^2+c^2)-(b^2-c^2)^2: b^2(a^2+b^2-c^2):c^2(a^2-b^2+c^2)) es el conjugado isogonal de Ha respecto al triángulo GMbMc, B' el conjugado isogonal de Hb respecto al triángulo GMcMa y C' el conjugado isogonal de Hc respecto al triángulo GMaMb.

    Entonces, los triángulos ABC y A'B'C' son perspectivos, con centro de perspectividad X25, el centro de homotecia de los triángulo órtico y tangencial.

    ( Mostrar/Ocultar figura )


    A"=(-a^6(b^2+c^2)+3a^4(b^4+c^4)-3a^2(b^2-c^2)^2(b^2+c^2)+(b^2-c^2)^2(b^4+c^4) : b^2(a^2-b^2+c^2)(-a^2+b^2+c^2)^2 : c^2(a^2+b^2-c^2)(-a^2+b^2+c^2)^2) es el conjugado isogonal de Ma respecto al triángulo HHbHc, B" el conjugado isogonal de Mb respecto al triángulo HHcHa y C" el conjugado isogonal de Hc respecto al triángulo HHaHb.

    Entonces, los triángulos ABC y A"B"C" son perspectivos, con centro de perspectividad el circuncentro, X3.

    El centro radical R de las circunferencias circunscritas a los triángulos AA'A", BB'B", CC'C" es el punto de coordenadas baricéntricas:

    ( a^2( 3a^16(b^2+c^2) -
    a^14(9b^4+26b^2c^2+9c^4) +
    3a^12(b^6+16b^4c^2+16b^2c^4+c^6) +
    a^10(15b^8-24b^6c^2-62b^4c^4-24b^2c^6+15c^8) -
    a^8(15b^10+12b^8c^2-11b^6c^4-11b^4c^6+12b^2c^8+15c^10) +
    a^6(-3b^12+30b^10c^2+51b^8c^4+116b^6c^6+51b^4c^8+30b^2c^10-3c^12) +
    a^4(9b^14-36b^12c^2-26b^10c^4+85b^8c^6+85b^6c^8-26b^4c^10-36b^2c^12+9c^14) -
    a^2(b^2-c^2)^2(3b^12-14b^10c^2-3b^8c^4+76b^6c^6-3b^4c^8-14b^2c^10+3c^12)-
    3b^2c^2(b^2-c^2)^6(b^2+c^2)) : ... : ... ),

    que tiene (6-9-13)-número de búsqueda en ETC: 0.3197478385508268447724636294

    Otras perspectividades:
    • Los triángulos HaHbHc y A"B"C" son simétricos respecto al centro de la circunferencia de Taylor, X389.
    • Los triángulos MaMbMc y A'B'C' son perspectivos con centro de perspectividad el punto de coordenadas baricéntricas:

    ( (a^2+2SA)(a^4+SBSC+S^2) : (b^2+2SB)(b^4+SCSA+S^2) : (c^2+2SC)(c^4+SASB+S^2) ),

    que tiene (6-9-13)-número de búsqueda en ETC: 0.6225282852339818799280226572

  • sábado, 13 de julio del 2013

    Triángulos circuncevianos ortológicos con el triángulo de referencia

    (Anopolis #579 Antreas P. Hatzipolakis)

    Sean ABC un triángulo y P un punto, denotamos por A'B'C' el triángulo circunceviano de P.
    Ab y Ac son las proyecciones ortogonales de A' sobre BB' y CC', respectivamente y Ha el ortocentro de A'AbAc. Similarmente se consideran los correspondientes ortocentros Hb y Hc.
    Los triángulos ABC y HaHbHc son ortológicos (las perpendiculares desde los vértices de uno de ellos a los correspondientes lados del otro son concurrentes) en los siguientes casos:

    1. Cuando P está sobre la circunferencia circunscrita: el triángulo circunceviano degenera en el punto P.

    2. Cuando P está sobre la cúbica de Darboux (K004 del catálogo de Bernard Gibert).

    ( Mostrar/Ocultar figura )


    3. Cuando P está en sobre una de tres circunferencias de diámetros BC, CA, AB. Ocurre en este caso que los ortocentros Ha, Hb y Hc están alineados.


  • martes, 9 de julio del 2013

    Triángulos homotéticos y la quíntica Stothers

    ( AdvancedPlaneGeometry #292 Angel Montesdeoca)

    Sean ABC un triángulo, P un punto y k un número real. A'B'C' el triángulo homotético a ABC, mediante la homotecia de centro P y razón k.
    Sea Ab=BC∩C'A', Ac=BC∩A'B', y se definen Bc, Ba, Ca, Cb cíclicamente.
    Esta seis intersecciones están en una cónica, c(P,k).
    Cuando k varía los centros Z(P,k) de las cónicas c(P,k) quedan sobre una recta d(P) que pasa por P.

    d(P) es un eje de las cónicas c(P,k), para todo k, si y solo si P está en la quíntica Stothers o en la recta del infinito".

    ( Mostrar/Ocultar figura )

    Si P(u:v:w), d(P): vw(v-w)x + wu(w-u)y + uv(u-v)z=0 es un diámetro de las cónicas c(P,k). El punto del infinito del diámetro conjugado de d(P) es (u(v-w) : v(w-u) : w(u-v)).
    Estos diámetros son perpendiculares (ejes) si sólo si (u+v+w)(c^2u^3v^2 - c^2u^2v^3 - a^2u^2v^2w + b^2u^2v^2w - b^2u^3w^2 + a^2u^2v*w^2 - c^2u^2v*w^2 - b^2u*v^2w^2 + c^2u*v^2w^2 + a^2v^3w^2 + b^2u^2w^3 - a^2v^2w^3)=0.

  • lunes, 8 de julio del 2013

    Circunferencias concurrentes en la recta de Euler del triángulo excentral

    (Anopolis #529 Antreas P. Hatzipolakis)

    Sean ABC un triángulo, A'B'C' el triángulo excentral, y A",B",C" los puntos medios de las alturas A'A, B'B, C'C del triángulo excentral
    Las circunferencias circunscritas a los triángulos A"BC, B"CA, C"AB concurren en el punto (sobre la recta de Euler del triángulo excentral) X1319 = Bevan-Schröder point.

    ( Mostrar/Ocultar figura )


    Además, Oa, Ob y Oc son los circuncentros de los triángulos A"BC, B"CA y C"AB, las rectas A"Oa, B"Ob y C"Oc concurren en el punto de coordenadas baricéntricas:

    ( a (2 a^3 - a^2 (b + c) - 2 a (b^2 - b*c + c^2) + b^3 + b^2 c + c^2 b + c^3): ... : ... ),

    que tiene (6-9-13)-número de búsqueda en ETC: 2.763124242396552353038

  • domingo, 7 de julio del 2013

    Circunferencias concurrentes en la recta de Euler

    (Anopolis #527 Antreas P. Hatzipolakis)

    Sean ABC un triángulo, A'B'C' el triángulo órtico, y A",B",C" los puntos medios de las alturas.
    Las circunferencias circunscritas a los triángulos A"B'C', B"C'A', C"A'B' concurren en el punto (sobre la recta de Euler) de coordenadas baricéntricas:

    ( SB SC(SA(c^2-b^2)^2 - SB SC(2SA-a^2)): ... : ... ),

    que tiene (6-9-13)-número de búsqueda en ETC: -3.381534618985148558065036
    ( Mostrar/Ocultar figura )


  • sábado, 6 de julio del 2013

    Hipérbola circunscrita a un triángulo dada una asíntota

    ( AdvancedPlaneGeometry #269 Paul Yiu)

    Sean ABC un triángulo y una recta h. La hipérbola circunscrita a ABC y con asíntota h, puede ser construida como en el caso (PPPtP), cuando se dan cuatro puntos y la tangente en uno de ellos.
    También podemos hacer uso del hecho de que la otra asíntota es la conjugada isotómica de la asíntota dada , y acudir a cualquiera de los casos de construcción de una hipérbola dado un punto y dos asíntotas: (hhP) o (hhP(2)).

    ( Mostrar/Ocultar figura )


  • miércoles, 3 de julio del 2013

    Rectángulos sobre los lados de un triángulo

    (Anopolis #510 Antreas P. Hatzipolakis)

    Sean ABC un triángulo, DEF su triángulo medial y t un número real. Denotamos por D', E', F' las imágenes de D, E, F mediante las homotecias de centro en el circuncentro O y razón t. Consideremos los rectángulos BCCaBa, CAAbCb, ABBcAc tales que D', E', F' son, respectivamente, los puntos medios de los lados CaBa, AbCb, BcAc.
    Entonces, las mediatrices de AbAc, BcBa, CaCb concurren en el punto, sobre la recta de Euler, P(t)=3(1+t) X(2) - (1+3t) X(3)


    Los seis puntos Ab, Ac, Bc, Ba, Ca, Cb están en una cónica con centro Z(t) = t X(2) + (1+t) X(3).

  • martes, 2 de julio del 2013

    Triángulos con la recta HI que biseca un lado del triángulo medial

    ( Art of Problem Solving)

    Enunciado:
    Si ABC es un triángulo con B', C' los puntos medios de AC, AB, respectivamente, H y I el ortocentro e incentro de ABC. Probar que si IH biseca a B'C' entonces ABC es isósceles.

    Las coordenadas baricéntricas del punto de intersección de las rectas IH y B'C' son:
    (2a^4 - a^2(b-c)^2 - a^3(b+c) + a(b-c)^2(b+c) - (b^2-c^2)^2 : (a-c)(a^3 + a^2(b-c) - a(b-c)^2 - (b-c)(b+c)^2) : (a-b)(a^3 - a(b-c)^2 - a^2(b-c) + (b-c)(b+c)^2) ).
    Para que este punto coincida con el punto medio de B'C' de coordenadas (2:1:1), ha de ocurrir que b=c (solución trivial) o bien que:
    c = a - b + (2*2^(2/3)a(a - b))/ (3^(1/3)(a(a - b)(-9b + Sqrt[-48a^2 + 48a*b + 81b^2]))^(1/3)) + (2^(1/3)(a(a - b)(-9b + Sqrt[-48a^2 + 48a*b + 81b^2]))^(1/3))/3^(2/3).

    Así, existen triángulos NO isósceles para los cuales IH y B'C'.

    ( Mostrar/Ocultar figura )


  • viernes, 21 de junio del 2013

    Triángulos equiláteros de mismo baricentro y puntos en la hipérbola de Kiepert

    ( AdvancedPlaneGeometry #214 Dao Thanh Oai)

    Sean ABC un triángulo y los dos triángulos equiláteros A1A2A3, A'1A'2A'3, con el mismo baricentro G que ABC y tales que los segmentos A2A3, A'2A'3 son iguales y paralelos al segmento BC.

    ( Mostrar/Ocultar figura )

    Similarmente, se consideran la pareja de triángulos equiláteros B1B2B3, B'1B'2B'3, con el mismo baricentro G que ABC y tales que los segmentos B2B3, B'2B'3 son iguales y paralelos al segmento CA.
    Y se consideran la pareja de triángulos equiláteros C1C2C3, C'1C'2C'3, con el mismo baricentro G que ABC y tales que los segmentos C2C3, C'2C'3 son iguales y paralelos al segmento AB.

    Las coordenadas baricéntricas de los vértices de A1A2A3 son (S es el doble del área de ABC):
    A1 = ( a^4 + (b^2 - c^2)^2 - 2a^2(b^2 + c^2 - 2Sqrt[3]S) :
    a^4 + (b^2 - c^2)(b^2 - c^2 - 2Sqrt[3] S) - 2a^2(b^2 + c^2 + Sqrt[3] S) :
    a^4 + (b^2 - c^2)(b^2 - c^2 + 2Sqrt[3] S) - 2a^2(b^2 + c^2 + Sqrt[3] S))

    A2 = ( b^4 - 2b^2(a^2 + c^2 + Sqrt[3] S) + (-a^2 + c^2)(-a^2 + c^2 + 2Sqrt[3] S) :
    b^4 + (-a^2 + c^2)^2 + 2b^2(-a^2 - c^2 + 2Sqrt[3] S) :
    b^4 + (-a^2 + c^2)(-a^2 + c^2 - 2Sqrt[3] S) - 2b^2(a^2 + c^2 + Sqrt[3] S) )

    A3 = (c^4 + (a^2 - b^2)(a^2 - b^2 - 2Sqrt[3] S) - 2c^2(a^2 + b^2 + Sqrt[3] S) :
    c^4 - 2c^2(a^2 + b^2 + Sqrt[3] S) + (a^2 - b^2) (a^2 - b^2 + 2Sqrt[3] S) :
    (a^2 - b^2)^2 + c^4 + 2c^2(-a^2 - b^2 + 2Sqrt[3] S) )

    Las coordenadas baricéntricas de los vértices de A'1A'2A'3 (triángulo simétrico de A1A2A3 respecto a G) son:
    A'1 = (a^4 + (b^2 - c^2)^2 - 2a^2(b^2 + c^2 + 2 Sqrt[3] S) :
    a^4 + 2a^2(-b^2 - c^2 + Sqrt[3] S) + (b^2 - c^2) (b^2 - c^2 + 2 Sqrt[3] S) :
    a^4 + (b^2 - c^2)(b^2 - c^2 - 2 Sqrt[3] S) + 2a^2(-b^2 - c^2 + Sqrt[3] S))

    A'2 = (2(a^4 + (b^2 - c^2)^2 + 2a^2(-b^2 - c^2 + Sqrt[3] S)) :
    -a^4 + 2a^2(b^2 + c^2 - Sqrt[3] S) - (b^2 - c^2) (b^2 - c^2 + 2 Sqrt[3] S) :
    5a^4 + (b^2 - c^2)(5b^2 - 5c^2 + 2 Sqrt[3] S) - 2a^2(5b^2 + 5c^2 + Sqrt[3] S))

    A'3 = (2(a^4 + (b^2 - c^2)^2 + 2a^2(-b^2 - c^2 + Sqrt[3] S)) :
    5a^4 + (b^2 - c^2)(5b^2 - 5c^2 - 2 Sqrt[3] S) - 2a^2(5b^2 + 5c^2 + Sqrt[3] S) :
    -a^4 + 2a^2(b^2 + c^2 - Sqrt[3] S) + (b^2 - c^2)(-b^2 + c^2 + 2 Sqrt[3] S)).

    • Las rectas AA1, BB1 y CC1 se cortan en X13, primer punto isogonal (Fermat - Torricelli).

    ( Mostrar/Ocultar figura )

    • Las rectas AA'1, BB'1 y CC'1 se cortan en X14, segundo punto isogonal.

    ( Mostrar/Ocultar figura )

    • Denotamos por M1, N1 y P1 los puntos medios de A2A3, B2B3 y C2C3, respectivamente. Las rectas AM1, BN1 y CP1 se cortan en el centro de perspectividad de Kiepert K1 (sobre la hipérbola de Kiepert) de coordenadas baricéntricas:

    K1 = ( 1 / (SA-2S/√3) : 1 / (SB-2S/√3) : 1 / (SC-2S/√3) ),

    que tiene (6-9-13)-número de búsqueda en ETC: 1.05544963972706758475865636
    ( Mostrar/Ocultar figura )

    • Denotamos por M'1, N'1 y P'1 los puntos medios de A'2A'3, B'2B'3 y C'2C'3, respectivamente. Las rectas AM'1, BN'1 y CP'1 se cortan en el punto K2 (sobre la hipérbola de Kiepert) de coordenadas baricéntricas:

    K2 = ( 1 / (SA+2S/√3) : 1 / (SB+2S/√3) : 1 / (SC+2S/√3) ),

    que tiene (6-9-13)-número de búsqueda en ETC: 0.982582066301352428967254767
    ( Mostrar/Ocultar figura )

    • Denotamos por M el punto medio de M1 y el punto medio Ma de BC, por N el punto medio de N1 y el punto medio Mb de CA y por P el punto medio Mc de P1 y el punto medio de AB. Las rectas AM, BN y CP se cortan en el punto K3 (sobre la hipérbola de Kiepert) de coordenadas baricéntricas:

    K3 = ( 1 / (SA-5S/√3) : 1 / (SB-5S/√3) : 1 / (SC -5S/√3) ),

    que tiene (6-9-13)-número de búsqueda en ETC: 4.995109899468789806283186502
    ( Mostrar/Ocultar figura )

    • Denotamos por M' el punto medio de M'1 y el punto medio Ma de BC, por N' el punto medio de N'1 y el punto medio Mb de CA y por P' el punto medio Mc de P'1 y el punto medio de AB. Las rectas AM', BN' y CP' se cortan en el punto K4 (sobre la hipérbola de Kiepert) de coordenadas baricéntricas:

    K4 = ( 1 / (SA+5S/√3) : 1 / (SB+5S/√3) : 1 / (SC+5S/√3) ),

    que tiene (6-9-13)-número de búsqueda en ETC: 1.781091236364735925107070176
    ( Mostrar/Ocultar figura )

    • Denotamos por Ga, Gb y Gb los baricentros de BCA1, CAB1 y ABC1, respectivamente. Las rectas AGa, BGb y CGc se cortan en el punto K5 (sobre la hipérbola de Kiepert) de coordenadas baricéntricas:

    K5 = ( 1 / (SA+4S/√3) : 1 / (SB+4S/√3) : 1 / (SC+4S/√3) ),

    que tiene (6-9-13)-número de búsqueda en ETC: 1.623294634697676359144692801
    ( Mostrar/Ocultar figura )

    • Denotamos por G'a, G'b y G'b los baricentros de BCA'1, CAB'1 y ABC'1, respectivamente. Las rectas AG'a, BG'b y CG'c se cortan en el punto K6 (sobre la hipérbola de Kiepert) de coordenadas baricéntricas:

    K6 = ( 1 / (SA-4S/√3) : 1 / (SB-4S/√3) : 1 / (SC-4S/√3) ),

    que tiene (6-9-13)-número de búsqueda en ETC: 7.402174494117207095178203633
    ( Mostrar/Ocultar figura )


    Otros centros del triángulo en esta configuración

    • Las rectas MaM1, MbN1 y McP1 concurren en X618, primer punto isogónico del triángulo medial.

    ( Mostrar/Ocultar figura )

    • Las rectas MaM'1, MbN'1 y McP'1 concurren en X619, segundo punto isogónico del triángulo medial.

    ( Mostrar/Ocultar figura )

    • Las rectas MM', NN' y PP' concurren en X549, punto medio del baricentro y el circuncentro de ABC.

    ( Mostrar/Ocultar figura )

    • Las rectas GaG'a, GbG'b y GcG'c concurren en X3524, en la recta de Euler de ABC.

    ( Mostrar/Ocultar figura )

    • Las rectas MGa, NGb y PGc concurren en el punto de coordenadas baricéntricas:

    ( 53a^4 + 11(b^2-c^2)^2 - 64a^2(b^2+c^2) - 2√3(a^2+3b^2+3c^2)S : ... : ... ),

    que tiene (6-9-13)-número de búsqueda en ETC: 4.3452957082099683645807850740
    ( Mostrar/Ocultar figura )

    • Las rectas M'G'a, N'G'b y P'G'c concurren en el punto de coordenadas baricéntricas:

    ( 53a^4 + 11(b^2-c^2)^2 - 64a^2(b^2+c^2) + 2√3(a^2+3b^2+3c^2)S : ... : ... ),

    que tiene (6-9-13)-número de búsqueda en ETC: 6.729170058363700358702487375
    ( Mostrar/Ocultar figura )


  • jueves, 20 de junio del 2013

    El centro del triángulo X(3162) como centro de una cónica

    Sean ABC un triángulo, P un punto, D, E y F los pies de las cevianas de P en los lados BC, CA y AB. Denotamos por A', B' y C' los puntos donde las mediatrices de AD, BE y CF cortan a BC, CA y AB, respectivamente. El lugar geométrico de los puntos P tales que A', B' y C' estén alineados es una séxtica ( Mostrar/Ocultar fórmula )

      a^4*c^2*x^4*y^2 - b^4*c^2*x^4*y^2 - a^2*c^4*x^4*y^2 + b^2*c^4*x^4*y^2 + 2*a^4*c^2*x^3*y^3 - 2*b^4*c^2*x^3*y^3 + a^4*c^2*x^2*y^4 - b^4*c^2*x^2*y^4 - a^2*c^4*x^2*y^4 + b^2*c^4*x^2*y^4 - 2*b^4*c^2*x^4*y*z + 2*b^2*c^4*x^4*y*z + 2*a^4*c^2*x^3*y^2*z - 4*b^4*c^2*x^3*y^2*z - 2*a^2*c^4*x^3*y^2*z + 2*b^2*c^4*x^3*y^2*z + 4*a^4*c^2*x^2*y^3*z - 2*b^4*c^2*x^2*y^3*z - 2*a^2*c^4*x^2*y^3*z + 2*b^2*c^4*x^2*y^3*z + 2*a^4*c^2*x*y^4*z - 2*a^2*c^4*x*y^4*z - a^4*b^2*x^4*z^2 + a^2*b^4*x^4*z^2 - b^4*c^2*x^4*z^2 + b^2*c^4*x^4*z^2 - 2*a^4*b^2*x^3*y*z^2 + 2*a^2*b^4*x^3*y*z^2 - 2*b^4*c^2*x^3*y*z^2 + 4*b^2*c^4*x^3*y*z^2 - 2*a^4*b^2*x^2*y^2*z^2 + 2*a^2*b^4*x^2*y^2*z^2 + 2*a^4*c^2*x^2*y^2*z^2 - 2*b^4*c^2*x^2*y^2*z^2 - 2*a^2*c^4*x^2*y^2*z^2 + 2*b^2*c^4*x^2*y^2*z^2 - 2*a^4*b^2*x*y^3*z^2 + 2*a^2*b^4*x*y^3*z^2 + 2*a^4*c^2*x*y^3*z^2 - 4*a^2*c^4*x*y^3*z^2 - a^4*b^2*y^4*z^2 + a^2*b^4*y^4*z^2 + a^4*c^2*y^4*z^2 - a^2*c^4*y^4*z^2 - 2*a^4*b^2*x^3*z^3 + 2*b^2*c^4*x^3*z^3 - 4*a^4*b^2*x^2*y*z^3 + 2*a^2*b^4*x^2*y*z^3 - 2*b^4*c^2*x^2*y*z^3 + 2*b^2*c^4*x^2*y*z^3 - 2*a^4*b^2*x*y^2*z^3 + 4*a^2*b^4*x*y^2*z^3 + 2*a^4*c^2*x*y^2*z^3 - 2*a^2*c^4*x*y^2*z^3 + 2*a^2*b^4*y^3*z^3 - 2*a^2*c^4*y^3*z^3 - a^4*b^2*x^2*z^4 + a^2*b^4*x^2*z^4 - b^4*c^2*x^2*z^4 + b^2*c^4*x^2*z^4 - 2*a^4*b^2*x*y*z^4 + 2*a^2*b^4*x*y*z^4 - a^4*b^2*y^2*z^4 + a^2*b^4*y^2*z^4 + a^4*c^2*y^2*z^4 - a^2*c^4*y^2*z^4=0.
    que pasa por los vértices de ABC (puntos dobles) y por los centros X1, X4 y X8.
    Además, ocurre que los seis puntos en los que la séxtica vuelve a cortar a los lados de ABC están en una cónica ( Mostrar/Ocultar fórmula )
      Σ(a^6 - a^4 (b^2 + c^2) - a^2 (b^2 - c^2)^2 + (b^2 - c^2)^2 (b^2 + c^2)) x^2 - 2 (b^2 + c^2) (a^4 - (b^2 - c^2)^2) y*z=0.
    de centro en X3162 y perspector de coordenadas baricéntricas:

    ( 1 / (a^6 + a^4(b^2+c^2) - a^2((b^2+c^2)^2-b^2 c^2) - (b^2+c^2)(b^4-b^2c^2+c^4)) : ... : ... ),

    que tiene (6-9-13)-número de búsqueda en ETC: 8.11040473813735146364958223
    ( Mostrar/Ocultar figura )


  • miércoles, 19 de junio del 2013

    Rectas de Euler concurrentes

    (Anopolis #443 Antreas P. Hatzipolakis)

    Sean ABC un triángulo y A'B'C' su triángulo de reflexión (ie. A', B' y C' son los simétricos de A, B y C respecto a BC, CA y AB, resp.). Denotamos por:
    Ab y Ac las proyecciones ortogonales de A' sobre BB' y CC', resp.
    Bc y Ba las proyecciones ortogonales de B' sobre CC' y AA', resp.
    Ca y Cb las proyecciones ortogonales de C' sobre AA' y BB', resp.
    Entonces, las rectas de Euler La, Lb y Lc de los triángulos A'AbAc, B'BcBa y C'CaCb, respectivamente, son concurrentes en el punto X de coordenadas baricéntricas:

    ( a^2 (a^8(b^2+c^2) - 2a^6(b^4+c^4) + a^4b^2c^2(b^2+c^2) + a^2(b^2-c^2)^2(2b^4+b^2c^2+2c^4) - b^10+b^8c^2+b^2c^8-c^10) : ... : ... ),

    que tiene (6-9-13)-número de búsqueda en ETC: -6.61866665524197553640898833
    ( Mostrar/Ocultar figura )


    Generalización (Anopolis #442)
    Cuando A', B' y C' son puntos sobre AHa, BHb, CHc tal que:
    AA' / AHa = BB' / BHb = C'C / CHc = t,
    el lugar geométrico del punto de concurrencia de las rectas de Euler La, Lb y Lc de los triángulos A'AbAc, B'BcBa y C'CaCb, respectivamente, es la recta que pasa por los centros del triángulo X51, X125, X132, X427, X1112, X2781 y X5480.


  • martes, 18 de junio del 2013

    Rectas de Euler de dos triángulos ortológicos

    (Anopolis #436 Antreas P. Hatzipolakis)

    Sean ABC un triángulo, A'B'C' su triángulo de reflexión (ie. A', B' y C' son los simétricos de A, B y C respecto a BC, CA y AB, resp.) y AtBtCt el triángulo antipedal del circuncentro O (triángulo tangencial).
    Sean Oa, Ob y Oc los circuncentros de los triángulos AtB'C', BtC'A' y CtA'B', resp. Ellos quedan en las alturas de ABC (sobre las rectas AA', BB' y CC').
    Por tanto, ABC y OaObOc son ortológicos con un centro de ortología en el ortocentro H de ABC.
    El otro centro de ortología es el punto de intersección de las perpendiculares por A, B y C a las rectas ObOc, OcOa y OaOb, resp., de ecuaciones baricéntricas:

    -SC(4SB²-a²c²)y + SB(4SC²-a²b²)z=0,    SC(4SA²-b^2c²)x - SA(4SC²-b^2a²)z=0,    -SB(4SA²-c²b^2)x + SA(4SB²-c²a²)y =0.
    Que se cortan en X265.
    ( Mostrar/Ocultar figura )

    Las rectas de Euler de los triángulos ABC y OaObOc se cortan en el punto de coordenadas baricéntricas:

    ( a^8(b^2+c^2) - 2a^6(b^4-b^2c^2+c^4) - a^4b^2c^2(b^2+c^2) + a^2(b^2-c^2)^2(2b^4-b^2c^2+2c^4) - (b^2-c^2)^4(b^2+c^2) : ... : ... ),

    que tiene (6-9-13)-número de búsqueda en ETC: -1.67191299053365041718858431

  • lunes, 17 de junio del 2013

    Punto sobre la recta de Euler

    (Anopolis #440 Randy Hutson)

    Dado un triángulo ABC, sea DEF el triángulo acotado por las rectas La, Lb y Lc, donde La es la polar del ortocentro H con respecto a la circunferencia centrada en A y pasando por N, centro de la circunferencia de los nueve puntos; Lb y Lc se definen cíclicamente.

    El triángulo DEF es homotético a ABC con centro de homotecia X (en la recta de Euler), de coordenadas baricéntricas:

    ((a^6-3a^4(b^2+c^2)+a^2(3b^4+b^2c^2+3c^4)-(b^2-c^2)^2(b^2+c^2)) /(b^2+c^2-a^2) : ... : ... ),

    que tiene (6-9-13)-número de búsqueda en ETC: 1.7457649287207846078851083739
    ( Mostrar/Ocultar figura )


  • domingo, 16 de junio del 2013

    Circunferencias inscritas de igual radio

    (TrianguloCabri #685 #686 Ricardo Barroso)

    Dado un triángulo ABC, construir:
    1. Tres circunferencias de igual radio, tangentes a los lados AB y AC, BC y BA, CA y CB, respectivamente, y que tengan un punto común.
    2. Tres circunferencias de igual radio, tangentes a los lados AB y AC, BC y BA, CA y CB, respectivamente, y tangentes exteriormente a otra circunferencia, también de mismo radio.

    1.   La circunferencia inscrita a ABC tiene por ecuación baricéntrica:
    4(c2x y + b2x z + a2y z) - (x+y+z)((b+c-a)2x + (c+a-b)2y + (a+b-c)2z)=0.
    La ecuación de la circunferencia Γa imagen mediante la homotecia de centro A y razón k es:
    4(c2xy + b2xz + a2yz)- (x + y + z)((b + c - a)2k2x + ((b + c - a)k - 2c)2y + ((b + c - a)k - 2b)2z)=0.
    Similarmente, se obtienen las circunferencias Γb y Γc imágenes de la circunferencia inscrita, mediante las homotecias de razón k y centros en B y C.
    El centro radical E (sobre la recta IO) de las tres circunferencias Γa, Γb y Γc tiene por coordenadas :
    E = ( a(a3 - a2(b+c)k - a(b2+c2-2bck)+ (b-c)2(b+c)k) :
    b(b3 - b2(c+a)k - b(c2+a2-2cak)+ (c-a)2(c+a)k) :
    c(c3 - c2(a+b)k - c(a2+b2-2abk)+ (a-b)2(a+b)k) ).
    Para que este punto esté en las tres circunferencias, se debe anular la expresión siguiente:
    (2abc + (a3 - a2b - ab2 + b3 - a2c - b2c - ac2 - bc2 + c3)k)
    (-2abc + (a3 - a2b - ab2 + b3 - a2c + 4abc - b2c - ac2 - bc2 + c3)k)=0.
    Los dos valores de k que la anulan, dan lugar a sendos centros radicales donde concurren las tres circunferencias. Estos puntos son:
    ( a2(b+c-a) : b2(c+a-b) : c2(a+b-c) )    y    (a2(a2-(b-c)2) : b2(b2-(c-a)2) : c2(c2-(a-b)2) ).
    Que son los centros de homotecia interior X55 y exterior X56, respectivamente, de las circunferencia inscrita y circunscrita a ABC.
    ( Mostrar/Ocultar figura )
    Los otros puntos de intersección de las circunferencias de igual radio, que se cortan en X55, forman un triángulo perspectivo con ABC, cuyo centro de perspectividad es X12: conjugado armónico del punto de Feuerback respecto al incentro y al centro de la circunferencia de los nueve puntos.
    ( Mostrar/Ocultar figura )
    Los otros puntos de intersección de las circunferencias de igual radio, que se cortan en X56, forman un triángulo perspectivo con ABC, cuyo centro de perspectividad es X11: punto de Feuerback, único punto común de las circunferencias inscrita y la de los nueve puntos.

    2.   El eje radical de la circunferencia Γa y la circunferencia Γ con el mismo radio, |kr|, y centro en E es:
    -4bc(k-1)(a3(b+c)k-a(b-c)2(b+c)k+(b2-c2)2k-a2(b2k+c2k+bc(1+k))) x +
    4c(k-1)(a^4c - a3b(b+c)k + ab(b-c)2(b+c)k + (b2-c2)2(c - bk) + a2(-2c3 + b2c(k-1) + b3k + bc2k)) y+
    4b(k-1) (a^4b - a3c(b+c)k + a(b - c)2c(b + c)k + (b2 - c2)2(b - ck) + a2(-2b3+ bc2(k-1) + b2ck+ c3k)) z=0.
    Esta recta es tangente a Γa si:
    (abc + (a3 - a2b - ab2 + b3 - a2c + abc - b2c - ac2 - bc2 + c3)k)
    (-abc + (a3 - a2b - ab2 + b3 - a2c + 3abc - b2c - ac2 - bc2 + c3)k) =0

    Resuelta esta ecuación en k, nos dan dos valores para la razón de homotecia que transforma la circunferencia inscrita en las circunferencias Γa, Γb y Γc. Los correspondientes centros radicales de cada terna son:
    ( a2(a2-b2+bc-c2) : b2(b2-c2+ca-a2) : c2(c2-a2+ab-b2) ),
    ( a2(a2-b2-bc-c2) : b2(b2-c2-ca-a2) : c2(c2-a2-ab-b2) ),
    que son, respectivamente, el inverso del incentro en la circunferencia circunscrita, X36, y el conjugado armónico de éste respecto al incentro y circuncentro, X35.

    Los puntos de tangencia de cada pareja de circunferencias Γa y Γ que se obtienen son:
    A1 =( 2a2b2c2(2a^4+ a2(-3b2+2bc-3c2) + (b2-c2)2) : -2a2b^4c2 (a2-b2-2ac+c2) : -2a2b2c^4(a2-2ab+b2-c2) ),
    A2=( 2a2b2c2(2a^4+(b2-c2)2- a2(3b2+2bc+3c2)) : -2a2b^4c2(a2-b2+2ac+c2) : -2a2b2c^4(a2+2ab+b2-c2) ).
    Permutando cíclicamente las coordenadas de estos puntos, se obtienen las de los puntos B1, C1 y B2, C2 de contacto de la circunferencia Γ con Γb y Γc. Se verifica que las rectas AA1, BB1 y CC1 concurren en el punto X56. ( Mostrar/Ocultar figura)
    Y las rectas AA2, BB2 y CC2 se cortan en el punto X55. ( Mostrar/Ocultar figura)


  • sábado, 15 de junio del 2013

    Reflexiones en la recta que pasa por dos puntos conjugados isogonales

    (Anopolis #420 Antreas P. Hatzipolakis)

    ◊  Sean ABC un triángulo, P y P* dos puntos conjugados isogonales y A'B'C' el triángulo antipedal de P.
    Las perpendiculares por A', B' y C' a las reflexiones de AP, BP y CP, respectivamente, respecto a PP* son concurrentes, en un punto Q sobre las circunferencia circunscrita a A'B'C'.

    En particular, si P=O (A'B'C' es el triángulo tangencial), el punto de concurrencia Q tiene coordenadas baricéntricas (Anopolis #419):

    ( a^2(a^16 - 2a^14(b^2+c^2)- 2a^12(b^4-5b^2c^2+c^4)+ 2a^10(b^2+c^2)(3b^4-7b^2c^2+3c^4) - a^8b^2c^2(3b^2-4c^2)(4b^2-3c^2)- 6a^6(b-c)^2(b+c)^2(b^2+c^2)(b^4-b^2c^2+c^4)+ a^4(b-c)^2(b+c)^2(2b^8+12b^6c^2-b^4c^4+12b^2c^6+2c^8)+ 2a^2(b-c)^2(b+c)^2(b^2+c^2)(b^8-4b^6c^2+3b^4c^4-4b^2c^6+c^8)- (b-c)^4(b+c)^4(b^8+2b^6c^2+2b^2c^6+c^8)) : ... : ...),

    y tiene (6-9-13)-número de búsqueda en ETC: -6.980189770294538.
    ( Mostrar/Ocultar figura )


    ◊  Sean ABC un triángulo, P y P* dos puntos conjugados isogonales y A'B'C' el triángulo antipedal de P.
    Para que las perpendiculares por A', B' y C' a las reflexiones de AP*, BP* y CP*, respectivamente, respecto a PP* sean concurrentes, el punto P ha de estar en la circunferencia circunscrita a ABC o sobre la curva algebraica de grado nueve (invariante por isoconjugación) de ecuación baricéntrica:
    ( Mostrar/Ocultar fórmula )
      -a^2*b^2*c^6*x^6*y^3 + b^2*c^8*x^6*y^3 - 3*a^2*b^2*c^6*x^5*y^4 - 3*a^2*b^2*c^6*x^4*y^5 - a^2*b^2*c^6*x^3*y^6 + a^2*c^8*x^3*y^6 + 3*b^4*c^6*x^6*y^2*z - 6*a^2*b^2*c^6*x^4*y^4*z + 3*a^4*c^6*x^2*y^6*z + 3*b^6*c^4*x^6*y*z^2 + 6*a^2*b^4*c^4*x^5*y^2*z^2 + 3*a^4*b^2*c^4*x^4*y^3*z^2 - 3*a^2*b^2*c^6*x^4*y^3*z^2 + 3*a^2*b^4*c^4*x^3*y^4*z^2 - 3*a^2*b^2*c^6*x^3*y^4*z^2 + 6*a^4*b^2*c^4*x^2*y^5*z^2 + 3*a^6*c^4*x*y^6*z^2 - a^2*b^6*c^2*x^6*z^3 + b^8*c^2*x^6*z^3 + 3*a^4*b^4*c^2*x^4*y^2*z^3 - 3*a^2*b^6*c^2*x^4*y^2*z^3 - 3*a^6*b^2*c^2*x^2*y^4*z^3 + 3*a^4*b^4*c^2*x^2*y^4*z^3 + a^8*c^2*y^6*z^3 - a^6*b^2*c^2*y^6*z^3 - 3*a^2*b^6*c^2*x^5*z^4 - 6*a^2*b^6*c^2*x^4*y*z^4 - 3*a^2*b^6*c^2*x^3*y^2*z^4 + 3*a^2*b^4*c^4*x^3*y^2*z^4 - 3*a^6*b^2*c^2*x^2*y^3*z^4 + 3*a^4*b^2*c^4*x^2*y^3*z^4 - 6*a^6*b^2*c^2*x*y^4*z^4 - 3*a^6*b^2*c^2*y^5*z^4 - 3*a^2*b^6*c^2*x^4*z^5 + 6*a^4*b^4*c^2*x^2*y^2*z^5 - 3*a^6*b^2*c^2*y^4*z^5 + a^2*b^8*x^3*z^6 - a^2*b^6*c^2*x^3*z^6 + 3*a^4*b^6*x^2*y*z^6 + 3*a^6*b^4*x*y^2*z^6 + a^8*b^2*y^3*z^6 - a^6*b^2*c^2*y^3*z^6=0.
    ( Mostrar/Ocultar figura )


  • viernes, 14 de junio del 2013

    Reflexiones de la recta que pasa por dos puntos conjugados isogonales

    (Anopolis #424 Antreas P. Hatzipolakis)

    Sean ABC un triángulo, P y P* dos puntos conjugados isogonales y A'B'C' el triángulo pedal de P.
    Las perpendiculares por A', B' y C' a las reflexiones de PP*, respecto a AP, BP y CP son concurrentes en un punto Q, si P está sobre la nónica que pasa por los vértices de ABC (puntos triples), por el incentro I, por el ortocentro H y por los exincentros Ia, Ib, Ic. ( Mostrar/Ocultar fórmula )

      -a^4*b^2*c^4*x^6*y^3 + 2*a^2*b^4*c^4*x^6*y^3 - b^6*c^4*x^6*y^3 + a^2*b^2*c^6*x^6*y^3 - b^4*c^6*x^6*y^3 - a^6*c^4*x^5*y^4 + 3*a^2*b^4*c^4*x^5*y^4 - 2*b^6*c^4*x^5*y^4 + 2*a^4*c^6*x^5*y^4 - 3*a^2*b^2*c^6*x^5*y^4 + b^4*c^6*x^5*y^4 - a^2*c^8*x^5*y^4 + b^2*c^8*x^5*y^4 - 2*a^6*c^4*x^4*y^5 + 3*a^4*b^2*c^4*x^4*y^5 - b^6*c^4*x^4*y^5 + a^4*c^6*x^4*y^5 - 3*a^2*b^2*c^6*x^4*y^5 + 2*b^4*c^6*x^4*y^5 + a^2*c^8*x^4*y^5 - b^2*c^8*x^4*y^5 - a^6*c^4*x^3*y^6 + 2*a^4*b^2*c^4*x^3*y^6 - a^2*b^4*c^4*x^3*y^6 - a^4*c^6*x^3*y^6 + a^2*b^2*c^6*x^3*y^6 + 3*a^2*b^4*c^4*x^6*y^2*z - 3*b^6*c^4*x^6*y^2*z - 3*b^4*c^6*x^6*y^2*z + 2*a^4*b^2*c^4*x^5*y^3*z + 2*a^2*b^4*c^4*x^5*y^3*z - 4*b^6*c^4*x^5*y^3*z - 4*a^2*b^2*c^6*x^5*y^3*z + 2*b^4*c^6*x^5*y^3*z + 2*b^2*c^8*x^5*y^3*z - a^6*c^4*x^4*y^4*z + a^4*b^2*c^4*x^4*y^4*z + a^2*b^4*c^4*x^4*y^4*z - b^6*c^4*x^4*y^4*z + 2*a^4*c^6*x^4*y^4*z - 2*a^2*b^2*c^6*x^4*y^4*z + 2*b^4*c^6*x^4*y^4*z - a^2*c^8*x^4*y^4*z - b^2*c^8*x^4*y^4*z - 4*a^6*c^4*x^3*y^5*z + 2*a^4*b^2*c^4*x^3*y^5*z + 2*a^2*b^4*c^4*x^3*y^5*z + 2*a^4*c^6*x^3*y^5*z - 4*a^2*b^2*c^6*x^3*y^5*z + 2*a^2*c^8*x^3*y^5*z - 3*a^6*c^4*x^2*y^6*z + 3*a^4*b^2*c^4*x^2*y^6*z - 3*a^4*c^6*x^2*y^6*z + 3*a^2*b^4*c^4*x^6*y*z^2 - 3*b^6*c^4*x^6*y*z^2 - 3*b^4*c^6*x^6*y*z^2 + a^4*b^4*c^2*x^5*y^2*z^2 - 2*a^2*b^6*c^2*x^5*y^2*z^2 + b^8*c^2*x^5*y^2*z^2 + a^4*b^2*c^4*x^5*y^2*z^2 + 2*a^2*b^4*c^4*x^5*y^2*z^2 - b^6*c^4*x^5*y^2*z^2 - 2*a^2*b^2*c^6*x^5*y^2*z^2 - b^4*c^6*x^5*y^2*z^2 + b^2*c^8*x^5*y^2*z^2 + 2*a^6*b^2*c^2*x^4*y^3*z^2 - 3*a^4*b^4*c^2*x^4*y^3*z^2 + b^8*c^2*x^4*y^3*z^2 + 7*a^2*b^4*c^4*x^4*y^3*z^2 - b^6*c^4*x^4*y^3*z^2 - 3*a^2*b^2*c^6*x^4*y^3*z^2 - b^4*c^6*x^4*y^3*z^2 + b^2*c^8*x^4*y^3*z^2 + a^8*c^2*x^3*y^4*z^2 - 3*a^4*b^4*c^2*x^3*y^4*z^2 + 2*a^2*b^6*c^2*x^3*y^4*z^2 - a^6*c^4*x^3*y^4*z^2 + 7*a^4*b^2*c^4*x^3*y^4*z^2 - a^4*c^6*x^3*y^4*z^2 - 3*a^2*b^2*c^6*x^3*y^4*z^2 + a^2*c^8*x^3*y^4*z^2 + a^8*c^2*x^2*y^5*z^2 - 2*a^6*b^2*c^2*x^2*y^5*z^2 + a^4*b^4*c^2*x^2*y^5*z^2 - a^6*c^4*x^2*y^5*z^2 + 2*a^4*b^2*c^4*x^2*y^5*z^2 + a^2*b^4*c^4*x^2*y^5*z^2 - a^4*c^6*x^2*y^5*z^2 - 2*a^2*b^2*c^6*x^2*y^5*z^2 + a^2*c^8*x^2*y^5*z^2 - 3*a^6*c^4*x*y^6*z^2 + 3*a^4*b^2*c^4*x*y^6*z^2 - 3*a^4*c^6*x*y^6*z^2 - a^4*b^4*c^2*x^6*z^3 + a^2*b^6*c^2*x^6*z^3 + 2*a^2*b^4*c^4*x^6*z^3 - b^6*c^4*x^6*z^3 - b^4*c^6*x^6*z^3 + 2*a^4*b^4*c^2*x^5*y*z^3 - 4*a^2*b^6*c^2*x^5*y*z^3 + 2*b^8*c^2*x^5*y*z^3 + 2*a^2*b^4*c^4*x^5*y*z^3 + 2*b^6*c^4*x^5*y*z^3 - 4*b^4*c^6*x^5*y*z^3 + 2*a^6*b^2*c^2*x^4*y^2*z^3 - 3*a^2*b^6*c^2*x^4*y^2*z^3 + b^8*c^2*x^4*y^2*z^3 - 3*a^4*b^2*c^4*x^4*y^2*z^3 + 7*a^2*b^4*c^4*x^4*y^2*z^3 - b^6*c^4*x^4*y^2*z^3 - b^4*c^6*x^4*y^2*z^3 + b^2*c^8*x^4*y^2*z^3 + a^8*c^2*x^2*y^4*z^3 - 3*a^6*b^2*c^2*x^2*y^4*z^3 + 2*a^2*b^6*c^2*x^2*y^4*z^3 - a^6*c^4*x^2*y^4*z^3 + 7*a^4*b^2*c^4*x^2*y^4*z^3 - 3*a^2*b^4*c^4*x^2*y^4*z^3 - a^4*c^6*x^2*y^4*z^3 + a^2*c^8*x^2*y^4*z^3 + 2*a^8*c^2*x*y^5*z^3 - 4*a^6*b^2*c^2*x*y^5*z^3 + 2*a^4*b^4*c^2*x*y^5*z^3 + 2*a^6*c^4*x*y^5*z^3 + 2*a^4*b^2*c^4*x*y^5*z^3 - 4*a^4*c^6*x*y^5*z^3 + a^6*b^2*c^2*y^6*z^3 - a^4*b^4*c^2*y^6*z^3 - a^6*c^4*y^6*z^3 + 2*a^4*b^2*c^4*y^6*z^3 - a^4*c^6*y^6*z^3 - a^6*b^4*x^5*z^4 + 2*a^4*b^6*x^5*z^4 - a^2*b^8*x^5*z^4 - 3*a^2*b^6*c^2*x^5*z^4 + b^8*c^2*x^5*z^4 + 3*a^2*b^4*c^4*x^5*z^4 + b^6*c^4*x^5*z^4 - 2*b^4*c^6*x^5*z^4 - a^6*b^4*x^4*y*z^4 + 2*a^4*b^6*x^4*y*z^4 - a^2*b^8*x^4*y*z^4 + a^4*b^4*c^2*x^4*y*z^4 - 2*a^2*b^6*c^2*x^4*y*z^4 - b^8*c^2*x^4*y*z^4 + a^2*b^4*c^4*x^4*y*z^4 + 2*b^6*c^4*x^4*y*z^4 - b^4*c^6*x^4*y*z^4 + a^8*b^2*x^3*y^2*z^4 - a^6*b^4*x^3*y^2*z^4 - a^4*b^6*x^3*y^2*z^4 + a^2*b^8*x^3*y^2*z^4 + 7*a^4*b^4*c^2*x^3*y^2*z^4 - 3*a^2*b^6*c^2*x^3*y^2*z^4 - 3*a^4*b^2*c^4*x^3*y^2*z^4 + 2*a^2*b^2*c^6*x^3*y^2*z^4 + a^8*b^2*x^2*y^3*z^4 - a^6*b^4*x^2*y^3*z^4 - a^4*b^6*x^2*y^3*z^4 + a^2*b^8*x^2*y^3*z^4 - 3*a^6*b^2*c^2*x^2*y^3*z^4 + 7*a^4*b^4*c^2*x^2*y^3*z^4 - 3*a^2*b^4*c^4*x^2*y^3*z^4 + 2*a^2*b^2*c^6*x^2*y^3*z^4 - a^8*b^2*x*y^4*z^4 + 2*a^6*b^4*x*y^4*z^4 - a^4*b^6*x*y^4*z^4 - a^8*c^2*x*y^4*z^4 - 2*a^6*b^2*c^2*x*y^4*z^4 + a^4*b^4*c^2*x*y^4*z^4 + 2*a^6*c^4*x*y^4*z^4 + a^4*b^2*c^4*x*y^4*z^4 - a^4*c^6*x*y^4*z^4 - a^8*b^2*y^5*z^4 + 2*a^6*b^4*y^5*z^4 - a^4*b^6*y^5*z^4 + a^8*c^2*y^5*z^4 - 3*a^6*b^2*c^2*y^5*z^4 + a^6*c^4*y^5*z^4 + 3*a^4*b^2*c^4*y^5*z^4 - 2*a^4*c^6*y^5*z^4 - 2*a^6*b^4*x^4*z^5 + a^4*b^6*x^4*z^5 + a^2*b^8*x^4*z^5 + 3*a^4*b^4*c^2*x^4*z^5 - 3*a^2*b^6*c^2*x^4*z^5 - b^8*c^2*x^4*z^5 + 2*b^6*c^4*x^4*z^5 - b^4*c^6*x^4*z^5 - 4*a^6*b^4*x^3*y*z^5 + 2*a^4*b^6*x^3*y*z^5 + 2*a^2*b^8*x^3*y*z^5 + 2*a^4*b^4*c^2*x^3*y*z^5 - 4*a^2*b^6*c^2*x^3*y*z^5 + 2*a^2*b^4*c^4*x^3*y*z^5 + a^8*b^2*x^2*y^2*z^5 - a^6*b^4*x^2*y^2*z^5 - a^4*b^6*x^2*y^2*z^5 + a^2*b^8*x^2*y^2*z^5 - 2*a^6*b^2*c^2*x^2*y^2*z^5 + 2*a^4*b^4*c^2*x^2*y^2*z^5 - 2*a^2*b^6*c^2*x^2*y^2*z^5 + a^4*b^2*c^4*x^2*y^2*z^5 + a^2*b^4*c^4*x^2*y^2*z^5 + 2*a^8*b^2*x*y^3*z^5 + 2*a^6*b^4*x*y^3*z^5 - 4*a^4*b^6*x*y^3*z^5 - 4*a^6*b^2*c^2*x*y^3*z^5 + 2*a^4*b^4*c^2*x*y^3*z^5 + 2*a^4*b^2*c^4*x*y^3*z^5 + a^8*b^2*y^4*z^5 + a^6*b^4*y^4*z^5 - 2*a^4*b^6*y^4*z^5 - a^8*c^2*y^4*z^5 - 3*a^6*b^2*c^2*y^4*z^5 + 3*a^4*b^4*c^2*y^4*z^5 + 2*a^6*c^4*y^4*z^5 - a^4*c^6*y^4*z^5 - a^6*b^4*x^3*z^6 - a^4*b^6*x^3*z^6 + 2*a^4*b^4*c^2*x^3*z^6 + a^2*b^6*c^2*x^3*z^6 - a^2*b^4*c^4*x^3*z^6 - 3*a^6*b^4*x^2*y*z^6 - 3*a^4*b^6*x^2*y*z^6 + 3*a^4*b^4*c^2*x^2*y*z^6 - 3*a^6*b^4*x*y^2*z^6 - 3*a^4*b^6*x*y^2*z^6 + 3*a^4*b^4*c^2*x*y^2*z^6 - a^6*b^4*y^3*z^6 - a^4*b^6*y^3*z^6 + a^6*b^2*c^2*y^3*z^6 + 2*a^4*b^4*c^2*y^3*z^6 - a^4*b^2*c^4*y^3*z^6=0.
    ( Mostrar/Ocultar figura )


    Si P=H el punto de concurrencia de las perpendiculares es X125, centro de la hipérbola de Jerabek sobre la circunferencia de los nueve puntos.

  • jueves, 13 de junio del 2013

    Reflexiones de las bisectrices en la recta IO

    ( Anopolis #405Antreas P. Hatzipolakis)

    Sean ABC un triángulo, I su incentro, O su circuncentro, IaIbIc el triángulo excentral y La, Lb, Lc las reflexiones de la recta IO en las bisectrices AI, BI y CI, respectivamente.
    Las perpendiculares a La, Lb y Lc por Ia, Ib y Ic, respectivamente, concurren en el punto de coordenadas baricéntricas:

    ( a^9 - 2a^8(b+c) + 6a^7b*c + 2a^6(b-2c)(2b-c)(b+c) - a^5(4b^4+4b^3c-17b^2c^2+4c^3b+4c^4)- 2a^4(b-c)^2(b+c)(b^2-6b*c+c^2)+ a^3(b-c)^2(4b^4+2b^3c-11b^2c^2+2c^3b+4c^4)- 2a^2b*c(b-c)^2(b+c)(3b^2-5b*c+3c^2)- a(b-c)^6(b+c)^2 : ... : ... ),

    y tiene (6-9-13)-número de búsqueda en ETC: 26.33820686266028262904684.
    ( Mostrar/Ocultar figura )


    Más información sobre este punto (Randy Hutson, Anopolis #404):
    Es el Y476 del triángulo excentral y queda en la circunferencia circunscrita a éste.
    Es el antipodal del incentro en la circunferencia que pasa por los pies D, E y F de las perpendiculares trazadas desde los exincentros a las rectas La, Lb, Lc.

  • miércoles, 12 de junio del 2013

    Propiedad de una cúbica de Musselman

    (J.R.Musselman.- Some loci connected with a triangle. American Math. Monthly, p.354-361, June-July 1940)

    Sean ABC un triángulo, p una recta que pasa por el ortocentro H y P un punto (distinto de H) sobre p. Se denotan por D, E y F los puntos simétricos de H respecto a las perpendiculares por P a los lados BC, CA y AB, respectivamente, y consideramos los puntos:

    A'=BE∩CF     B'=CF∩AD     C'=AD∩BE,

    y las tres cónicas, (Ca) que pasa por B, C, H, P, A', (Cb) que pasa por C, A, H, P, B' y (Cc) que pasa por A, B, H, P, C'.
    Estas tres cónicas tienen un punto común Q. El lugar geométrico que describe Q cuando P se mueve sobre la recta p es una hipérbola rectangular (H) circunscrita a ABC.
    El segundo punto X de intersección de la hipérbola (H) son la recta p, describe la (tercera) cúbica de Musselman (K028 del catálogo de Bernard Gibert), cuando p gira alrededor de H.
    ( Mostrar/Ocultar figura )

    Si p es la recta de Euler, Q=X265 (para todo P) y X=X3.

    Información de Bernard Gibert:
    This description is a "remake" of CL055 since K028 is spK(X3, X5): a line through H meets the isogonal transform -hyperbola (H)- of its parallel at O at H and X.

  • sábado, 8 de junio del 2013

    Centro radical de circunferencias exinscritas

    ( Anopolis #365 Antreas P. Hatzipolakis)

    Sean ABC un triángulo y O su circuncentro. Consideramos las circunferencias exinscritas relativas al ángulo en O de los triángulos OBC, OCA y OAB; su centro radical es el punto de coordenadas baricéntricas:

    ( a(a^2-b^2-c^2)(2a^5(b+c)^2 - 4a^3(b^2+b*c+c^2)^2 + 2a(b-c)^2(b+c)^4 + 2S(b+c)(a^4-a^2(2b^2+5b*c+2c^2) + b^4-b^3c-c^3b+c^4)) : ... : ... ),

    donde S es el doble del área de ABC, y tiene (6-9-13)-número de búsqueda en ETC: 5.60319717911870517696190546.

    La circunferencia exinscrita a OBC, respecto al lado BC, toca a éste en su punto medio y tiene centro en el punto A1 = ( -2a^2(b*c+S) : b(a^2c+b^2c-c^3+2b*S) : c(a^2b-b^3+b*c^2+2c*S) ).
    ( Mostrar/Ocultar figura )

    Las paralelas por los vértices de ABC a los correspondientes ejes radicales se cortan en:

    ( a(a^2-b^2-c^2)(a^4-2a^2(b+c)^2 - 4a(b+c)S + (b^2-c^2)^2) : ... : ... ).

    con (6-9-13)-número de búsqueda en ETC: 6.13472549396753240388995633.
    X6213 Perspector of excentral triangle and inner Vecten triangle. (César E. Lozada - Dec. 2013).

  • viernes, 7 de junio del 2013

    Centros de circunferencias de los nueve puntos y paralelas por los vértices del triángulo de referencia

    ( Anopolis #361 Antreas P. Hatzipolakis)

    Sean ABC un triángulo, P un punto y A'B'C' su triángulo ceviano. Adoptemos las siguientes notaciones:

    Ab y Ac son las proyecciones ortogonales de A sobre BB' y CC',
    Bc y Ba son las proyecciones ortogonales de B sobre CC' y AA',
    Ca y Cb son las proyecciones ortogonales de C sobre AA' y BB',
    A2 y A3 son las proyecciones ortogonales de A' sobre BB' y CC',
    B3 y B1 son las proyecciones ortogonales de B' sobre CC' y AA',
    C1 y C2 son las proyecciones ortogonales de C' sobre AA' y BB'.

    Na, Nb y Nc los centros de las circunferencia de los nueve puntos de los triángulos AAbAc, BBcBa, CCaCb,
    N1, N2 y N3 los centros de las circunferencia de los nueve puntos de los triángulos A'A2A3, B'B3B1, C'C1C2.

    Se verifica:
    Las rectas N1Na, N2Nb y N3Nc concurren en el punto P. De hecho, N1N2N3 es el triángulo ceviano de P respecto a NaNbNc.

    ( Mostrar/Ocultar figura )


    Sea la, lb y lc las paralelas a N1Na, N2Nb y N3Nc por A, B y C, resp. y las paralelas ma, mb y mc por A', B' y C', resp.

    El lugar geométrico de los puntos P, tales que las rectas la, lb y lc son concurrentes, son dos curvas algebraicas de grados 6, Σ6, ( Mostrar/Ocultar fórmula )
      Σ6:
      2*a^4*c^2*x^4*y^2 - 4*a^2*b^2*c^2*x^4*y^2 + 2*b^4*c^2*x^4*y^2 - 4*a^2*c^4*x^4*y^2 + b^2*c^4*x^4*y^2 + 2*c^6*x^4*y^2 + 4*a^4*c^2*x^3*y^3 - 8*a^2*b^2*c^2*x^3*y^3 + 4*b^4*c^2*x^3*y^3 - 3*a^2*c^4*x^3*y^3 - 3*b^2*c^4*x^3*y^3 - c^6*x^3*y^3 + 2*a^4*c^2*x^2*y^4 - 4*a^2*b^2*c^2*x^2*y^4 + 2*b^4*c^2*x^2*y^4 + a^2*c^4*x^2*y^4 - 4*b^2*c^4*x^2*y^4 + 2*c^6*x^2*y^4 - a^6*x^4*y*z + 3*a^4*b^2*x^4*y*z - 3*a^2*b^4*x^4*y*z + b^6*x^4*y*z + 3*a^4*c^2*x^4*y*z - 7*a^2*b^2*c^2*x^4*y*z + 4*b^4*c^2*x^4*y*z - 3*a^2*c^4*x^4*y*z + 4*b^2*c^4*x^4*y*z + c^6*x^4*y*z - a^6*x^3*y^2*z + 3*a^4*b^2*x^3*y^2*z - 3*a^2*b^4*x^3*y^2*z + b^6*x^3*y^2*z + 4*a^4*c^2*x^3*y^2*z - 9*a^2*b^2*c^2*x^3*y^2*z + 5*b^4*c^2*x^3*y^2*z - 5*a^2*c^4*x^3*y^2*z - 8*b^2*c^4*x^3*y^2*z + 2*c^6*x^3*y^2*z + a^6*x^2*y^3*z - 3*a^4*b^2*x^2*y^3*z + 3*a^2*b^4*x^2*y^3*z - b^6*x^2*y^3*z + 5*a^4*c^2*x^2*y^3* z - 9*a^2*b^2*c^2*x^2*y^3*z + 4*b^4*c^2*x^2*y^3*z - 8*a^2*c^4*x^2*y^3*z - 5*b^2*c^4*x^2*y^3*z + 2*c^6*x^2*y^3*z + a^6*x*y^4*z - 3*a^4*b^2*x*y^4*z + 3*a^2*b^4*x*y^4*z - b^6*x*y^4*z + 4*a^4*c^2*x*y^4*z - 7*a^2*b^2*c^2*x*y^4*z + 3*b^4*c^2*x*y^4*z + 4*a^2*c^4*x*y^4*z - 3*b^2*c^4*x*y^4*z + c^6*x*y^4*z + 2*a^4*b^2*x^4*z^2 - 4*a^2*b^4*x^4*z^2 + 2*b^6*x^4*z^2 - 4*a^2*b^2*c^2*x^4*z^2 + b^4*c^2*x^4*z^2 + 2*b^2*c^4*x^4*z^2 - a^6*x^3*y*z^2 + 4*a^4*b^2*x^3*y*z^2 - 5*a^2*b^4*x^3*y*z^2 + 2*b^6*x^3*y*z^2 + 3*a^4*c^2*x^3*y*z^2 - 9*a^2*b^2*c^2*x^3*y*z^2 - 8*b^4*c^2*x^3*y*z^2 - 3*a^2*c^4*x^3*y*z^2 + 5*b^2*c^4*x^3*y*z^2 + c^6*x^3*y*z^2 - a^6*x^2*y^2*z^2 + a^4*b^2*x^2*y^2*z^2 + a^2*b^4*x^2*y^2*z^2 - b^6*x^2*y^2*z^2 + a^4*c^2*x^2*y^2*z^2 - 24*a^2*b^2*c^2*x^2*y^2*z^2 + b^4*c^2*x^2*y^2*z^2 + a^2*c^4*x^2*y^2*z^2 + b^2*c^4*x^2*y^2*z^2 - c^6*x^2*y^2*z^2 + 2*a^6*x*y^3*z^2 - 5*a^4*b^2*x*y^3*z^2 + 4*a^2*b^4*x*y^3*z^2 - b^6*x*y^3*z^2 - 8*a^4*c^2*x*y^3*z^2 - 9*a^2*b^2*c^2*x*y^3*z^2 + 3*b^4*c^2*x*y^3*z^2 + 5*a^2*c^4*x*y^3*z^2 - 3*b^2*c^4*x*y^3*z^2 + c^6*x*y^3*z^2 + 2*a^6*y^4*z^2 - 4*a^4*b^2*y^4*z^2 + 2*a^2*b^4*y^4*z^2 + a^4*c^2*y^4*z^2 - 4*a^2*b^2*c^2*y^4*z^2 + 2*a^2*c^4*y^4*z^2 + 4*a^4*b^2*x^3*z^3 - 3*a^2*b^4*x^3*z^3 - b^6*x^3*z^3 - 8*a^2*b^2*c^2*x^3*z^3 - 3*b^4*c^2*x^3*z^3 + 4*b^2*c^4*x^3*z^3 + a^6*x^2*y*z^3 + 5*a^4*b^2*x^2*y*z^3 - 8*a^2*b^4*x^2*y*z^3 + 2*b^6*x^2*y*z^3 - 3*a^4*c^2*x^2*y*z^3 - 9*a^2*b^2*c^2*x^2*y*z^3 - 5*b^4*c^2*x^2*y*z^3 + 3*a^2*c^4*x^2*y*z^3 + 4*b^2*c^4*x^2*y*z^3 - c^6*x^2*y*z^3 + 2*a^6*x*y^2*z^3 - 8*a^4*b^2*x*y^2*z^3 + 5*a^2*b^4*x*y^2*z^3 + b^6*x*y^2*z^3 - 5*a^4*c^2*x*y^2* z^3 - 9*a^2*b^2*c^2*x*y^2*z^3 - 3*b^4*c^2*x*y^2*z^3 + 4*a^2*c^4*x*y^2*z^3 + 3*b^2*c^4*x*y^2*z^3 - c^6*x*y^2*z^3 - a^6*y^3*z^3 - 3*a^4*b^2*y^3*z^3 + 4*a^2*b^4*y^3*z^3 - 3*a^4*c^2*y^3*z^3 - 8*a^2*b^2*c^2*y^3*z^3 + 4*a^2*c^4*y^3*z^3 + 2*a^4*b^2*x^2*z^4 + a^2*b^4*x^2*z^4 + 2*b^6*x^2*z^4 - 4*a^2*b^2*c^2*x^2*z^4 - 4*b^4*c^2*x^2*z^4 + 2*b^2*c^4*x^2*z^4 + a^6*x*y*z^4 + 4*a^4*b^2*x*y*z^4 + 4*a^2*b^4*x*y*z^4 + b^6*x*y*z^4 - 3*a^4*c^2*x*y*z^4 - 7*a^2*b^2*c^2*x*y*z^4 - 3*b^4*c^2*x*y*z^4 + 3*a^2*c^4*x*y*z^4 + 3*b^2*c^4*x*y*z^4 - c^6*x*y*z^4 + 2*a^6*y^2*z^4 + a^4*b^2*y^2*z^4 + 2*a^2*b^4*y^2*z^4 - 4*a^4*c^2*y^2*z^4 - 4*a^2*b^2*c^2*y^2*z^4 + 2*a^2*c^4*y^2*z^4=0.
    y de grado 7, Σ7, ( Mostrar/Ocultar fórmula )
      Σ7:
      -a^2*c^4*x^4*y^3 + b^2*c^4*x^4*y^3 + c^6*x^4*y^3 - a^2*c^4*x^3*y^4 + b^2*c^4*x^3*y^4 - c^6*x^3*y^4 + a^4*c^2*x^4*y^2*z - b^4*c^2*x^4*y^2*z - 2*a^2*c^4*x^4*y^2*z + 2*b^2*c^4*x^4*y^2*z + c^6*x^4*y^2*z + 2*a^4*c^2*x^3*y^3* z - 2*b^4*c^2*x^3*y^3*z - 2*a^2*c^4*x^3*y^3*z + 2*b^2*c^4*x^3*y^3*z + a^4*c^2*x^2*y^4*z - b^4*c^2*x^2*y^4*z - 2*a^2*c^4*x^2*y^4*z + 2*b^2*c^4*x^2*y^4*z - c^6*x^2*y^4*z - a^4*b^2*x^4*y*z^2 + 2*a^2*b^4*x^4*y*z^2 - b^6*x^4*y*z^2 - 2*b^4*c^2*x^4*y*z^2 + b^2*c^4*x^4*y*z^2 - a^4*b^2*x^3*y^2*z^2 + 2*a^2*b^4*x^3*y^2*z^2 - b^6*x^3*y^2*z^2 + a^4*c^2*x^3*y^2*z^2 - 3*b^4*c^2*x^3*y^2*z^2 - 2*a^2*c^4*x^3*y^2*z^2 + 3*b^2*c^4*x^3*y^2*z^2 + c^6*x^3*y^2*z^2 + a^6*x^2*y^3*z^2 - 2*a^4*b^2*x^2*y^3*z^2 + a^2*b^4*x^2*y^3*z^2 + 3*a^4*c^2*x^2*y^3*z^2 - b^4*c^2*x^2*y^3*z^2 - 3*a^2*c^4*x^2*y^3*z^2 + 2*b^2*c^4*x^2*y^3*z^2 - c^6*x^2*y^3*z^2 + a^6*x*y^4*z^2 - 2*a^4*b^2*x*y^4*z^2 + a^2*b^4*x*y^4*z^2 + 2*a^4*c^2*x*y^4*z^2 - a^2*c^4*x*y^4*z^2 + a^2*b^4*x^4*z^3 - b^6*x^4*z^3 - b^4*c^2*x^4*z^3 - 2*a^4*b^2*x^3*y*z^3 + 2*a^2*b^4*x^3*y*z^3 - 2*b^4*c^2*x^3*y*z^3 + 2*b^2*c^4*x^3*y*z^3 - a^6*x^2*y^2*z^3 - 3*a^4*b^2*x^2*y^2*z^3 + 3*a^2*b^4*x^2*y^2*z^3 + b^6*x^2*y^2*z^3 + 2*a^4*c^2*x^2*y^2*z^3 - 2*b^4*c^2*x^2*y^2*z^3 - a^2*c^4*x^2*y^2*z^3 + b^2*c^4*x^2*y^2*z^3 - 2*a^4*b^2*x*y^3*z^3 + 2*a^2*b^4*x*y^3*z^3 + 2*a^4*c^2*x*y^3*z^3 - 2*a^2*c^4*x*y^3*z^3 + a^6*y^4*z^3 - a^4*b^2*y^4*z^3 + a^4*c^2*y^4*z^3 + a^2*b^4*x^3*z^4 + b^6*x^3*z^4 - b^4*c^2*x^3*z^4 - a^4*b^2*x^2*y*z^4 + 2*a^2*b^4*x^2*y*z^4 + b^6*x^2*y*z^4 - 2*b^4*c^2*x^2*y*z^4 + b^2*c^4*x^2*y*z^4 - a^6*x*y^2*z^4 - 2*a^4*b^2*x*y^2*z^4 + a^2*b^4*x*y^2*z^4 + 2*a^4*c^2*x*y^2*z^4 - a^2*c^4*x*y^2*z^4 - a^6*y^3*z^4 - a^4*b^2*y^3*z^4 + a^4*c^2*y^3*z^4=0.
    que pasa por los vértices de ABC (puntos triples), por I(incentro), por H (ortocentro) y por los pies de las cevianas de X69.


    El lugar geométrico de los puntos P tales que las rectas ma, mb y mc son concurrentes son dos curvas algebraicas de grados 6, Σ6 (que coincide con el caso anterior), y de grado 10, Δ10, ( Mostrar/Ocultar fórmula )
      Δ10:
      -a^2*c^4*x^6*y^4 + b^2*c^4*x^6*y^4 + c^6*x^6*y^4 - 2*a^2*c^4*x^5*y^5 + 2*b^2*c^4*x^5*y^5 - a^2*c^4*x^4*y^6 + b^2*c^4*x^4*y^6 - c^6*x^4*y^6 + a^4*c^2*x^6*y^3*z - 2*a^2*b^2*c^2*x^6*y^3* z + b^4*c^2*x^6*y^3*z - 2*a^2*c^4*x^6*y^3*z + 2*b^2*c^4*x^6*y^3* z + c^6*x^6*y^3*z + a^4*c^2*x^5*y^4*z - 2*a^2*b^2*c^2*x^5*y^4*z + b^4*c^2*x^5*y^4*z - 4*a^2*c^4*x^5*y^4*z + 4*b^2*c^4*x^5*y^4*z + 3*c^6*x^5*y^4*z - a^4*c^2*x^4*y^5*z + 2*a^2*b^2*c^2*x^4*y^5* z - b^4*c^2*x^4*y^5*z - 4*a^2*c^4*x^4*y^5*z + 4*b^2*c^4*x^4*y^5* z - 3*c^6*x^4*y^5*z - a^4*c^2*x^3*y^6* z + 2*a^2*b^2*c^2*x^3*y^6*z - b^4*c^2*x^3*y^6*z - 2*a^2*c^4*x^3*y^6*z + 2*b^2*c^4*x^3*y^6*z - c^6*x^3*y^6*z + a^6*x^5*y^3*z^2 - 3*a^4*b^2*x^5*y^3*z^2 + 3*a^2*b^4*x^5*y^3*z^2 - b^6*x^5*y^3*z^2 + 2*a^4*c^2*x^5*y^3*z^2 - 2*b^4*c^2*x^5*y^3* z^2 - 7*a^2*c^4*x^5*y^3*z^2 + 5*b^2*c^4*x^5*y^3*z^2 + 4*c^6*x^5*y^3*z^2 + 2*a^6*x^4*y^4*z^2 - 6*a^4*b^2*x^4*y^4*z^2 + 6*a^2*b^4*x^4*y^4*z^2 - 2*b^6*x^4*y^4*z^2 + 4*a^4*c^2*x^4*y^4*z^2 - 4*b^4*c^2*x^4*y^4*z^2 - 6*a^2*c^4*x^4*y^4*z^2 + 6*b^2*c^4*x^4*y^4*z^2 + a^6*x^3*y^5*z^2 - 3*a^4*b^2*x^3*y^5*z^2 + 3*a^2*b^4*x^3*y^5*z^2 - b^6*x^3*y^5*z^2 + 2*a^4*c^2*x^3*y^5*z^2 - 2*b^4*c^2*x^3*y^5*z^2 - 5*a^2*c^4*x^3*y^5*z^2 + 7*b^2*c^4*x^3*y^5*z^2 - 4*c^6*x^3*y^5*z^2 - a^4*b^2*x^6*y*z^3 + 2*a^2*b^4*x^6*y*z^3 - b^6*x^6*y*z^3 + 2*a^2*b^2*c^2*x^6*y*z^3 - 2*b^4*c^2*x^6*y*z^3 - b^2*c^4*x^6*y*z^3 - a^6*x^5*y^2*z^3 - 2*a^4*b^2*x^5*y^2*z^3 + 7*a^2*b^4*x^5*y^2*z^3 - 4*b^6*x^5*y^2*z^3 + 3*a^4*c^2*x^5*y^2*z^3 - 5*b^4*c^2*x^5*y^2*z^3 - 3*a^2*c^4*x^5*y^2*z^3 + 2*b^2*c^4*x^5*y^2*z^3 + c^6*x^5*y^2*z^3 - 4*a^4*b^2*x^4*y^3*z^3 + 8*a^2*b^4*x^4*y^3*z^3 - 4*b^6*x^4*y^3*z^3 + 4*a^4*c^2*x^4*y^3*z^3 - 6*b^4*c^2*x^4*y^3*z^3 - 8*a^2*c^4*x^4*y^3*z^3 + 6*b^2*c^4*x^4*y^3*z^3 + 4*c^6*x^4*y^3*z^3 + 4*a^6*x^3*y^4*z^3 - 8*a^4*b^2*x^3*y^4*z^3 + 4*a^2*b^4*x^3*y^4*z^3 + 6*a^4*c^2*x^3*y^4*z^3 - 4*b^4*c^2*x^3*y^4*z^3 - 6*a^2*c^4*x^3*y^4*z^3 + 8*b^2*c^4*x^3*y^4*z^3 - 4*c^6*x^3*y^4*z^3 + 4*a^6*x^2*y^5*z^3 - 7*a^4*b^2*x^2*y^5*z^3 + 2*a^2*b^4*x^2*y^5*z^3 + b^6*x^2*y^5*z^3 + 5*a^4*c^2*x^2*y^5*z^3 - 3*b^4*c^2*x^2*y^5*z^3 - 2*a^2*c^4*x^2*y^5*z^3 + 3*b^2*c^4*x^2*y^5*z^3 - c^6*x^2*y^5*z^3 + a^6*x*y^6*z^3 - 2*a^4*b^2*x*y^6*z^3 + a^2*b^4*x*y^6*z^3 + 2*a^4*c^2*x*y^6*z^3 - 2*a^2*b^2*c^2*x*y^6*z^3 + a^2*c^4*x*y^6*z^3 + a^2*b^4*x^6*z^4 - b^6*x^6*z^4 - b^4*c^2*x^6*z^4 - a^4*b^2*x^5*y*z^4 + 4*a^2*b^4*x^5*y*z^4 - 3*b^6*x^5*y*z^4 + 2*a^2*b^2*c^2*x^5*y*z^4 - 4*b^4*c^2*x^5*y*z^4 - b^2*c^4*x^5*y*z^4 - 2*a^6*x^4*y^2*z^4 - 4*a^4*b^2*x^4*y^2*z^4 + 6*a^2*b^4*x^4*y^2*z^4 + 6*a^4*c^2*x^4*y^2*z^4 - 6*b^4*c^2*x^4*y^2*z^4 - 6*a^2*c^4*x^4*y^2*z^4 + 4*b^2*c^4*x^4*y^2*z^4 + 2*c^6*x^4*y^2*z^4 - 4*a^6*x^3*y^3*z^4 - 6*a^4*b^2*x^3*y^3*z^4 + 6*a^2*b^4*x^3*y^3*z^4 + 4*b^6*x^3*y^3*z^4 + 8*a^4*c^2*x^3*y^3*z^4 - 8*b^4*c^2*x^3*y^3*z^4 - 4*a^2*c^4*x^3*y^3*z^4 + 4*b^2*c^4*x^3*y^3*z^4 - 6*a^4*b^2*x^2*y^4*z^4 + 4*a^2*b^4*x^2*y^4*z^4 + 2*b^6*x^2*y^4*z^4 + 6*a^4*c^2*x^2*y^4*z^4 - 6*b^4*c^2*x^2*y^4*z^4 - 4*a^2*c^4*x^2*y^4*z^4 + 6*b^2*c^4*x^2*y^4*z^4 - 2*c^6*x^2*y^4*z^4 + 3*a^6*x*y^5*z^4 - 4*a^4*b^2*x*y^5*z^4 + a^2*b^4*x*y^5*z^4 + 4*a^4*c^2*x*y^5*z^4 - 2*a^2*b^2*c^2*x*y^5*z^4 + a^2*c^4*x*y^5*z^4 + a^6*y^6*z^4 - a^4*b^2*y^6*z^4 + a^4*c^2*y^6*z^4 + 2*a^2*b^4*x^5*z^5 - 2*b^4*c^2*x^5*z^5 + a^4*b^2*x^4*y*z^5 + 4*a^2*b^4*x^4*y*z^5 + 3*b^6*x^4*y*z^5 - 2*a^2*b^2*c^2*x^4*y*z^5 - 4*b^4*c^2*x^4*y*z^5 + b^2*c^4*x^4*y*z^5 - a^6*x^3*y^2*z^5 - 2*a^4*b^2*x^3*y^2*z^5 + 5*a^2*b^4*x^3*y^2*z^5 + 4*b^6*x^3*y^2*z^5 + 3*a^4*c^2*x^3*y^2*z^5 - 7*b^4*c^2*x^3*y^2*z^5 - 3*a^2*c^4*x^3*y^2*z^5 + 2*b^2*c^4*x^3*y^2*z^5 + c^6*x^3*y^2*z^5 - 4*a^6*x^2*y^3*z^5 - 5*a^4*b^2*x^2*y^3*z^5 + 2*a^2*b^4*x^2*y^3*z^5 + b^6*x^2*y^3*z^5 + 7*a^4*c^2*x^2*y^3*z^5 - 3*b^4*c^2*x^2*y^3*z^5 - 2*a^2*c^4*x^2*y^3*z^5 + 3*b^2*c^4*x^2*y^3*z^5 - c^6*x^2*y^3*z^5 - 3*a^6*x*y^4*z^5 - 4*a^4*b^2*x*y^4*z^5 - a^2*b^4*x*y^4*z^5 + 4*a^4*c^2*x*y^4*z^5 + 2*a^2*b^2*c^2*x*y^4*z^5 - a^2*c^4*x*y^4*z^5 - 2*a^4*b^2*y^5*z^5 + 2*a^4*c^2*y^5*z^5 + a^2*b^4*x^4*z^6 + b^6*x^4*z^6 - b^4*c^2*x^4*z^6 + a^4*b^2*x^3*y*z^6 + 2*a^2*b^4*x^3*y*z^6 + b^6*x^3*y*z^6 - 2*a^2*b^2*c^2*x^3*y*z^6 - 2*b^4*c^2*x^3*y*z^6 + b^2*c^4*x^3*y*z^6 - a^6*x*y^3*z^6 - 2*a^4*b^2*x*y^3*z^6 - a^2*b^4*x*y^3*z^6 + 2*a^4*c^2*x*y^3*z^6 + 2*a^2*b^2*c^2*x*y^3*z^6 - a^2*c^4*x*y^3*z^6 - a^6*y^4*z^6 - a^4*b^2*y^4*z^6 + a^4*c^2*y^4*z^6=0.
    que pasa por los vértices de ABC (puntos cuádruples) y por H (ortocentro).
    ( Mostrar/Ocultar figura )

    Cuando el punto P recorre la séxtica Σ6, los triángulos N1N2N3 y NaNbNc degeneran: sus vértices yacen en una misma recta. Y todas la rectas la, lb, lc, ma, mb y mc son paralelas a ella.

    Si P=I, el punto de concurrencia de las rectas la, lb y lc tiene coordenadas baricéntricas:

    ( a^4 - 3a^3(b+c)+ 9a^2b*c + 3a(b-c)^2(b+c) - (b^2-c^2)^2 : ... : ... ).


    (con (6-9-13)-número de búsqueda en ETC: 2.733571194876240757704921053)

    Si P=H, el punto de concurrencia de las rectas la, lb y lc tiene coordenadas baricéntricas:

    ( a^2/(4a^4 - (b^2-c^2)^2 - 3a^2(b^2+c^2)) : b^2/(4b^4 - (c^2-a^2)^2 - 3b^2(c^2+a^2)) : c^2/(4c^4 - (a^2-b^2)^2 - 3c^2(a^2+b^2)) ).

    Este punto es el conjugado isogonal de X550, es decir, es el punto donde concurren las tangentes en los vértices de ABC a la isocúbica pK(X6,X550) (ver la sección La isocúbica pK(X6,X550))

    Si P=H, el punto de concurrencia de las rectas ma, mb y mc tiene coordenadas baricéntricas:

    ( a^2(a^2+b^2-c^2)(a^2-b^2+c^2) (a^4 - 2a^2(b^2+c^2) + b^4+3b^2c^2+c^4 ) (a^4(b^2+c^2) - 2a^2(b^4-3b^2c^2+c^4) + (b^2-c^2)^2(b^2+c^2)): ... : ... ).


    (con (6-9-13)-número de búsqueda en ETC: -336.44857544131622352057412)

  • jueves, 6 de junio del 2013

    Un centro de una circunferencia de los nueve puntos sobre la recta de Euler

    ( Anopolis #336 Antreas P. Hatzipolakis)

    Sean ABC un triángulo, A'B'C' el antipedal de N (centro de la circunferencia de los nueve puntos de ABC) y N1, N2, N3 los centros de la circunferencias de los nueve puntos de los triángulos NB'C', NC'A', NA'B', respectivamente.

    El centro de la circunferencia de los nueve puntos del triángulo N1N2N3 es el punto X5500 (8th Hatzipolakis-Montesdeoca Point) que está en la recta de Euler de ABC y sus coordenadas baricéntricas son:

    ( 2a^22
    - 15a^20(b^2+c^2)
    + 6a^18(8b^4+13b^2c^2+8c^4)-(b^2-c^2)^8(b^6+b^4c^2+b^2c^4+c^6)
    - a^16(81b^6+152b^4c^2+152b^2c^4+81c^6)
    + a^14(64b^8+111b^6c^2+128b^4c^4+111b^2c^6+64c^8)
    + a^12(14b^10+29b^8c^2+36b^6c^4+36b^4c^6+29b^2c^8+14c^10)
    - a^10(84b^12+67b^10c^2+56b^8c^4+48b^6c^6+56b^4c^8+67b^2c^10+84c^12)
    + a^8(82b^14-23b^12c^2-31b^10c^4-19b^8c^6-19b^6c^8-31b^4c^10-23b^2c^12+82c^14)
    - a^6(b^2-c^2)^2(34b^12+11b^10c^2-30b^8c^4-35b^6c^6-30b^4c^8+11b^2c^10+34c^12)
    + a^4(b^2-c^2)^4(b^10-2b^8c^2-22b^6c^4-22b^4c^6-2b^2c^8+c^10)
    + a^2(b^2-c^2)^6(4b^8+5b^6c^2+8b^4c^4+5b^2c^6+4c^8) : ... : ... ).


    ( Mostrar/Ocultar figura )


  • miércoles, 5 de junio del 2013

    Circunferencias coaxiales y reflexiones de la recta de Euler

    Sean ABC un triángulo y l la recta de Euler, que corta a los lados BC, CA y AB en los puntos A', B' y C', respectivamente. Las circunferencias de diámetros AA', BB' y CC' son coaxiales y el eje d pasa por el ortocentro. Las tres circunferencias concurren en el punto X107 sobre la circunferencia circunscrita y en X125 (centro de la hipérbola de Jerabek: hipérbola equilátera circunscrita a ABC y que pasa por el circuncentro).

    ( Mostrar/Ocultar figura )

    Las reflexiones e1, e2 y e3 de la recta de Euler en los lados BC, CA y AB, respectivamente, se cortan en X110 (foco de la parábola de Kiepert), sobre la circunferencia circunscrita.

    Las reflexiones d1, d2 y d3 del eje d en los lados BC, CA y AB, respectivamente, se cortan en X1304, sobre la circunferencia circunscrita.

    Los puntos L11=e1∩d1, L22=e2∩d2 y L33=e3∩d3 son los vértices del triángulo circunceviano del ortocentro.

    Se definen los puntos L23=e2∩d3, L32=e3∩d2; L31=e3∩d1, L13=e1∩d3; L12=e1∩d2, L21=e2∩d1.
    Los triángulos L23L31L12 y L32L13L21 son perspectivos y su centro de perspectividad es el punto X de coordenadas baricéntricas:
    ( a^2(a^2-b^2)(a^2-c^2) (2a^6-4a^2(b^2-c^2)^2-a^4(b^2+c^2)+3(b^2-c^2)^2(b^2+c^2)): ... : ... ),
    (con (6-9-13)-número de búsqueda en ETC: 0.71653354827873787712696972)
    Incorporado a ETC como el punto X5502 (10th Hatzipolakis-Montesdeoca point).

  • martes, 4 de junio del 2013

    Circunferencias de los nueve puntos concurrentes

    ( Anopolis #354 Antreas P. Hatzipolakis)

    Sean ABC un triángulo, para todo punto P las circunferencias de los nueve puntos de los triángulos PBC, PCA y PAB son concurrentes, en un punto sobre la circunferencia de los nueve puntos de ABC.

    ( Mostrar/Ocultar figura )
    Si P=(u:v:w), en coordenadas baricéntricas, el punto de concurrencia Q (punto de Poncelet de de los puntos A, B, C y P) tiene coordenadas baricéntricas:
    ( u((a^2-b^2+c^2)v-(a^2+b^2-c^2)w)(b^2w(u+v)-c^2v(u+w)): v((b^2-c^2+a^2)w-(b^2+c^2-a^2)u)(c^2u(v+w)-a^2w(v+u)): w((c^2-a^2+b^2)u-(c^2+a^2-b^2)v)(a^2v(w+u)-b^2u(w+v)) ).

    Algunos ejemplos:
    Q=X11 si P=X1, X7, X8, X9, X21, X79, X80, X84, X90, X104, ...
    Q=X113 si P= X110, X112, ...
    Q=X114 si P= X99, ...
    Q=X115 si P= X2, X10, X13, X14, X17, X18, X76, X83, X94, X96, X98, ...
    Q=X116 si P= X103, ...
    Q=X117 si P= X109, ...
    Q=X118 si P= X101, ...
    Q=X119 si P= X100, ...
    Q=X122 si P= X20, ...
    Q=X124 si P=X58, X102, ...
    Q=X125 si P= X3, X6, X54, X64, X65, X66, X67, X68, X69, X70, X71, X72, X73, X74, ...
    Q=X127 si P=X22, ...
    Q=X130 si P=X51, ...
    Q=X133 si P=X107, ...
    Q=X134 si P=X52, ...
    Q=X135 si P=X34, ...
    Q=X137 si P=X5, X53, ...
    Q=X2679 si P=X32, ...
    Q=X3258 si P=X30, ...
    Q=X3259 si P=X56, ...
    Q=X5099 si P=X23, ...
    Q=X5139 si P=X25, ...
    Q=X5190 si P=X27, X92, ...

  • lunes, 3 de junio del 2013

    La isocúbica pK(X6,X550)

    ( Anopolis #351 Antreas P. Hatzipolakis)

    Sean ABC un triángulo, X un punto; N1, N2, N3 los centros de las circunferencias de los nueve puntos de los triángulos XBC, XCA, XAB, resp. and A'B'C' el triángulo antipedal de X. Los triángulos N1N2N3 y A'B'C' son perspectivos si y solo si X queda sobre la cúbica isogonal pK(X6,X550) (junto con la recta del infinito y la circunferencia circunscrita a ABC).

    ( Mostrar/Ocultar figura )

    Si X está sobre la cúbica pK(X6,X550) los triángulos N1N2N3 y A'B'C' son inversamente semejantes.

    El pivote P=X(550) de la isocúbica pK(X6,X550) es el punto medio del circuncentro y el punto de De Longchamps, X(20). La cúbica pK(X6,X550) es un elemento del Euler-haz de isocúbicas pivotales con pivote en la recta de Euler (OP = k OH, vectores, k=-1/2) y, por consiguiente, pasa por A, B , C, I=X(1), Ia, Ib, Ic (vértices del triángulo excentral), X(3), X(4), X(550),..; por los pies de las cevianas de X(550) (como en toda isocúbica, §1.4 Pivotal isocubics, Bernard Gibert.- Special Isocubics in the Triangle Plane).
    Las tangentes en los vértices A, B y C a la cúbica concurren en el punto P* (conjugado isogonal del pivote: 6º punto de intersección de la cúbica con la hipérbola equilátera circunscrita a ABC y que pasa por el circuncentro, hipérbola de Jerabek), de coordenadas baricéntricas:

    ( a^2(a^8 + a^6(b^2+c^2)+ a^4(-9b^4+19b^2c^2-9c^4) + 11a^2(b^2-c^2)^2(b^2+c^2) - (b^2-c^2)^2(4b^4+17b^2c^2+4c^4)): ... : ... ),

    que está sobre la cúbica y con (6-9-13)-número de búsqueda en ETC: -702.0040014335101057823010.

    Si X está en la circunferencia circunscrita a ABC su triángulo antipedal degenera en el punto antipodal de X.

  • domingo, 2 de junio del 2013

    Curvas algebraicas de grado cinco asociadas a centros de circunferencias de los nueve puntos

    ( Anopolis #344 Antreas P. Hatzipolakis)

    Sean ABC un triángulo, P un punto, N1, N2 y N3 los centros de las circunferencias de los nueve puntos de los triángulos PBC, PCA y PAB, respectivamente, O' el circuncentro de N1N2N3 y N' el centro de la circunferencia de los nueve puntos de N1N2N3.

    Si P=N (centro de la circunferencia de los nueve puntos de ABC), el punto O', que está en la recta de Euler de ABC, tiene coordenadas baricéntricas:

    ( -2a^16+ 9a^14(b^2+c^2)-
    a^10(b^6+b^4c^2+b^2c^4+c^6)+
    a^8(25b^8+10b^6c^2+8b^4c^4+10b^2c^6+25c^8) +
    a^6(-33b^10+31b^8c^2+11b^6c^4+11b^4c^6+31b^2c^8-33c^10)+
    a^4(b^2-c^2)^2(21b^8-20b^6c^2-25b^4c^4-20b^2c^6+21c^8)-
    a^2(b^2-c^2)^4(7b^6-13b^4c^2-13b^2c^4+7c^6)+
    (b^2-c^2)^6(b^4-4b^2c^2+c^4)-a^12(13b^4+18b^2c^2+13c^4): ... : ... )

    (con (6-9-13)-número de búsqueda en ETC: -3.187693716504994256890940). Incorporado a ETC como el punto X5501 (9th Hatzipolakis-Montesdeoca Point).

    Si P=O (centro de la circunferencia circunscrita a ABC), el punto N', que está en la recta de Euler de ABC, tiene coordenadas baricéntricas:

    ( 2a^10 - 5a^8(b^2+c^2) + 2a^6(b^4+5b^2c^2+c^4) + a^4(4b^6-5b^4c^2-5b^2c^4+4c^6) - a^2(b^2-c^2)^2(4b^4+5b^2c^2+4c^4) + (b^2-c^2)^4(b^2+c^2) : ... : ...)

    (con (6-9-13)-número de búsqueda en ETC: 4.7800096839999025703058)

    El lugar geométrico de P tal que N, P y O' están alineados es una curva algebraica de quinto grado que contiene a los vértices A, B y C (que son puntos doble) y pasa por los puntos X(1), X(4), X(5), ...
    Su ecuación baricéntrica es: ( Mostrar/Ocultar fórmula )
      a^6c^2x^3y^2 - 3a^4b^2c^2x^3y^2 + 3a^2b^4c^2x^3y^2 - b^6c^2x^3y^2 - 3a^4c^4x^3y^2 + 3a^2b^2c^4x^3y^2 + 3a^2c^6x^3y^2 + 2b^2c^6x^3y^2 - c^8x^3y^2 + a^6c^2x^2y^3 - 3a^4b^2c^2x^2y^3 + 3a^2b^4c^2x^2y^3 - b^6c^2x^2y^3 - 3a^2b^2c^4x^2y^3 + 3b^4c^4x^2y^3 - 2a^2c^6x^2y^3 - 3b^2c^6x^2y^3 + c^8x^2y^3 + a^2b^4c^2x^3yz - b^6c^2x^3yz - a^2b^2c^4x^3yz + b^2c^6x^3yz + 2a^4b^2c^2x^2y^2z - 2a^2b^4c^2x^2y^2z + a^6c^2xy^3z - a^4b^2c^2xy^3z + a^2b^2c^4xy^3z - a^2c^6xy^3z - a^6b^2x^3z^2 + 3a^4b^4x^3z^2 - 3a^2b^6x^3z^2 + b^8x^3z^2 + 3a^4b^2c^2x^3z^2 - 3a^2b^4c^2x^3z^2 - 2b^6c^2x^3z^2 - 3a^2b^2c^4x^3z^2 + b^2c^6x^3z^2 - 2a^4b^2c^2x^2yz^2 + 2a^2b^2c^4x^2yz^2 + 2a^2b^4c^2xy^2z^2 - 2a^2b^2c^4xy^2z^2 - a^8y^3z^2 + 3a^6b^2y^3z^2 - 3a^4b^4y^3z^2 + a^2b^6y^3z^2 + 2a^6c^2y^3z^2 + 3a^4b^2c^2y^3z^2 - 3a^2b^4c^2y^3z^2 + 3a^2b^2c^4y^3z^2 - a^2c^6y^3z^2 - a^6b^2x^2z^3 + 2a^2b^6x^2z^3 - b^8x^2z^3 + 3a^4b^2c^2x^2z^3 + 3a^2b^4c^2x^2z^3 + 3b^6c^2x^2z^3 - 3a^2b^2c^4x^2z^3 - 3b^4c^4x^2z^3 + b^2c^6x^2z^3 - a^6b^2xyz^3 + a^2b^6xyz^3 + a^4b^2c^2xyz^3 - a^2b^4c^2xyz^3 + a^8y^2z^3 - 2a^6b^2y^2z^3 + a^2b^6y^2z^3 - 3a^6c^2y^2z^3 - 3a^4b^2c^2y^2z^3 - 3a^2b^4c^2y^2z^3 + 3a^4c^4y^2z^3 + 3a^2b^2c^4y^2z^3 - a^2c^6y^2z^3=0
    ( Mostrar/Ocultar figura )

    El lugar geométrico del los puntos P tales que O (circuncentro de ABC), P y O' están alineados es una curva algebraica de grado cinco que pasa por los vértices A, B, C (que son puntos dobles), por los pies de las cevianas de X(5) y por los puntos X(3), X(4), X(5), ...
    Su ecuación baricéntrica es: ( Mostrar/Ocultar fórmula )
      a^4c^4x^3y^2 - a^2b^2c^4x^3y^2 - 2a^2c^6x^3y^2 - b^2c^6x^3y^2 + c^8x^3y^2 + a^2b^2c^4x^2y^3 - b^4c^4x^2y^3 + a^2c^6x^2y^3 + 2b^2c^6x^2y^3 - c^8x^2y^3 + a^6b^2x^3yz - 3a^4b^4x^3yz + 3a^2b^6x^3yz - b^8x^3yz - a^6c^2x^3yz + b^6c^2x^3yz + 3a^4c^4x^3yz - 3a^2c^6x^3yz - b^2c^6x^3yz + c^8x^3yz + a^8x^2y^2z - 2a^6b^2x^2y^2z + 2a^2b^6x^2y^2z - b^8x^2y^2z - 2a^6c^2x^2y^2z + 2b^6c^2x^2y^2z + a^4c^4x^2y^2z - b^4c^4x^2y^2z + a^8xy^3z - 3a^6b^2xy^3z + 3a^4b^4xy^3z - a^2b^6xy^3z - a^6c^2xy^3z + b^6c^2xy^3z - 3b^4c^4xy^3z + a^2c^6xy^3z + 3b^2c^6xy^3z - c^8xy^3z - a^4b^4x^3z^2 + 2a^2b^6x^3z^2 - b^8x^3z^2 + a^2b^4c^2x^3z^2 + b^6c^2x^3z^2 - a^8x^2yz^2 + 2a^6b^2x^2yz^2 - a^4b^4x^2yz^2 + 2a^6c^2x^2yz^2 + b^4c^4x^2yz^2 - 2a^2c^6x^2yz^2 - 2b^2c^6x^2yz^2 + c^8x^2yz^2 + a^4b^4xy^2z^2 - 2a^2b^6xy^2z^2 + b^8xy^2z^2 - 2b^6c^2xy^2z^2 - a^4c^4xy^2z^2 + 2a^2c^6xy^2z^2 + 2b^2c^6xy^2z^2 - c^8xy^2z^2 + a^8y^3z^2 - 2a^6b^2y^3z^2 + a^4b^4y^3z^2 - a^6c^2y^3z^2 - a^4b^2c^2y^3z^2 - a^2b^6x^2z^3 + b^8x^2z^3 - a^2b^4c^2x^2z^3 - 2b^6c^2x^2z^3 + b^4c^4x^2z^3 - a^8xyz^3 + a^6b^2xyz^3 - a^2b^6xyz^3 + b^8xyz^3 + 3a^6c^2xyz^3 - 3b^6c^2xyz^3 - 3a^4c^4xyz^3 + 3b^4c^4xyz^3 + a^2c^6xyz^3 - b^2c^6*xyz^3 - a^8y^2z^3 + a^6b^2y^2z^3 + 2a^6c^2y^2z^3 + a^4b^2c^2*y^2z^3 - a^4c^4y^2z^3=0
    ( Mostrar/Ocultar figura )

    El lugar geométrico del los puntos P tales que O (circuncentro de ABC), P y N' están alineados es una curva algebraica de grado cinco que pasa por los vértices A, B, C (que son puntos dobles), por los pies de las cevianas de X(3) y por los puntos X(3), X(4), ...
    Su ecuación baricéntrica es: ( Mostrar/Ocultar fórmula )
      a^2b^2c^4x^3y^2 - b^4c^4x^3y^2 + b^2c^6x^3y^2 + a^4c^4x^2y^3 - a^2b^2c^4x^2y^3 - a^2c^6x^2y^3 - a^6b^2x^3yz + 3a^4b^4x^3yz - 3a^2b^6x^3yz + b^8x^3yz + a^6c^2x^3yz + 2a^2b^4c^2x^3yz - 3b^6c^2x^3yz - 3a^4c^4x^3yz - 2a^2b^2c^4x^3yz + 3a^2c^6x^3yz + 3b^2c^6x^3yz - c^8x^3yz - a^8x^2y^2z + 2a^6b^2x^2y^2z - 2a^2b^6x^2y^2z + b^8x^2y^2z + 3a^6c^2x^2y^2z + a^4b^2c^2x^2y^2z - a^2b^4c^2x^2y^2z - 3b^6c^2x^2y^2z - 3a^4c^4x^2y^2z + 3b^4c^4x^2y^2z + a^2c^6x^2y^2z - b^2c^6x^2y^2z - a^8xy^3z + 3a^6b^2xy^3z - 3a^4b^4xy^3z + a^2b^6xy^3z + 3a^6c^2xy^3z - 2a^4b^2c^2xy^3z - b^6c^2xy^3z + 2a^2b^2c^4xy^3z + 3b^4c^4xy^3z - 3a^2c^6xy^3z - 3b^2c^6xy^3z + c^8xy^3z - a^2b^4c^2x^3z^2 - b^6c^2x^3z^2 + b^4c^4x^3z^2 + a^8x^2yz^2 - 3a^6b^2x^2yz^2 + 3a^4b^4x^2yz^2 - a^2b^6x^2yz^2 - 2a^6c^2x^2yz^2 - a^4b^2c^2x^2yz^2 + b^6c^2x^2yz^2 + a^2b^2c^4x^2yz^2 - 3b^4c^4x^2yz^2 + 2a^2c^6x^2yz^2 + 3b^2c^6x^2yz^2 - c^8x^2yz^2 + a^6b^2xy^2z^2 - 3a^4b^4xy^2z^2 + 3a^2b^6xy^2z^2 - b^8xy^2z^2 - a^6c^2xy^2z^2 + a^2b^4c^2xy^2z^2 + 2b^6c^2xy^2z^2 + 3a^4c^4xy^2z^2 - a^2b^2c^4xy^2z^2 - 3a^2c^6xy^2z^2 - 2b^2c^6xy^2z^2 + c^8xy^2z^2 + a^6c^2y^3z^2 + a^4b^2c^2y^3z^2 - a^4c^4y^3z^2 - a^4b^4x^2z^3 + a^2b^6x^2z^3 + a^2b^4c^2x^2z^3 + a^8xyz^3 - 3a^6b^2xyz^3 + 3a^2b^6xyz^3 - b^8xyz^3 - 3a^6c^2xyz^3 + 2a^4b^2c^2xyz^3 - 2a^2b^4c^2xyz^3 + 3b^6c^2xyz^3 + 3a^4c^4xyz^3 - 3b^4c^4xyz^3 - a^2c^6xyz^3 + b^2c^6xyz^3 - a^6b^2y^2z^3 + a^4b^4y^2z^3 - a^4b^2c^2y^2z^3=0
    ( Mostrar/Ocultar figura )


  • sábado, 1 de junio del 2013

    Perpendiculares a cevianas conjugadas isogonales

    ( Anopolis #343 Antreas P. Hatzipolakis)

    Sean ABC un triángulo y P un punto, P* su conjugado isogonal, A'B'C' y A"b"C" los triángulos cevianos de P y P*, respectivamente.

    A* es el punto de intersección de la perpendicular a AA' por A' y de la perpendicular a AA" por A",
    B* es el punto de intersección de la perpendicular a BB' por B' y de la perpendicular a BB" por B" y
    C* es el punto de intersección de la perpendicular a CC' por C' y de la perpendicular a CC" por C".

    Entonces, los triángulos ABC y A*B*C* son perspectivos, para cualquier punto P, y el centro de perspectividad es el circuncentro de ABC.

    ( Mostrar/Ocultar figura )

    Si las coordenadas baricéntricas de P son (u:v:w),
    A*=( a^2(v-w)-(b^2-c^2)(v+w))(a^2(c^2v-b^2w)+(b^2-c^2)(c^2v+b^2w)) : 2b^2(-a^2 + b^2 - c^2) (c^2v(v+w)+w(-a^2v+b^2(v+w))) : 2c^2(-a^2 - b^2 + c^2) (c^2v(v+w) +w(-a^2v +b^2(v+w))) )

    Por lo que la ecuación de la recta AA* es c^2(a^2+b^2-c^2)y - b^2(a^2-b^2+c^2)z=0, que pasa por el circuncentro de ABC.

  • viernes, 31 de mayo del 2013

    Caracterización geométrica de la cúbica de Napoleon-Feuerbach

    ( Anopolis #335 Antreas P. Hatzipolakis)

    Sean ABC un triángulo, P un punto, N, N1, N2, N3 los centro de las circunferencias de los nueve puntos de los triángulos ABC, PBC, PCA y PAB, respectivamente y N* el centro de la circunferencia de los nueve puntos del triángulo N1N2N3.

    Los puntos P, N y N* están alineados si y solo si P está en la circunferencia circunscrita a ABC o en la cúbica de Napoleon-Feuerbach (K005 del catálogo de Bernard Gibert)

    ( Mostrar/Ocultar figura )

    Si P varía sobre la circunferencia circunscrita a ABC, el lugar geométrico de N* es la circunferencia con centro en X140 y radio R/2.

  • jueves, 30 de mayo del 2013

    Reflexiones en cevianas y una curva algebraica de grado nueve

    Sea ABC un triángulo y A'B'C' el triángulo ceviano de un punto P. Se denota por
    L11 la perpendicular a AA' por A',
    L22 la perpendicular a BB' por B',
    L33 la perpendicular a CC' por C'.

    L12 la simétrica de L11 respecto a BB',
    L13 la simétrica de L11 respecto a CC'.

    A"=L12∩L13 y similarmente se definen B" y C".

    El lugar geométrico de los puntos P tal que los triángulos ABC y A"B"C" son perspectivos es una curva algebraica de grado nueve ( Mostrar/Ocultar ecuacíón )

      -2*a^2*c^4*x^5*y^4 + 2*b^2*c^4*x^5*
  y^4 + 2*c^6*x^5*y^4 - 
 2*a^2*c^4*x^4*y^5 + 2*b^2*c^4*x^4*
  y^5 - 2*c^6*x^4*y^5 - 
 a^4*c^2*x^5*y^3*z + 2*a^2*b^2*c^2*
  x^5*y^3*z - b^4*c^2*x^5*y^3*z + 
 2*a^2*c^4*x^5*y^3*z + 
 6*b^2*c^4*x^5*y^3*z - 
 c^6*x^5*y^3*z + a^4*c^2*x^3*y^5*z - 
 2*a^2*b^2*c^2*x^3*y^5*z + 
 b^4*c^2*x^3*y^5*z - 6*a^2*c^4*x^3*
  y^5*z - 2*b^2*c^4*x^3*y^5*z + 
 c^6*x^3*y^5*z - a^6*x^4*y^3*z^2 + 
 3*a^4*b^2*x^4*y^3*z^2 - 
 3*a^2*b^4*x^4*y^3*z^2 + 
 b^6*x^4*y^3*z^2 - a^4*c^2*x^4*y^3*
  z^2 + 2*a^2*b^2*c^2*x^4*y^3*z^2 - 
 b^4*c^2*x^4*y^3*z^2 + 
 5*a^2*c^4*x^4*y^3*z^2 + 
 3*b^2*c^4*x^4*y^3*z^2 - 
 3*c^6*x^4*y^3*z^2 - a^6*x^3*y^4*
  z^2 + 3*a^4*b^2*x^3*y^4*z^2 - 
 3*a^2*b^4*x^3*y^4*z^2 + 
 b^6*x^3*y^4*z^2 + a^4*c^2*x^3*y^4*
  z^2 - 2*a^2*b^2*c^2*x^3*y^4*z^2 + 
 b^4*c^2*x^3*y^4*z^2 - 
 3*a^2*c^4*x^3*y^4*z^2 - 
 5*b^2*c^4*x^3*y^4*z^2 + 
 3*c^6*x^3*y^4*z^2 + a^4*b^2*x^5*y*
  z^3 - 2*a^2*b^4*x^5*y*z^3 + 
 b^6*x^5*y*z^3 - 2*a^2*b^2*c^2*x^5*y*
  z^3 - 6*b^4*c^2*x^5*y*z^3 + 
 b^2*c^4*x^5*y*z^3 + a^6*x^4*y^2*
  z^3 + a^4*b^2*x^4*y^2*z^3 - 
 5*a^2*b^4*x^4*y^2*z^3 + 
 3*b^6*x^4*y^2*z^3 - 3*a^4*c^2*x^4*
  y^2*z^3 - 2*a^2*b^2*c^2*x^4*y^2*
  z^3 - 3*b^4*c^2*x^4*y^2*z^3 + 
 3*a^2*c^4*x^4*y^2*z^3 + 
 b^2*c^4*x^4*y^2*z^3 - 
 c^6*x^4*y^2*z^3 - 3*a^6*x^2*y^4*
  z^3 + 5*a^4*b^2*x^2*y^4*z^3 - 
 a^2*b^4*x^2*y^4*z^3 - 
 b^6*x^2*y^4*z^3 + 3*a^4*c^2*x^2*y^4*
  z^3 + 2*a^2*b^2*c^2*x^2*y^4*z^3 + 
 3*b^4*c^2*x^2*y^4*z^3 - 
 a^2*c^4*x^2*y^4*z^3 - 
 3*b^2*c^4*x^2*y^4*z^3 + 
 c^6*x^2*y^4*z^3 - a^6*x*y^5*z^3 + 
 2*a^4*b^2*x*y^5*z^3 - 
 a^2*b^4*x*y^5*z^3 + 6*a^4*c^2*x*y^5*
  z^3 + 2*a^2*b^2*c^2*x*y^5*z^3 - 
 a^2*c^4*x*y^5*z^3 + 2*a^2*b^4*x^5*
  z^4 - 2*b^6*x^5*z^4 - 
 2*b^4*c^2*x^5*z^4 + a^6*x^3*y^2*
  z^4 - a^4*b^2*x^3*y^2*z^4 + 
 3*a^2*b^4*x^3*y^2*z^4 - 
 3*b^6*x^3*y^2*z^4 - 3*a^4*c^2*x^3*
  y^2*z^4 + 2*a^2*b^2*c^2*x^3*y^2*
  z^4 + 5*b^4*c^2*x^3*y^2*z^4 + 
 3*a^2*c^4*x^3*y^2*z^4 - 
 b^2*c^4*x^3*y^2*z^4 - 
 c^6*x^3*y^2*z^4 + 3*a^6*x^2*y^3*
  z^4 - 3*a^4*b^2*x^2*y^3*z^4 + 
 a^2*b^4*x^2*y^3*z^4 - 
 b^6*x^2*y^3*z^4 - 5*a^4*c^2*x^2*y^3*
  z^4 - 2*a^2*b^2*c^2*x^2*y^3*z^4 + 
 3*b^4*c^2*x^2*y^3*z^4 + 
 a^2*c^4*x^2*y^3*z^4 - 
 3*b^2*c^4*x^2*y^3*z^4 + 
 c^6*x^2*y^3*z^4 + 2*a^6*y^5*z^4 - 
 2*a^4*b^2*y^5*z^4 + 2*a^4*c^2*y^5*
  z^4 + 2*a^2*b^4*x^4*z^5 + 
 2*b^6*x^4*z^5 - 2*b^4*c^2*x^4*z^5 - 
 a^4*b^2*x^3*y*z^5 + 6*a^2*b^4*x^3*y*
  z^5 - b^6*x^3*y*z^5 + 
 2*a^2*b^2*c^2*x^3*y*z^5 + 
 2*b^4*c^2*x^3*y*z^5 - 
 b^2*c^4*x^3*y*z^5 + a^6*x*y^3*z^5 - 
 6*a^4*b^2*x*y^3*z^5 + 
 a^2*b^4*x*y^3*z^5 - 2*a^4*c^2*x*y^3*
  z^5 - 2*a^2*b^2*c^2*x*y^3*z^5 + 
 a^2*c^4*x*y^3*z^5 - 2*a^6*y^4*z^5 - 
 2*a^4*b^2*y^4*z^5 + 2*a^4*c^2*y^4*
  z^5=0
    Esta curva contiene a A, B y C que son puntos cuádruples; dos de las cuatro tangentes en A son las dos bisectrices. Pasa por los pies de las alturas. Contiene al baricentro X2, y a los dos centros isogónicos X13 y X14.
    ( Mostrar/Ocultar figura )

    Cuando P=X2, las rectas AA", BB" y BB" son paralelas, con punto del infinito X524. Si P es uno de los centros isogónicos el centro de perspectividad Q de ABC y A"B"C" coincide con cada uno de ellos.

  • miércoles, 29 de mayo del 2013

    Circunferencias concurrentes con centros cocíclicos

    ( Anopolis #327 Antreas P. Hatzipolakis)

    Sea ABC un triángulo y A'B'C' el triángulo ceviano del incentro I. Se denota por
    L11 la perpendicular a AA' por A',
    L22 la perpendicular a BB' por B',
    L33 la perpendicular a CC' por C'.

    L12 la simétrica de L11 respecto a BB',
    L13 la simétrica de L11 respecto a CC'.

    M12 la paralela a L12 por B',
    M13 la paralela a L13 por C'.

    A"=M12∩M13 y similarmente se definen B" y C".

    O1 el circuncentro del triángulo A"B'C',
    O2 el circuncentro del triángulo B"C'A',
    O3 el circuncentro del triángulo C"A'B'.

    ( Mostrar/Ocultar figura )

    Entonces, los puntos I, O1, O2 y O3 son cocíclicos y el centro X de la circunferencia donde están tiene coordenadas baricéntricas:
    ( a(b+c)(a^5 - 2a^3(b^2+c^2)- a^2b c(b+c) + a(b^4-b^2c^2+c^4) + b(b-c)^2c(b+c) ) : ... : ... ).
    (con (6-9-13)-número de búsqueda en ETC: 0.4282485572315154394723079182). Incluido en la "Encyclopedia of Triangle Centers- ETC" con el número X5496="4th Hatzipolakis-Montesdeoca point".

    Además, las circunferencias circunscritas a los triángulos A"B'C', B"C'A' y C"A'B' son concurrentes en el punto Y:
    ( a(a^6-a^5(b+c) - a^4(b+c)^2+ a^3(2b^3+b^2c+b c^2+2c^3) - a^2(b^4-b^3c-3b^2c^2-b c^3+c^4) - a(b^5+b^4c+b c^4+c^5) + (b^2-c^2)^2(b^2+c^2) ) : ... : ... )
    (con (6-9-13)-número de búsqueda en ETC: 1.1800557516066828130300968705). Incluido en la "Encyclopedia of Triangle Centers- ETC" con el número X5497="5th Hatzipolakis-Montesdeoca point".

  • martes, 28 de mayo del 2013

    Circuncentros cocíclicos

    ( Anopolis #321 Antreas P. Hatzipolakis)

    Sea ABC un triángulo y A'B'C' el triángulo ceviano del incentro I. Se denota por
    L1 la perpendicular a AA' por A',
    L2 la perpendicular a BB' por B',
    L3 la perpendicular a CC' por C',
    (i.e. las rectas determinan el triángulo antipedal de I con respecto a A'B'C'),

    L11 la simétrica de L1 respecto a AA' (coincide con L1),
    L12 la simétrica de L1 respecto a BB',
    L13 la simétrica de L1 respecto a CC'.

    L21 la simétrica de L2 respecto a AA',
    L22 la simétrica de L2 respecto a BB' (coincide con L2),
    L23 la simétrica de L2 respecto a CC'.

    L31 la simétrica de L3 respecto a AA',
    L32 la simétrica de L3 respecto a BB',
    L33 la simétrica de L3 respecto a CC' (coincide con L3).

    O1 el circuncentro del triángulo AaAbAc acotado por las rectas L11, L12 y L13,
    O2 el circuncentro del triángulo BaBbBc acotado por las rectas L21, L22 y L23,
    O3 el circuncentro del triángulo CaCbCc acotado por las rectas L31, L32 y L33.

    ( Mostrar/Ocultar figura )

    Entonces, O1, O2, O3 y O (circuncentro de ABC) son cocíclicos en el punto X de coordenadas baricéntricas:
    ( a^2 ( a^7 (b + c) - a^6 (b^2 + c^2) - a^5(3 b^3 + 2 b^2 c + 2 b c^2 + 3 c^3) + a^4 (3 b^4 - b^3 c + 4 b^2 c^2 - b c^3 + 3 c^4) + a^3 (3 b^5 + b^4c + 2 b^3c^2 + 2 b^2 c^3 + b c^4 + 3 c^5) - a^2 (3 b^6 - 2 b^5 c - 2 b c^5 + 3 c^6) - a (b^7 - b^4 c^3 - b^3 c^4 + c^7) + (b^2 - c^2)^2(b^4 - b^3 c - b^2 c^2 - b c^3 + c^4)) : ... : ... ).
    (con (6-9-13)-número de búsqueda en ETC: 0.02511187325738577837)

    NOTA: Este punto ha sido incluido (28/05/2013) en la "Encyclopedia of Triangle Centers- ETC" con el número X5495="3rd Hatzipolakis-Montesdeoca point".

    El centro radical de las circunferencias circunscritas a los triángulos AaAbAc, BaBbBc y CaCbCc es X500, ortocentro del triángulo A'B'C'. (Randy Hutson)



  • domingo, 26 de mayo del 2013

    Concurrencia de rectas paralelas a rectas de Euler

    ( Anopolis #308 )

    Sea ABC un triángulo y A'B'C' el triángulo ceviano del incentro. Se denota por
    Ab, Ac las reflexiones de A' en BB' y CC', resp.
    Bc, Ba las reflexiones de B' en CC', AA', resp.
    Ca, Cb las reflexiones de C' en AA', BB', resp.
    La, Lb y Lc las rectas de Euler de los triángulos A'AbAc, B'BcBa, C'CaCb, resp.
    L'a, L'b, L'c las rectas paralelas a La, Lb, Lc por A, B y C, resp.

    Entonces, las rectas L'a, L'b, L'c concurren en X80 (simétrico del incentro respecto al punto de Feuerbach).

    ( Mostrar/Ocultar figura )


  • sábado, 25 de mayo del 2013

    Concurrencia de rectas simétricas de rectas de Euler

    ( Anopolis #301 )

    Sea ABC un triángulo y A'B'C' el triángulo ceviano del incentro. Se denota por
    Ab, Ac las reflexiones de A' en BB' y CC', resp.
    Bc, Ba las reflexiones de B' en CC', AA', resp.
    Ca, Cb las reflexiones de C' en AA', BB', resp.
    L, La, Lb y Lc las rectas de Euler de los triángulos ABC, AAbAc, BBcBa, CCaCb, resp.
    L'a, L'b, L'c las rectas simétricas de La, Lb, Lc respecto a AA', BB', CC', resp.

    Entonces, las rectas L,La, Lb, Lc son paralelas y las rectas L'a, L'b, L'c concurren en el punto de coordenadas baricéntricas:

    ( a(a^9 - a^8(b+c) - a^7(b-c)^2 + a^6(2b^3-b^2c-b*c^2+2c^3) - a^5(3b^4+b^3c-7b^2c^2+b*c^3+3c^4) + 4a^4b*c(b-c)^2(b+c) + a^3(b^2-c^2)^2(5b^2-4b*c+5c^2) - a^2(b-c)^2(2b^5+5b^4c+b^3c^2+b^2c^3+5b*c^4+2c^5) - a(b^2-c^2)^2(2b^4-3b^3c+5b^2c^2-3b*c^3+2c^4) + (b-c)^4(b+c)^3(b^2+c^2)) : ... : ... ).
    ( Mostrar/Ocultar figura )

    Este punto ha sido incluido (28/05/2013) en la "Encyclopedia of Triangle Centers- ETC" con el número X5494="2nd Hatzipolakis-Montesdeoca point".



  • viernes, 24 de mayo del 2013

    Circunferencias de Euler y triángulos ortológicos

    ( Anopolis #295 )

    Un triángulo ABC es ortológico respecto otro DEF si las perpendiculares por A, B y C a los lados de DEF se cortan en un punto. En tal caso, se verifica que, recíprocamente, DEF es ortológico respecto ABC: las perpendiculares por D, E y F a los lados de ABC también se cortan en un punto. A estos puntos de concurrencia se les llama centros de ortología.

    Sea ABC un triángulo y A'B'C', A"B"C" los triángulo órtico y medial, respectivamente.
    Se denota por:
    (Ab), (Ac) las circunferencias de los nueve puntos de A'BC", A'B"C, resp.
    (Bc), (Ba) las circunferencias de los nueve puntos de B'CA", B'C"A, resp.
    (Ca), (Cb) las circunferencias de los nueve puntos de C'AB", C'A"B, resp.
    Entonces, los triángulos ABC y DEF, el acotado por las recta (que pasan por los centros de las circunferencias) BcCb, CaAc, AbBa, son ortológicos.

    ( Mostrar/Ocultar figura )

    El centro de ortología de ABC y DEF es X1994. (Randy Hutson)

    El centro de ortología Y de DEF y ABC tiene coordenadas baricéntricas:

    ( 5a^8(b^2+c^2) - a^6(13b^4+10b^2c^2+13c^4) + a^4(9b^6-8b^4c^2-8b^2c^4+9c^6)+ a^2(b^2-c^2)^2(b^4+9b^2c^2+c^4) - 2(b^2-c^2)^4(b^2+c^2) : ... : ... ).


  • jueves, 23 de mayo del 2013

    Circunferencias de Euler y ejes radicales concurrentes

    ( Anopolis #280 )

    Sea ABC un triángulo y A'B'C', A"B"C" los triángulo órtico y medial, respectivamente.
    Se denota por:
    (Ab), (Ac) las circunferencias de los nueve puntos de A'BC", A'B"C, resp.
    (Bc), (Ba) las circunferencias de los nueve puntos de B'CA", B'C"A, resp.
    (Ca), (Cb) las circunferencias de los nueve puntos de C'AB", C'A"B, resp.
    r1 el eje radical de (Bc), (Cb);   r2 el eje radical de (Ca), (Ac);   r3 el eje radical de (Ab), (Ba)

    Entonces, r1, r2, r3 son concurrentes en el punto de X de coordenadas baricéntricas:

    ( 2a^6 - 4a^2b^2c^2 - 3a^4(b^2 + c^2) + (b^2 - c^2)^2 (b^2 + c^2) : ... : ... ).

    ( Mostrar/Ocultar figura )

    Las paralelas a r1, r2, r3 por A, B, C concurren en X1994.

    Las paralelas a r1, r2, r3 por A", B", C" concurren en el punto de Y de coordenadas baricéntricas:

    Y = ( a^4(b^2+c^2) - 2a^2(b^4 + b^2c^2 + c^4)+ (b^2 - c^2)^2 (b^2 + c^2) : ... : ...),

    Este punto es el complemento de X1994 y, además, el punto X obtenido antes es el punto medio del segmento YX1994.

  • miércoles, 22 de mayo del 2013

    Centro radical sobre la recta de Euler

    ( Anopolis #279 )

    Sea ABC un triángulo y A'B'C', A"B"C" los triángulo órtico y medial, respectivamente.
    Se denota por:
    (I11) la circunferencia exinscrita a A'B"C" respecto al ángulo B"A'C"
    (I22) la circunferencia exinscrita a A"B'C" respecto al ángulo C"B'A"
    (I33) la circunferencia exinscrita a A"B"C' respecto al ángulo A"C'B".

    ( Mostrar/Ocultar figura )

    Entonces, el centro radical de las circunferencia (I11), (I22), (I331) es X442, que está sobre la recta de Euler.

    Además, los triángulos ABC y I11I22I33 son perspectivos, con centro de perspectividad en X80 (simétrico del incentro respecto al punto de Feuerbach).
    Los puntos de contacto D, E y F de las circunferencia (I11), (I22) y (I33)con las rectas B"C", C"A" y A"B", resp., coinciden con las puntos de contacto de la circunferencia inscrita al triángulo medial con sus lados.

  • sábado, 11 de mayo del 2013

    Triángulos simétricos del triángulo preceviano de un punto, perspectivos con el triángulo de referencia

    (Advanced Plane Geometry #33 Paul Yiu)

    Dados un triángulo ABC y un punto P. sea A'B'C' el triángulo preceviano (anticeviano) de P, entonces el lugar geométrico de los puntos Q tales que ABC es perspectivo con el triángulo A"B"C", simétrico de A'B'C' respecto a Q, es la cónica biceviana C(G,P') del baricentro G y el tripolo P' de la recta pasando por P y por el cociente ceviano G/P, de G y P.

    ( Mostrar/Ocultar figura )

    Si P(p:q:r) y Q(x:y:z), en coordenadas baricéntricas, los vértices del triángulo anticeviano de P y los de su triángulo simétrico respecto a Q son:

    {A',B', C'} = {(-p : q : r), (p : -q : r), (p : q : -r)},
    {A", B", C"} = {(2(-p+q+r)x+p(x+y+z) : 2(-p+q+r)y-q(x+y+z) : 2(-p+q+ r)z-r(x+y+z)),   (2(p-q+r)x-p(x+y+z) : 2(p-q+r)y+q(x+y+z) : 2(p-q+r)z-r(x+y+z)),   (2(p+q-r)x-p(x+y+z) : 2(p+q-r)y-q(x+y+z) : 2(p+q-r)z+r(x+y+z))}.

    Las rectas AA", BB" y CC" son concurrentes si Q está en la cónica:

    qr(q-r)x^2 + rp(r-p)y^2 + pq(p-q)z^2 - p(p-q-r)(q-r)yz - q(q-r+p)(r-p)zx - r(r-p-q)(p-q)xy=0.

    Que se puede poner en la forma:
    \newfont{\euii}{eufm10 scaled \magstep4} 
{\hbox{\euii S}}_{ 
\begin{array}
 ppqr\\[-3pt]
 xyz
\end{array}}
\left[\frac{x^2}{\displaystyle\frac{1}{qr(q-r)}}
-\left(\frac{1}{\displaystyle\frac{1}{pq(p-q)}}+
\frac{1}{\displaystyle\frac{1}{rp(r-p)}}
\right)yz
\right]=0

    De donde se observa (Introduction to the Geometry of the Triangle §10.1.2) que es la ecuación de la cónica biceviana C(G,P') de los puntos G(1:1:1) y P'( 1/(qr(q-r)) : 1/((rp(r-p)) : 1/(pq(p-q)) ). Este último es el tripolo de recta que pasa por G y el cociente ceviano G/P( p(q+r-p) : q(r+p-q) : r(p+q-r) ); y también P' es el cuarto punto de intersección de las cónicas circunscritas a ABC de perspectores P y G/P (o equivalentemente —Introduction to the Geometry of the Triangle §9.3—, de las cónicas circunscritas de centro P y G/P).

    El lugar geométrico de los centros de perspectividad Q' de ABC y A"B"C", cuando Q varía en la cónica biceviana C(G,P'), es la cónica circunscrita que pasa por P y G/P.



  • viernes, 10 de mayo del 2013

    Triángulos inscritos en la circunferencia circunscrita a un triángulo, con circunferencias inscritas de mismo radio

    (Advanced Plane Geometry #29 Nikos Dergiades)

    Dado un triángulo ABC, se consideran los puntos A', B' y C', donde las paralelas por A, B y C a sus correspondientes lados opuestos, vuelven a cortar a la circunferencia circunscrita a ABC. Entonces, cada par de triángulos (AA'B, AA'C), (BB'C, BB'A) y (CC'A, CC'B) tienen circunferencias inscritas de mismo radio.

    ( Mostrar/Ocultar figura )

    Se tienen las siguientes coordenadas baricéntricas: A'(a^2: c^2-b^2 : b^2-c^2), B'(c^2-a^2 : b^2 : a^2-c^2) y C'(b^2-c^2 : a^2-b^2 : c^2).
    Los incentros de los triángulos AA'B y AA'C son, respectivamente: (a^2 : c(c-b)+a|b-c| : c(b-c)), y (a^2 : b(c-b) : b(b-c)+a|b-c|); éstos determinan las recta de ecuación |b-c|x-ay-az=0.
    El valor común del radio de las circunferencias inscritas a AA'B y AA'C es: bc(b^2-c^2)^2 / (2R(ab(b^2-c^2) + (b^2+c(a-c))|b^2-c^2|), con R el radio de la circunferencia circunscrita a ABC.

    Similarmente, los triángulos de las rectas que unen los incentros de los pares de triángulos restantes son: bx-|c-a|y+bz=0, cx+cy-|a-b|z=0.

    Estas tres rectas determinan un triángulo A1B1C1 homotético a ABC, con centro de homotecia:

    ( a(c+|a-b|)(b+|a-c|) : b(a+|b-c|)(c+|b-a|) : c(b+|c-a|)(a+|c-a|) ),

    y razón de homotecia: -(|a-b|(ab-|a-c||b-c|)+c(a|a-c| + b(2a+|b-c|)) / ((c+|a-b|)(b+|a-c|)(a+|b-c|)).

  • jueves, 09 de mayo del 2013

    Circunferencia lugar del incentro de un triángulo variable

    (Anopolis #229)

    Sean ABC un triángulo acutángulo y δ una recta que pasa por el ortocentro H. Denotemos por δa, δb y δc las reflexiones de δ en AH, BH, CH, resp.; δ'a, δ'b y δ'c las paralelas a δa, δb y δc a través de A, B, C, resp., y I' el incentro del triángulo A'B'C' acotado por las rectas δ'a, δ'b y δ'c.

    El lugar geométrico de I' cuando δ gira alrededor de H es la circunferencia de centro H y que pasa por X265, su radio es el de la circunferencia circunscrita a ABC.

    ( Mostrar/Ocultar figura )

    Ocurre además que los vértices de A'B'C' describen las circunferencias (B,C, X265), (C, A, X265) y (A, B, X265).



  • miércoles, 08 de mayo del 2013

    Recta de Euler e hipérbola de Jerabek

    (Anopolis #227)

    Sean ABC un triángulo, H su ortocentro y (u:v:w) las coordenadas baricéntricas de un punto P. Se denota por Ha, Hb, Hc los ortocentros de los triángulos PBC, PCA, PAB, resp.
    Las reflexiones de las rectas HHa en los lados de ABC concurren en P1:

    ( a²vw(SBv+SC(u+v))(SCw+SB(u+w)) : -b²uw(SBv+SC(u+v))(SBv-SCw) : c²uv(SBv-SCw)(SCw+SB(u+w)) ),
    situado sobre la circunferencia circunscrita a ABC.

    Las reflexiones de las rectas HHa en los lados de PBC en P'1:

    ( a²SBSCuvw(u+v+w) : SCw(SC²(u+v)² w+SB²v²(u+w)+SAu²(-SBv+SCw)-SBSCv( u²+uv-2vw)) : SBv(SC²(u+v) w²+SB²v(u+w)²+SAu²(SBv-SCw)- SBSCw(u²+uw-2vw)) ),
    situado sobre la circunferencia circunscrita a PBC. Donde se ha utilizado la notación de Conway.

    Siendo la ecuación de la recta P1P'1:
    u(a²SA+2SBSC)(SBv-SCw)x + a²vSB(SCw+SB(u+w))y - a²wSC(SBv+SC(u+v))z=0.

    Procediendo de forma similar, obtenemos:
    Las reflexiones de la recta HHb en los lados de ABC concurren en P2 y en los lados de PCA en P'2.
    Las reflexiones de la recta HHc en los lados de ABC concurren en P3 y en los lados de PAB en P'3.

    El lugar geométrico de los puntos P tales que los triángulos P1P2P3 y P'1P'2P'3 son perspectivos (junto con la recta del infinito) es la hipérbola de Jerabek (conjugada isogonal de la recta de Euler):

    a²(c²-b²)(b²+c²-a²)yz+ b²(a²-c²)(c²+a²-b²)zx + c²(b²-a²)(a²+b²-c²)xy = 0.

    • Cuando P se mueve en la hipérbola de Jerabek las rectas P1P'1, P2P'2 y P3P'3 concurren en Q, sobre la recta de Euler.

    ( Mostrar/Ocultar figura )

    • También, cuando P se mueve en la hipérbola de Jerabek las rectas paralelas a P1P'1, P2P'2 y P3P'3 a través de los vértices del triángulo de contacto interior, son concurrentes en el punto medio M de H y Q.



  • miércoles, 08 de mayo del 2013

    Un centro del triángulo de E. M. H. Lemoine

    (Advanced Plane Geometry #2 Paul Yiu)

      Dado un punto P en el plano del triángulo ABC, se traza la paralela por P al lado BC que interseca a AB en Ca y a AC en Ba. Similarmente, se construyen Ab, Cb sobre BC, BA, y Bc, Ac sobre CA, CB, respectivamente.

      (1) El punto P para el que BcCb, CaAc, y AbBa son todas tangentes a la circunferencia inscrita es el anticomplemento del punto de Nagel, X145 en ETC

    (E. M. H. Lemoine.- Étude sur de nouveaux points remarquables du plan d’un triangle, Journal de Math. Spéciales, ser. 2, 2 (1883) 3--6).

      Si P(x:y:z), coordenadas baricéntricas, entonces Ca(x : y+z : 0), Ba(x : 0 : y-z), Ab(0 : y : x+z), Cb(x+z : y : 0), Bc(x+y : 0 : z) y Ac(0 : x+y : z).
      Para que la recta BcCb sea tangente al circunferencia inscrita se ha de verificar que debe contener a su polo respecto a ella; por lo que el punto P ha de estar en las cónicas (la misma situación para las otras dos rectas CaAc, y AbBa):
    (a-b-c)x² + (a-b+c)y² + (a+b-c)z² + (a-b-c)yz + 2(a-c)zx + 2(a-b)xy=0,
    (b-c-a)y² + (b-c+a)z² + (b+c-a)x² + (b-c-a)zx + 2(b-a)xy + 2(b-c)yz=0,
    (c-a-b)z² + (c-a+b)x² + (c+a-b)y² + (c-a-b)xy + 2(c-b)yz + 2(c-a)zx=0.

      El punto común a estas tres cónicas es X145, simétrico del punto de Nagel respecto al incentro.
    ( Mostrar/Ocultar figura )

    (2) Los puntos de tangencia determinan un triángulo XYZ perspectivo con ABC, con centro de perspectividad el punto de coordenadas baricéntricas:

    ( (3a-b-c)² / (b+c-a) : (3b-c-a)² / (c+a-b) : (3c-a-b)² / (a+b-c) ),

    (con (6-9-13)-número de búsqueda en ETC: 0.02670313601037967069540658222).

      Este punto es conjugado isogonal del punto de concurrencia descrito en el problema O126:
      "O126. Let ABC be a triangle and let Ka be the A-mixtilinear incircle (the circle tangent to sides (AB), (AC) and internally tangent to the circumcircle Γ of triangle ABC). Denote by A' the tangency point of Ka with Γ and let A" be the diametrically opposed point of A' with respect to Ka. Similarly, define B" and C". Prove that lines (AA"), (BB") and (CC") are concurrent".
      Proposed by Cosmin Pohoata, NationalCollege ”Tudor Vianu”, Romania
    ( Mostrar/Ocultar figura )


      César Lozada da las coordenadas del punto de intersección de las rectas (AA"), (BB") y (CC") en ADGEOM #1098.

    ( a²(b+c-a)/(3a-b-c)² : b²(c+a-b)/(3b-c-a)² : c²(a+b-c)/(3c-a-b)² ).

    (3) El triángulo XYZ es perspectivo con el triángulo de contacto interior DEF y el centro de perspectividad también es el X145.

    ( Mostrar/Ocultar figura )

    (4) Los seis puntos Ab, Ac, Bc, Ba, Ca y Cb están en una misma cónica:

    SumaCíclica[ 2(a+b-3c)(a-b-c)(a-3b+c) x^2 + (3a-b-c)(5a^2-2a(b+c)-7b^2+18bc-7c^2) yz]=0.
      De centro:
    Q = ( 39a3 - 43a2(b+c) + a(37b2-22bc+37c2) - 9b3+5b2c+5c2b-9c3 : ... : ... ),
    (con (6-9-13)-número de búsqueda en ETC: 0.512214699835464592430071714) y perspector (centro de perspectividad del triángulo ABC y el triángulo formado por las polares de sus vértices respecto a la cónica):


  • martes, 07 de mayo del 2013

    Reflexión de centros de circunferencias de los nueve puntos

    (Anopolis #224)

    Sean ABC un triángulo y P un punto, se denota por P11, P22, P33 los centros de las circunferencias de los nueve puntos de los triángulos PBC, PCA, PAB, resp.
    Consideremos los puntos:
    P12, P13 las reflexiones de P11 in PB, PC, resp.
    P23, P21 las reflexiones de P22 in PC, PA, resp.
    P31, P32 las reflexiones de P33 in PA, PB, resp.


    Los triángulos P11P12P13, P21P22P23, P31P32P33 son concéntricos, con circuncentro P.

    Sean Na, Nb, Nc los centros de las circunferencias de los nueve puntos de P11P12P13, P21P22P23, P31P32P33 , resp.

    El lugar geométrico de los puntos P tales que los triángulos P11P22P33 y NaNbNc son perspectivos (junto con la recta del infinito y la circunferencia circunscrita) es la cúbica de McCay, K003 en el catálogo de Bernard Gibert.

    Si P(u:v:w) son las coordenadas baricéntricas de un punto sobre la cúbica de McCay o en la circunferencia circunscrita, el centro de perspectividad Q de los triángulos P11P22P33 y NaNbNc es:

    a^6(b^2w(3v*w(v-w) + u^2(v+w) + u(v^2+2v*w-2w^2)) + c^2v(3v*w(w-v) + u^2(v+w) + u(w^2+2v*w-2v^2))) +
    a^4(c^4v(u^3 + 2v*w(v-2w) - u^2(v+w) + u(v^2+v*w-2w^2)) + b^4w(u^3 + 2v*w(w-2v) - u^2(v+w) + u(w^2+v*w-2v^2)) + b^2c^2(u^3(v+w)+2u(v-w)^2(v+w)-6u^2(v^2+w^2)-2v*w(v^2-4v*w+w^2))) -
    a^2(-c^6v(-2u^3+ u^2(v-w) + u*w(w-4v) + v*w(v+w)) - b^6w(-2u^3+ u^2(w-v) + u*v(v-4w) + v*w(v+w)) + b^4c^2(2u^3(2v+w) - u^2(5v^2+v*w-6w^2) + v*w(-v^2+v*w+2w^2) + u*w(10v^2-v*w+4w^2)) + b^2*c^4(2u^3(v+2w) + u^2(6v^2-v*w-5w^2) + v*w(2v^2+ v*w-w^2) + u*v(4v^2-v*w+10w^2))) +
    (b^2-c^2)^2u(c^4v(u^2 + u(w-v) + v(v+w)) + b^4w(u^2+u(v-w)+w(v+w)) + b^2c^2(-4u*v*w+3u^2(v+w)+2v*w(v+w)) ) : ... : ...

    Nota: Las coordenadas simétricas de Q han sido obtenidas usando el método descrito por Barry Wolk en el mensaje #19239 of Hyacinthos.

    Si P está sobre la circunferencia circunscrita a ABC, el centro de perspectividad Q de los triángulos P11P22P33 y NaNbNc es el punto del infinito de la recta OP (O circuncentro de ABC).
    ( Mostrar/Ocultar figura )


    Para puntos de la cúbica de McCay: ( P=I=X(1), Q=X(10) ); ( P=O=X(3), Q=X(3) ); ( P=H=X(4), Q=X(5) ).
    ( Mostrar/Ocultar figura )


  • lunes, 06 de mayo del 2013

    Dos nuevos centros del triángulo en la recta IO

    (Anopolis #220 Antreas P. Hatzipolakis)

    Consideremos un triángulo ABC de incentro I y denotamos por A'B'C' el triángulo ceviano de I.

    •  Sea W- el centro radical de las circunferencias A'(A'B), C'(C'A) y B'(B'C), y W+ el centro radical de las circunferencias A'(A'C), B'(B'A) y C'(C'B).
    Los puntos W+W- forman un par bicéntrico, por lo que el punto medio de W+W- es un centro del triángulo (sobre la recta IO), que tiene (6-9-13)-número de búsqueda en ETC: -4.58771580677070881713647893, y coordenadas baricéntricas:

    ( a(a4(b-c)2 + a3(b3+bc(b+c)+c3) - a2(b4-3bc(b2+c2)+c4) - a(b5+bc(b3+c3)+c5) - bc(b2-c2)2) : ... : ....)

    NOTA: Este punto ha sido incluido en la "Encyclopedia of Triangle Centers- ETC" con el número X5482="1st Hatzipolakis-Montesdeoca point".

    ( Mostrar/Ocultar figura )

    El punto del infinito de la recta W+W-, que es un centro, tiene (6-9-13)-número de búsqueda en ETC: 0.973211383408006530639583531 y coordenadas baricéntricas:
    ( a(b - c)(a4(b+c) - a3(b-c)2 - a2(b3 - 4bc(b+c) + c3) + a(b4+2bc(b2+c2)+c4) - bc(b+c)3) : ... : ...)


    Generalización:
    Si sustituimos el incentro por otro punto P(u:v:w), el punto medio de W+W- es:
    ( a4(u(v-w)2+2vw(v+w)) - a2(c2(u2(v-w)+2v2w+uw(w-3v))+b2(uv(v-3w)+2vw2+u2(w-v)))- (b2-c2)u(v+w)(b2(u+w)-c2(u+v)): ... : ... )

    Si P=H, el ortocentro, el punto medio de W+W- es X1147 (sobre la recta que une el ortocentro con el foco de la parábola de Kiepert):
    ( a4(a2-b2-c2)(a4-2a2(b2+c2)+b4+c4) : ... : ...)
    Y el punto del infinito de la recta W+W- es:
    ( a2(b2-c2)(a6 - a4(b2+c2) - a2(b4-4b2c2+c4) + b6-3b2c2(b2+c^2)+c6) : ... : ...)


    •  Otro caso de punto medio de centros radicales:   Sea W+ el centro radical de las circunferencias A'(A'B'), B'(B'C') y C'(C'A'), y W- el centro radical de las circunferencias A'(A'C'), B'(B'A') y C'(C'B').
    Los puntos W+W- forman un par bicéntrico y el punto medio de W+W-, situado sobre la recta IO, (con (6-9-13)-número de búsqueda en ETC: 2.182865285269536019160224004) es:

    ( a(3a5(b+c) + a4(b2+4bc+c2) - a3(b+c)(6b2-bc+6c2) -2a2(b4+3b3c+3b2c2+3bc3+c4)+ a(b-c)2(b+c)(3b2+5bc+3c2)+ (b-c)2(b+c)4) : ... : ...)
    ( Mostrar/Ocultar figura )


    Generalización:
    Si sustituimos el incentro por otro punto P(u:v:w), el punto medio de W+W- es:
    ( a^4vw(2v^2w^2+3u^3(v+w)+uvw(v+w)+ u^2(v^2+4vw+w^2)) - a^2u(b^2w(v^2(v-w)w+u^2(6v^2+5vw+w^2) + 2uv(v^2+3vw+2w^2)) + c^2v(vw^2(w-v) + 2uw(2v^2+3vw+w^2) + u^2(v^2+5vw+6w^2))) + (b^2-c^2)u^2(v+w)(b^2w(u(3v-w)+v(v+w))+c^2v(u(v-3w)-w(v+w))): ... : ... )

    Si P=G, el baricentro, el punto medio de W+W- es X140. Y el punto del infinito de la recta W+W- es X1499, punto de Biham.

    Si P=X7, el punto de Gergonne, el punto medio de W+W- es X942, inverso en la circunferencia inscrita del inverso del incentro en la circunferencia circunscrita. Y el punto del infinito de la recta W+W- es:
    ( a(b-c)(a^2- 2a(b+c) + b^2+4bc+c^2) : b(c-a)(b^2 - 2b(c+a) + c^2+4ca+a^2) : c(a-b)(c^2 - 2c(a+b) + a^2+4ab+b^2) ),

    con (6-9-13)-número de búsqueda en ETC: 5.739583333333333333.

  • viernes, 03 de mayo del 2013

    Triángulos homotéticos

    (Anopolis #212)

    • Consideremos un triángulo ABC de circuncentro O y denotamos por A'B'C' el triángulo ceviano de O. Sea da el eje radical de la circunferencia O(OA'), de centro O y radio OA', y de la circunferencia circunscrita a OBC; de forma similar se consideran los ejes radicales db y dc.
    El triángulo ABC y el acotado por las rectas da, db y dc son homotéticos

    El centro de homotecia tiene coordenadas baricéntricas:

    Es el cuadrado baricéntrico de X2167 y también el conjugado isogonal del cuadrado baricéntrico del centro de la circunferencia de los nueve puntos, X5.
    ( Mostrar/Ocultar figura )

    • Consideremos un triángulo ABC de circuncentro O y denotamos por A'B'C' su triángulo medial. Sea A"B"C" el triángulo circunceviano de O respecto a A'B'C', D=BC∩AA", E=CA∩BB" y F=AB∩CC". Denotamos por da el eje radical de la circunferencia A"(A"D), de centro A" y radio A"D, y de la circunferencia circunscrita a A"BC; de forma similar se consideran los ejes radicales db y dc.
    El triángulo ABC y el acotado por las rectas da, db y dc son homotéticos

    El centro de homotecia tiene coordenadas baricéntricas:

    (con (6-9-13)-número de búsqueda en ETC: 1.32064524330236458037845738)
    ( Mostrar/Ocultar figura )


  • miércoles, 1 de mayo del 2013

    Construcción de triángulo

    Problema 683. ( Laboratorio Virtual del Triángulo con Cabri)

    Construir un triángulo rectángulo con el baricentro en la circunferencia inscrita.
    Barroso, R. (2013): Comunicación personal.

    Mi aportación en HTML, o en PDF.

  • martes, 30 de abril del 2013

    Triángulos perspectivos

    Consideremos un triángulo ABC de circuncentro O y denotamos por DEF su triángulo medial. La circunferencia de centro en el punto medio Da de OD y que pasa por B y C, vuelve a cortar a las circunferencias de diámetros AC y AB en los puntos Ab y Ac respectivamente. Procediendo cíclicamente se definen los pares de puntos Bc y Ba, Ca y Cb.
    Las rectas AbAc, BcBa, CaCb determinan un triángulo A'B'C' perspectivo con ABC, cuyo centro de perspectividad es el centro del triángulo de (6,9,13)-número de búsqueda 1.87843618353880172287205618 en ETC y coordenadas baricéntricas:
    
\left(\frac{1}{S_A(10S_B^2S_C^2 + 17a^2S_AS_BS_C + 2S_A^2(3S_B^2 +  7S_BS_C + 3S_C^2))}, 
      \frac{1}{S_B(10S_A^2S_C^2 + 17b^2S_AS_BS_C + 2S_B^2(3S_A^2 +  7S_AS_C + 3S_C^2))}, 
      \frac{1}{S_C(10S_A^2S_B^2 + 17c^2S_AS_BS_C + 2S_C^2(3S_A^2 +  7S_AS_B + 3S_B^2))}\right)
    ( Mostrar/Ocultar figura )


  • lunes, 29 de abril del 2013

    Triángulos pedales y circunferencias centradas en los vértices de un triángulo

    El mensaje #159 del grupo Anopolis de Antreas Hatzipolakis sugiere el siguiente planteamiento, considerando pares de circunferencias con centros en los vértices de un lado de un triángulo:

    Consideremos un triángulo ABC y un punto U. Denotamos por UaUbUc su triángulo pedal, por O(R) su circunferencia circunscrita, por B(BUc) y C(CUb) las circunferencias centradas en B y C, de radios BUc y CUb respectivamente. Tomemos un punto P∈O(R), la recta PB corta a B(BUc) en B', B", la recta PC corta a C(PUb) en C', C". Entonces, las circunferencias (PB'C') y (PB"C") pasan por un punto fijo A1 sobre O(R), cuando el punto P varía. Así mismo, al variar P sobre O(R), las circunferencias (PB'C") y (PB"C') pasan por un punto A2, fijo sobre O(R). Similarmente se definen los puntos B1 y B2, C1 y C2, procediendo cíclicamente sobre los vértices de ABC.
    Las rectas A1A2, B1B2 y C1C2 son concurrentes si y sólo si el punto U está en O(R) o en la cúbica de Darboux (K004 en el catálogo de Bernard
    ( Mostrar/Ocultar figura )

    •   Cuando U está en la circunferencia circunscrita, el punto de intersección de las rectas A1A2, B1B2 y C1C2 es el propio punto U.

    •   Cuando U se mueve en la cúbica de Darboux el lugar geométrico (?) de los puntos U' de intersección de las rectas A1A2, B1B2 y C1C2, contiene a los puntos A, B, C, antipodales de A, B, C en la circunferencia circunscrita, X3, X6, X22, X24, X1498, X1604, X1617, ...

    Puntos U' como centros de perspectividad de los triángulos tangencial y circunceviano de un punto V:
    (Clark Kimberling.- Triangle Centers and Central Triangles, (TCCT) §7.18, p. 201)

    U, en la cúbica de Darboux U', punto de intersección de las rectas A1A2, B1B2 y C1C2 V, tal que U' es el centro perspectividad de los triángulos tangencial y circunceviano de V
    X1 X1617 X57
    X4 X24 X4
    X20 X22 X2
    X40 X3 X1
    X64 ?
    X84 ?
    X1490 X1604 X9
    X1498 X1498 X83


  • viernes, 26 de abril del 2013

    Tríada de circunferencias centradas en los vértices de un triángulo

    El mensaje #159 del grupo Anopolis de Antreas Hatzipolakis sugiere el siguiente planteamiento:

    Consideremos un triángulo ABC y denotamos por O(R) su circunferencia circunscrita, por A(ρa), B(ρb) y C(ρc) circunferencias centradas en el vértice correspondiente y radios ρa, ρb y ρc, respectivamente. Tomemos un punto P∈O(R), la recta PB corta a B(ρb) en B', B", la recta PC corta a C(ρc) en C', C". Entonces, las circunferencias (PB'C') y (PB"C") pasan por un punto fijo A1 sobre O(R), cuando el punto P varía. Así mismo, al variar P sobre O(R), las circunferencias (PB'C") y (PB"C') pasan por un punto A2, fijo sobre O(R). Similarmente se definen los puntos B1 y B2, C1 y C2, verificándose que las rectas A1A2, B1B2 y C1C2 son siempre concurrentes.
    ( Mostrar/Ocultar figura )
      Tercer punto de Brocard


    Casos particulares

    •   Para las circunferencias centradas en los vértices del triángulo ABC y mutuamente tangentes, el punto común de las rectas A1A2, B1B2 y C1C2 es el X1617, centro de perspectividad de los triángulos tangencial y circunceviano de X57.

    •   Para las circunferencias A(a), B(b) y C(c), centradas en los vértices del triángulo ABC y de radios la longitud del lado opuesto, el punto común de las rectas A1A2, B1B2 y C1C2 es el X22, centro de perspectividad de los triángulos tangencial y circunceviano de X2.

    •   Para las circunferencias A(ma), B(mb) y C(mc), centradas en los vértices del triángulo ABC y de radios la longitud de la mediana correspondiente, el punto común de las rectas A1A2, B1B2 y C1C2 es el X1995 (como en el caso de las circunferencias que pasan por el baricentro).

    •   Para las circunferencias A(ha), B(hb) y C(hc), centradas en los vértices del triángulo ABC y tangentes al lado opuesto, el punto común de las rectas A1A2, B1B2 y C1C2 es el simediano, X6.

    •   Para las circunferencias A(AOa), B(BOb) y C(COc), centradas en los vértices del triángulo ABC y que pasan por los pies de las cevianas del circuncentro, el punto común de las rectas A1A2, B1B2 y C1C2 tiene primera coordenada baricéntrica:

    a^2 (a^18 - 6a^16(b^2+c^2) + 3a^14(5b^4+8b^2c^2+5c^4) - a^12(21b^6+37b^4c^2+37b^2c^4+21c^6) + a^10(21b^8+28b^6c^2+31b^4c^4+28b^2c^6+21c^8) - 3a^8(7b^10+2b^8c^2+3b^6c^4+3b^4c^6+2b^2c^8+7c^10) + a^6(b^2-c^2)^2(21b^8+22b^6c^2+24b^4c^4+22b^2c^6+21c^8) - a^4(b^2-c^2)^2(15b^10-5b^8c^2-6b^6c^4-6b^4c^6-5b^2c^8+15c^10) + 3a^2(b^2-c^2)^4(2b^8-b^4c^4+2c^8) - (b^2-c^2)^6(b^6+c^6) ) .

    •   Para las circunferencias A(AI), B(BI) y C(CI), centradas en los vértices del triángulo ABC y que pasan todas por el incentro (I=X1), el punto común de las rectas A1A2, B1B2 y C1C2 es el X56, centro de perspectividad de los triángulos tangencial y circunceviano de X266.

    •   Para las circunferencias A(AG), B(BG) y C(CG), centradas en los vértices del triángulo ABC y que pasan todas por el baricentro (G=X2), el punto común de las rectas A1A2, B1B2 y C1C2 es el X1995 (como en el caso de radio las medianas), centro de perspectividad de los triángulos tangencial y circunceviano de la raíz baricéntrica de X1383.

    •   Para las circunferencias A(AO), B(BO) y C(CO), centradas en los vértices del triángulo ABC y que pasan todas por el circuncentro (O=X3), el punto común de las rectas A1A2, B1B2 y C1C2 (mediatrices de ABC) es el X3, centro de perspectividad de los triángulos tangencial y circunceviano de X1.

    •   Para las circunferencias A(AH), B(BH) y C(CH), centradas en los vértices del triángulo ABC y que pasan todas por el ortocentro (H=X4), el punto común de las rectas A1A2, B1B2 y C1C2 es el X24, centro de perspectividad de los triángulos tangencial y circunceviano de X4.

    •   Para las circunferencias A(AK), B(BK) y C(CK), centradas en los vértices del triángulo ABC y que pasan todas por el simediano (K=X6), el punto común de las rectas A1A2, B1B2 y C1C2 es el X1384, centro de perspectividad de los triángulos tangencial y circunceviano del punto con coordenadas baricéntricas:

    ( a²/(a²-2(b²+c²))½ : b²/(b²-2(c²+a²))½ : c²/(c²-2(a²+b²))½ ) .


  • martes, 23 de abril del 2013

    Anopolis #147

    En el grupo Anopolis (mensaje #147) de Antreas Hatzipolakis se propone lo siguiente:

    Sea ABC un triángulo, P un punto, se designa por Ha, Hb, Hc los ortocentros de los triángulos PBC, PCA, PAB, resp. y por O1, O2, O3 los circuncentros de los triángulos HaBC, HbCA, HbAB, resp.
    ¿Cuál es el lugar geométrico de P para que los triángulos HaHbHc y O1O2O3 sean perspectivos?

    Mi respuesta (mensaje #150):

    El lugar geométrico de P tal que las rectas HaO1, HbO2 y HcO3 sean concurrentes consta de la circunferencia circunscrita a ABC y de la cúbica de McCay (K003 en el catálogo de Bernard Gibert).

    •  Si P está sobre la circunferencia circunscrita, las rectas HaO1, HbO2 y HcO3 son paralelas y su punto del infinito es SR(P,Q), donde Q es punto diametralmente opuesto a P.
    Una descripción del punto Simson-Rigby SR(R,U), con P y U puntos distintos sobre la circunferencia circunscrita al triángulo ABC, figura justo antes del punto X2677 en ETC.
    Si en coordenadas baricéntricas P = (p : q : r) y U = (u : v : w), entonces:
    SR(P,U)= ( (rv - qw)/(b²rw-c²qv) : (pw - ru)/(c²pu-a²rw) : (qu - pv)/(a²qv-b²pu) ).

    Si P se mueve en la circunferencia circunscrita, los puntos O1, O2, O3 son los simétrico de A, B, C, respecto al centro (X5) de la circunferencia NPC de los nueve puntos; y los triángulos HaHbHc son simétrico de ABC respecto al punto medio (sobre NPC) de P y el ortocentro.


    •  Si P queda sobre la cúbica de McCay, el punto de intersección de las rectas HaO1, HbO2 y HcO3 está sobre la tercera cúbica de Musselman (K028 en el catálogo de Bernard Gibert).


  • lunes, 22 de abril del 2013

    Puntos en cónicas inscritas

    Dado un triángulo ABC y un punto P, vamos a asociar a cada recta d que pasa por P un punto sobre la cónica inscrita en ABC de perspector P (conjugado isotómico de P). Además, daremos una propiedad geométrica relativa a dicho punto.
    ( Mostrar/Ocultar figura )
      Tercer punto de Brocard

    Si (u:v:w) son las coordenadas baricéntricas de P, respecto a ABC, la ecuación de una recta d variable por P la podemos expresar, en función de dos parámetros m y n, como sigue:
    (v - w) (m + v w n)x+ (w-u ) (m + wu n)y+ (u - v) (m + u v n)z=0,
    La cónica circunscrita d, conjugada isotómica de d, tiene por ecuación:
    (m(v - w) + n vw(v - w)) yz + (m(w - u) + n wu(w - u)) zx + (m(u - v) + n uv(u - v)) xy =0.
    Su perspector está en la tripolar de P (recta G-Dual de P).

    Sean Q el cuarto punto de intersección de d y la cónica circunscrita C(P) de perspector P, C(P,Q) la cónica que pasa por los pies Pa, Pb y Pc de las cevianas de P y Qa, Qb y Qc de Q (cónica biceviana de P y Q) y ℑ(P) la cónica inscrita de perspector P.

    El cuarto punto t(P,d) de intersección de ℑ(P) y C(P,Q) es el tripolo de la tangente a d en P (recta que une los perspectores de ℑ(P) y C(P,Q) respecto al triángulo ceviano de P). Las coordenadas de t(P,d) son:

    t(P,d) = ( u(v - w)²(m + n vw)² : v(w - u)²(m + n wu)² : w(u - v)²(m + n uv)² ).

    También: t(P,d) es el punto de tangencia con ℑ(P) de la tripolar del perspector de la cónica d (conjugada isotómica de la recta d).

    En particular, si P=(u:v:w), a la recta d=PP le corresponde el punto:
    ( u3(v2-w2)2 : v3(u2-w2)2 : w3(u2-v2)2 ).
    Se trata del punto de tangencia de la tripolar del punto de intersección de las tripolares de P y P con la cónica inscrita de perspector P .

    Una propiedad del punto t(P,d):
    Denotamos por Ca la cónica tangente en Pa a BC y que pasa por Qb, Qc y Q. Las cónicas Cb y Cc se definen de forma similar.
    Las tres cónicas Ca, Cb y Cc son tangentes en t(P,d) a la tripolar del perspector de la cónica d.


    Algunos casos particulares de obtención de puntos t(P,d):

    • P=X7 (punto de Gergonne), ℑ(X8) es la elipse inscrita de Mandart, de perspector el punto de Nagel.

      En el mensaje #14278 de Hyacinthos, Steve Sigur se extrañaba que en esta elipse hubiera sólo un punto de entre los 3000 primeros de ETC. Continuó comentando (mensaje #14291) que tal elipse podría no ser interesante ya que incluso Bernard Gibert en su excelente artículo en Forum Geometricurum, Volume 4 (2004) 177–198.(Generalized Mandart Conics) habla poco de ella. Por otra parte, Floor Van Lamoen menciona la siguiente propiedad (mensaje #14263): "La elipse de Mandart es cofocal con la elipse circunscrita de perspector el incentro. Todos los triángulos porísticos relativos a estas dos cónicas tienen el mismo perímetro, que es el máximo que un triángulo inscrito a la citada elipse circunscrita puede tener."

      Algunos puntos t(X7,d) en la elipse inscrita de Mandart, correspondientes a rectas por X7:
      • A la recta X1X7 (m=0) le corresponde el punto X4081.
      • A la recta X2X7 (n=0) le corresponde el punto X11, punto de Feuerbach.
      • A la recta X7X8 ( m=a+b+c, n=-(b+c-a)(a² - b² - c² + 2bc) ) le corresponde el punto X3271.
      • A la recta X7X21 ( m=a+b+c, n=(b+c-a)(a² - b² - c² + 2bc) ) le corresponde el punto X4092.
      • A la recta d que pasa por X7 de parámetros (m=área(ABC)=S/2, n=1) le corresponde el punto de coordenadas baricéntricas:

        ( (b-c)²(b+c-a)(15a-17(b+c))² : (c-a)²(c+a-b)(15b-17(c+a))² : (a-b)²(a+b-c)(15c-17(a+b))² ).



    • P=X8 (punto de Nagel), ℑ(X7) es la circunferencia inscrita (de perspector el punto de Gergonne).

      Algunos puntos t(X8,d) en la circunferencia inscrita, correspondientes a rectas por X8:
      • A la recta X1X8 (n=0) le corresponde el punto X11, punto de Feuerbach.
      • A la recta X3X8 ( m = a3 + b3 + c3 - a2b - ab2 - a2c - b2c - ac2 - bc2 + 6abc, n = -(a+b+c) ) le corresponde el punto X3326.
      • A la recta X4X8 ( m = 2(a3 - ab2 - ac2) - (a+b+c)(a2 - b2- c2 + 2bc ), n = a+b+c ) le corresponde el punto X1364.
      • A la recta d5=X5X8 le corresponde el punto:
        t(X8,d5) = ( (b+c-a)(b-c)2(2a3-a2(b+c)-2a(b2-bc+c2)+(b-c)2(b+c))2 : ... : ... ).

      • A la recta d6=X6X8 (m = (a+b+c)2, n=-1) le corresponde el punto de coordenadas baricéntricas:

        t(X8,d6) = ( (b+c-a)(b-c)²(b²+c²+a(b+c))² : (c+a-b)(c-a)²(c²+a²+b(c+a))² : (a+b-c)(a-b)²(a²+b²+c(a+b))² ).

      • A la recta X7X8 (m = a² + b² + c² - 2ab - 2ac - 2bc, n=1) le corresponde el punto X3022.
      • A la recta X8 X37 (m=(a+b+c)², n=0) le corresponde el punto X3026.
      • A la recta X8 X522 ( m = a5 - 2a4(b + c) + a3(b2 + 3bc + c2) - a(b - c)2(2b2 + bc + 2c2) + (b - c)2 (b3 + c3) + a2(b3 - 2b2c - 2bc2 + c3), n = a3 - a2(b + c) + (b - c)2(b + c) - a(b2 - 3bc + c2) ) le corresponde el punto X1317.
      • A la recta d que pasa por X8 de parámetros (m=r², n=1) le corresponde el punto de coordenadas baricéntricas:

        ( (b-c)((b-c)²-a²)(3a+5(b+c)) : (c-a)((c-a)²-b²)(3b+5(c+a)) : (a-b)((a-b)²-c²)(3c+5(a+b)) ).

      • A la recta d que pasa por X8 de parámetros (m=área(ABC)=S/2, n=1) le corresponde el punto de coordenadas baricéntricas:

        ( (b-c)²(34bc-30SA+8S) : (c-a)²(34ca-30SB+8S) : (a-b)²(34ab-30SC+8S) ).



    • P=X69, ℑ(X4) es la "orthic inconic" de perspector el ortocentro.

      Algunos puntos t(X69,d) en la "orthic inconic", correspondientes a rectas por X69:
      • A la recta d1 =X1X69 ( m = a4+2a3(b+c)+2a2(b2+3bc+c2)+2a(b+c)3+(b+c)2(b2+c2) , n = 4 ) le corresponde el punto de coordenadas baricéntricas:

        t(X69,d1) = ( (b-c)²(b²+c²-a²)(a²+(b+c)²)² : (c-a)²(c²+a²-b²)(b²+(c+a)²)² : (a-b)²(a²+b²-c²)(c²+(a+b)²)² ).

      • A la recta X2X69 (n=0) le corresponde el punto X125, centro de la hipérbola de Jerabek.
      • A la recta d3=X3X69 (m = 0) le corresponde el punto de coordenadas baricéntricas:

        t(X69,d3) = ( (b²-c²)² ⁄ (b²+c²-a²) : (c²-a²)² ⁄ (c²+a²-b²) : (a²-b²)² ⁄ (a²+b²-c²) ).

      • A la recta d4=X4X69 (m = S², n=-1) le corresponde el punto de coordenadas baricéntricas:

        t(X69,d4) = ( a4(b2-c2)2(b2+c2-a2)3 : b4(c2-a2)2(c2+a2-b2)3 : c4(a2-b2)2(a2+b2-a2)3 ).

      • A la recta d5=X5X69 (m = 2S², n=-1) le corresponde el punto de coordenadas baricéntricas:

        t(X69,d5) = ( SA(SB-SC)² (2S² -SBSC)² : SB(SC-SA)² (2S² -SCSA)² : SC(SA-SB)² (2S² -SASB)² ).

      • A la recta X7X69 (m = a^4-2abc(b+c)+(b²-c²)²-2a²(b²+bc+c²), n=4) le corresponde el punto X3270.
      • A la recta X63X69 (m = (b²+c²-a²)(a²+c²-b^2)(a²+b²-c²), n = -4(a+b+c)² ) le corresponde el punto X2969.


  • domingo, 21 de abril del 2013

    Cónicas conjugadas isotómicas de rectas que pasan por un punto

    Dados un triángulo ABC, un punto P y una recta d variable que gira alrededor de P, sea d la cónica circunscrita conjugada isotómica de d.
    Las rectas polares p de P respecto a las cónicas d, cuando d varía, pasan por un punto fijo Q.

    Los puntos de intersección de las restas d y p describen la hipérbola que pasa por G (baricentro), P y Q y las tangente en estos últimos se cortan en el conjugado isotómico P de P.

    Si P=(u:v:w), coordenadas baricéntricas respecto a ABC, las coordenadas de Q son:
    ( u(v²+w²-u²) : v(w²+u²-v²) : w(u²+v²-w²) ).

    Algunos casos de centros del triángulo:
    P X1 X3 X6 X9 X10 X30 X37
    Q X63 X1993 X22 X3870 X3995 X2 X4651

    De hecho, las polares de un punto del infinito, respecto a las cónica conjugadas isotómicas de rectas que pasan por él (paralelas), pasan por el baricentro; como es el caso de rectas paralelas a la recta de Euler, de punto del infinito X30.

  • jueves, 18 de abril del 2013

    In Memoriam (Juan Bosco Romero Márquez)

    El profesor Francisco Javier García Capitán le dedica el Problema 3721 de Crux Mathematicorum.
    tangente
    Addenda:
    Supongamos que D es un punto del plano del triángulo ABC y A' es el pie de la ceviana AD. Renombramos entonces los puntos P y Q del enunciado a Pa y Qa, respectivamente. Procediendo cíclicamente, definimos de forma análoga los puntos Pb y Pc, Qb y Qc.
    Entonces, las recta APa, BPb y CPc son concurrentes si y sólo si el punto D queda en la cúbica K003 (McCAY CUBIC, pK(X6, X3)) ó sobre la cúbica K006 ( ORTHOCUBIC, pK(X6, X4)).

    Las rectas PaQa, PbQb y PcQc son concurrentes si D está en la séxtica de ecuación baricéntrica:
    Σ [y z (b2 c2 x4 + 3 b2 c2 x3 y - a2 x2 y (-3 c2 y + (a2 - 2 b2) z))+ a4 y2 z2]=0.


  • martes, 16 de abril del 2013

    Puntos asociados al tercer punto de Brocard

    Sean ABC un triángulo, P un punto, P su conjugado isotómico y WaWbWc el triángulo ceviano del tercer punto de Brocard X76 (conjugado isotómico del simediano).
    Consideremos las cónicas circunscritas C(X76) de perspector X76 y la que pasa por X76 y P; denotemos por Q su cuarto punto de intersección y por QaQbQc su triángulo ceviano.
    Ahora, consideramos las cónicas:
    Ca tangente en Wa a BC y que pasa por Qb, Qc y Q,
    Cb tangente en Wb a CA y que pasa por Qc, Qa y Q,
    Cc tangente en Wc a AB y que pasa por Qa, Qb y Q.

    ( Mostrar/Ocultar figura )
      Tercer punto de Brocard
    Se verifica que las cuatro cónicas Ca, Cb y Cc y la cónica biceviana C(X76,Q) tienen en común el punto ω3(P), situado sobre la cónica inscrita de perspector X76, de coordenadas baricéntricas:
    ( a²(c²v - b²w)² : b²(c²u - a²w)² : c²(b²u - a²v)² ).
    Además, en este punto las tres cónicas Ca, Cb y Cc y la cónica inscrita de perspector X76 tienen recta tangente común.

    El punto ω3(P) es fijo para cualquier punto P sobre una recta que pasa por el simediano.
    Si P está en la recta X1X6, ω3(P)=X1086.
    Si P está en la recta X2X6, ω3(P)=X3124, "Danneels Perspector for X76".
    Si P está en la recta X3X6, ω3(P)=X338.
    Si P está en la recta X513X6, ω3(P)=X4437.

  • sábado, 13 de abril del 2013

    Cónica biceviana C(P,Q) pasando por P

    En un triángulo ABC, sean A1B1C1 y A2B2C2 los triángulos cevianos de los puntos P y Q, respectivamente. Por los seis puntos A1, B1, C1, A2, B2 y C2 pasa una cónica C(P,Q),cónica biceviana de P y Q. Supongamos que C(P,Q) pasa por P, entonces Q ha de estar en la cónica circunscrita de perspector P.

    Si P=(u:v:w) en coordenadas baricéntricas, un punto genérico de la cónica circunscrita de perspector P se puede poner en la forma:
    Q=(b²c²(b²u-a²v)(c²u-a²w)(a²b²+tuv)(a²c²+tuw): c²a²(c²v-b²w)(a²v-b²u)(b²c²+tvw)(b²a²+tvu): a²b²(a²w-c²u)(b²w-c²v)(c²a²+twu)(c²b²+twv)).

    Consideramos las cónicas:
    Ca tangente en A1 a BC y que pasa por B2, C2 y Q,
    Cb tangente en B1 a CA y que pasa por C2, A2 y Q,
    Cc tangente en C1 a AB y que pasa por A2, B2 y Q.

    Se verifica que las cuatro cónicas C(P,Q), Ca, Cb y Cc tienen en común el punto X de primera coordenada baricéntrica:
    u(-2a4b²c²vw + 2b²c²tu²vw + a²u(c²v+b²w)(b²c²-tvw))².

    Además, en este punto las tres cónicas Ca, Cb y Cc tiene como tangente común la recta de ecuación:
    tangente
    Un caso particular: Si P=X76 y t=0 entonces Q=X689. El punto común a las cuatro cónicas C(X76,X689), Ca, Cb y Cc es:
    (b2c2(b4 + c4 - 2a4 )2 : c2a2(a4 + c4 - 2b4)2 : a2b2(a4 + b4 - 2c4)2).


  • viernes, 12 de abril del 2013

    Cónica biceviana C(X76,X689)

    En un triángulo ABC, sean A1B1C1 y A2B2C2 los triángulos cevianos de los centros X76 y X689, respectivamente. Por los seis puntos A1, B1, C1, A2, B2 y C2 pasa una cónica C(X76,X689),cónica biceviana, que además pasa por X76; luego X689, al estar en la circunferencia circunscrita, es el cuarto punto de intersección de ésta con la cónica circunscrita de perspector X76.
    Consideramos las cónicas:
    Ca tangente en A1 a BC y que pasa por B2, C2 y X689,
    Cb tangente en B1 a CA y que pasa por C2, A2 y X689,
    Cc tangente en C1 a AB y que pasa por A2, B2 y X689.

    ( Mostrar/Ocultar figura )
      Cónica biceviana
    Se verifica que las cuatro cónicas C(X76,X689), Ca, Cb y Cc tienen en común el punto X de coordenadas baricéntricas:
    (b2c2(b4 + c4 - 2a4 )2 : c2a2(a4 + c4 - 2b4)2 : a2b2(a4 + b4 - 2c4)2).

    Además, en este punto las tres cónicas Ca, Cb y Cc tiene como tangente común la recta de coeficientes las coordenadas de X755 (recta G-Dual de X755).
    Generalización
    ¿Se tiene estos mismos resultados en otra cónica biceviana C(P,Q) que pasa por P?

    Las rectas que unen cada par de puntos de intersección de las cónicas Ca, Cb y Cc con los lados de ABC, distintos de A1, B1, C1, A2, B2 y C2, determinan un triángulo perspectivo con ABC, y su centro de perspectividad Y es:
    X?

    Propiedad geométrica de la cónica Ca:
    • Es el lugar geométrico del vértice D', del triángulo G-Dual D'E'F' de DEF (triángulo circunceviano de P), cuando P varía en la "hipérbola de Lemoine".
    • La tangente en el punto común a las cónicas Ca, Cb y Cc es la recta G-Dual de X755, cuarto punto de intersección de la circunferencia circunscrita y la "Hipérbola de Lemoine".

  • jueves, 11 de abril del 2013

    "Hipérbola de Lemoine"

    El lugar geométrico de los puntos P, tales que su triángulo circunceviano DEF y el triángulo G-Dual D'E'F' de éste, tienen como centro de perspectividad el centro X689, consta del Eje de Lemoine (tripolar del simediano) y de la hipérbola circunscrita (que podría denominarse "Hipérbola de Lemoine") de perspector X3005, punto de intersección del eje de Lemoine y la recta de De Longchamps (tripolar del conjugado isotómico del simediano).
    La "Hipérbola de Lemoine" contiene a los centros X(6), X(39), X(76), X(141), X(755), X(882), X(1843), X(2353) y X(3954)
    ( Mostrar/Ocultar figura (Caso Eje de Lemoine))
      Eje de Lemoine
    Para todo punto P sobre el Eje de Lemoine, el triángulo D'E'F' queda inscrito en la cónica circunscrita de perspector X76, que pasa por X689.
    El centro de perspectividad P'' de ABC y D'E'F' queda sobre recta de De Longchamps.

    ( Mostrar/Ocultar figura (Caso Hipérbola de Lemoine))
      Hipérbola de Lemoine
    La hipérbola a la que hemos denominado "Hipérbola de Lemoine", puede tener justificado su nombre por las siguientes propiedades:

    - Es la hipérbola circunscrita a ABC que pasa por el punto de Lemoine (simediano=X6) y por su conjugado isotómico (X76).
    - Su centro es X3124, "Danneels Perspector" para X6 (Eric Danneels.- "A Simple Perspectivity," Forum Geometricorum 6 (2006) 199-203).
    - Su perspector es X3005, punto de intersección del eje de Lemoine y de la polar trilineal del conjugado isotómico del simediano (recta de De Longchamps).
    - Para todo punto Y sobre la "Hipérbola de Lemoine" el centro de perspectividad de DEF y D'E'F' es el punto X689 (tripolo de la recta que pasa por X6 y X76).

    - Para todo punto Y sobre la "Hipérbola de Lemoine" el centro de perspectividad Y'' de ABC y D'E'F' queda sobre la cónica biceviana C(X76,X689) que pasa por los pies de las cevianas de X76 y X689; y como también pasa por X76 se verifica que X689 es el cuarto punto de intersección de la circunferencia circunscrita con la cónica circunscrita de perspector X76 (Bernard Gibert.- Bicevian Conics. Corollary 2. C(P,Q) also passes through P if and only if Q lies on the circum-conic with perspector P).

  • martes, 09 de abril del 2013

    Triángulo G-Dual

      Dado un triángulo ABC, para cada punto P de su plano se denota por dP la recta que tienen las mismas coordenadas baricéntricas que P, respecto a ABC. Si P no está en los lados de ABC, geométricamente dP es la tripolar del conjugado isotómico de P. Aunque algebraicamente dP siempre está bien definida. Diremos que dP es la recta G-Dual de P respecto al triángulo ABC.
       Cyril F. Parry en "The Isogonal Tripolar Conic" (Forum Geometricorum 1 (2001) 33–42) llama tripolar isotómica a la recta G-Dual.
       La recta G-Dual de P es la polar, respecto a la elipse circunscrita de Steiner, de la imagen de P mediante la homotecia de centro G y razón -2.
       En particular, si P está sobre una de las elipses de Steiner, su recta G-Dual es tangente a la otra elipse de Steiner.

       Dado un triángulo DEF, se considera el triángulo D'E'F' de vértices los puntos D'=dE∩dF, E'=dF∩dE y F'=dD∩dE. Diremos que D'E'F' es el triángulo G-Dual de DEF.

      Los triángulos DEF y D'E'F' son perspectivos. Además, si D, E y F están en una cónica circunscrita a ABC, el centro de perspectividad también está en la cónica.
      Si ABC y DEF son perspectivos, también son perspectivos ABC y D'E'F'.

    ( Mostrar/Ocultar figura)
      Triángulo G-Dual
    EJEMPLOS:
    • Sea DEF el triángulo circunceviano de un punto P=(u:v:w) y D'E'F' su triángulo G-Dual.
    El centro de perspectividad de DEF y D'E'F', que está en la circunferencia circunscrita a ABC, es:


    El centro de perspectividad de ABC y D'E'F' es P''=(c²v + b²w: c²u + a²w: b²u + a²v).
    ( Mostrar/Ocultar figura)
      Triángulo G-dual Circunceviano


    • Sea DEF el triángulo ceviano de un punto P=(u:v:w) y D'E'F' su triángulo G-Dual.
    .
    El centro de perspectividad de ABC y D'E'F' es el conjugado isotómico de P.

    El centro de perspectividad de DEF y D'E'F' es:
    (vw(u²v² + u²w² - v²w²): wu(v²w² + v²u² - w²u²): uv(w²u² + w²v² - u²v²).

    Algunos pares (X,Y) de centros de ETC, tales que Y es el centro de perspectividad del triángulo ceviano de X y su triángulo G-Dual:
    (X2,X2), (X75,X63), (X76,X22), (X85,X3870), (X86,X3995), (X99,X2), ...
  • lunes, 08 de abril del 2013

    Propiedad geométrica de X(2052)

    X2052

    ( Mostrar/Ocultar figura)
      Propiedad X(2052)
    Parábola determinada por su directriz y dos tangentes (dttC_p)

    Otra consideración:
    Sean Ba y Ca los puntos de contacto con los lados AC y AB de la parábola de directriz BC; similarmente, se define los puntos Cb, Ab Ac y Bc de contacto de las otras dos parábolas con los correspondientes lados.
    Los puntos Ba, Ca, Cb, Ab Ac y Bc están en una misma cónica.

    Los puntos A, CbAb∩AcBc, BaAb∩CaAc están en una recta.
    Los puntos B, AcBc∩BaCa, CbBc∩AbBa están en una recta.
    Los puntos C, BaCa∩CbAb, AcCa∩BcCb están en una recta.
    Estas tres rectas concurren en el centro de coordenadas baricéntricas:

    X=(1/(SA2(a4 - SBSC) - SB2SC2): ... : ... )

    con (6-9-13)-número de búsqueda en : 7.600894940587217504799796347 y es la reflexión del ortocentro en X8798 (centro de perspectividad de los triángulos de y del del ).   Es el punto de intersección de las rectas XiXj s.e.u.o., para los índices {i,j}: {4,8798}, {20,2979}, {64,1294}, {216,631}, {382,10152}, {1093,2972}, {5930,11362}.
  • domingo, 07 de abril del 2013

    Polar trilineal del punto de Gergonne

    Sea A' el punto de intersección de las tangentes en los vértices a las dos parábolas con foco en A y que pasan por B y C; los puntos B' y C' se definen similarmente. Los puntos A',B' y C' están el la polar trilineal del punto de Gergonne.
    ( Mostrar/Ocultar figura)
      Tripolar_del_punto_de_Gergonne
    Parábolas dadas por el foco y dos puntos (FPPC_p)
Página Personal